Sei sulla pagina 1di 470

REPASO

1. Let f(x) = 7 – 2x and g(x) = x + 3.


(a) Find (g ° f)(x).
(2)
(b) Write down g–1(x).
(1)
–1
(c) Find (f ° g )(5).
(2)
(Total 5 marks)
2. Consider f(x) = 2kx2 – 4kx + 1, for k ≠ 0. The equation f(x) = 0 has two equal roots.
(a) Find the value of k.
(5)
(b) The line y = p intersects the graph of f. Find all possible values of p.
(2)
(Total 7 marks)
3. The velocity v m s–1 of a particle at time t seconds, is given by v = 2t + cos2t, for 0 ≤ t ≤ 2.
(a) Write down the velocity of the particle when t = 0.
(1)
When t = k, the acceleration is zero.
π
(b) (i) Show that k = .
4
π
(ii) Find the exact velocity when t = .
4
(8)
π dv π dv
(c) When t < , > 0 and when t > , > 0.
4 dt 4 dt
Sketch a graph of v against t.
(4)
(d) Let d be the distance travelled by the particle for 0 ≤ t ≤ 1.
(i) Write down an expression for d.
(ii) Represent d on your sketch.
(3)
(Total 16 marks)
4. Let f(x) = 3 ln x and g(x) = ln 5x3.
(a) Express g(x) in the form f(x) + ln a, where a  +
.
(4)

(b) The graph of g is a transformation of the graph of f. Give a full geometric description of
this transformation.
(3)
(Total 7 marks)
5. The following diagram shows part of the graph of a quadratic function f.

The x-intercepts are at (–4, 0) and (6, 0) and the y-intercept is at (0, 240).
(a) Write down f(x) in the form f(x) = –10(x – p)(x – q).
(2)
(b) Find another expression for f(x) in the form f(x) = –10(x – h)2 + k.
(4)
2
(c) Show that f(x) can also be written in the form f(x) = 240 + 20x – 10x .
(2)
–1
A particle moves along a straight line so that its velocity, v m s , at time t seconds is given by
v = 240 + 20t – 10t2, for 0 ≤ t ≤ 6.
(d) (i) Find the value of t when the speed of the particle is greatest.
(ii) Find the acceleration of the particle when its speed is zero.
(7)
(Total 15 marks)
6. Let f(x) = 3x2. The graph of f is translated 1 unit to the right and 2 units down.
The graph of g is the image of the graph of f after this translation.
(a) Write down the coordinates of the vertex of the graph of g.
(2)
2
(b) Express g in the form g(x) = 3(x – p) + q.
(2)
The graph of h is the reflection of the graph of g in the x-axis.
(c) Write down the coordinates of the vertex of the graph of h.
(2)
(Total 6 marks)
7. A company uses two machines, A and B, to make boxes. Machine A makes 60 % of the boxes.
80 % of the boxes made by machine A pass inspection.
90 % of the boxes made by machine B pass inspection.
A box is selected at random.
(a) Find the probability that it passes inspection.
(3)
(b) The company would like the probability that a box passes inspection to be 0.87.
Find the percentage of boxes that should be made by machine B to achieve this.
(4)
(Total 7 marks)
x 2
8. The following diagram shows the graph of f(x) = e .

The points A, B, C, D and E lie on the graph of f. Two of these are points of inflexion.
(a) Identify the two points of inflexion.
(2)
(b) (i) Find f′(x).
 x2
(ii) Show that f″(x) = (4x2 – 2) e .
(5)
(c) Find the x-coordinate of each point of inflexion.
(4)
(d) Use the second derivative to show that one of these points is a point of inflexion.
(4)
(Total 15 marks)
x
9. Let f(x) = log3 + log3 16 – log3 4, for x > 0.
2
(a) Show that f(x) = log3 2x.
(2)
(b) Find the value of f(0.5) and of f(4.5).
(3)
ln ax
The function f can also be written in the form f(x) = .
ln b
(c) (i) Write down the value of a and of b.
(ii) Hence on graph paper, sketch the graph of f, for –5 ≤ x ≤ 5, –5 ≤ y ≤ 5, using a
scale of 1 cm to 1 unit on each axis.
(iii) Write down the equation of the asymptote.
(6)
–1
(d) Write down the value of f (0).
(1)
The point A lies on the graph of f. At A, x = 4.5.
(e) On your diagram, sketch the graph of f–1, noting clearly the image of point A.
(4)
(Total 16 marks)
10. Let f(x) = 3x, g(x) = 2x – 5 and h(x) = (f ° g)(x).
(a) Find h(x).
(2)
–1
(b) Find h (x).
(3)
(Total 5 marks)
1
11. Let g(x) = x sin x, for 0 ≤ x ≤ 4.
2
(a) Sketch the graph of g on the following set of axes.

(4)
(b) Hence find the value of x for which g(x) = –1.
(2)
(Total 6 marks)
12. Let f(x) = 8x – 2x2. Part of the graph of f is shown below.

(a) Find the x-intercepts of the graph.


(4)
(b) (i) Write down the equation of the axis of symmetry.
(ii) Find the y-coordinate of the vertex.
(3)
(Total 7 marks)
13. Let f(x) = log3 x , for x > 0.
(a) Show that f–1(x) = 32x.
(2)
–1
(b) Write down the range of f .
(1)
Let g(x) = log3 x, for x > 0.
(c) Find the value of (f –1 ° g)(2), giving your answer as an integer.
(4)
(Total 7 marks)
1 3
14. Let f(x) = x  x 2  3x . Part of the graph of f is shown below.
3

There is a maximum point at A and a minimum point at B(3, –9).


(a) Find the coordinates of A.
(8)
(b) Write down the coordinates of
(i) the image of B after reflection in the y-axis;
  2
(ii) the image of B after translation by the vector   ;
 5 
(iii) the image of B after reflection in the x-axis followed by a horizontal stretch with
1
scale factor .
2
(6)
(Total 14 marks)
15. Let f(x) = p(x – q)(x – r). Part of the graph of f is shown below.

The graph passes through the points (–2, 0), (0, –4) and (4, 0).
(a) Write down the value of q and of r.
(2)
(b) Write down the equation of the axis of symmetry.
(1)
(c) Find the value of p.
(3)
(Total 6 marks)
16. Let f(x) = cos 2x and g(x) = 2x2 – 1.
π
(a) Find f   .
2
(2)
π
(b) Find (g ° f)   .
2
(2)
(c) Given that (g ° f)(x) can be written as cos (kx), find the value of k, k  .
(3)
(Total 7 marks)
17. Solve log2x + log2(x – 2) = 3, for x > 2.
(Total 7 marks)
18. Let f(x) = 6 + 6sinx. Part of the graph of f is shown below.

The shaded region is enclosed by the curve of f, the x-axis, and the y-axis.
(a) Solve for 0 ≤ x < 2π.
(i) 6 + 6sin x = 6;
(ii) 6 + 6 sin x = 0.
(5)
(b) Write down the exact value of the x-intercept of f, for 0 ≤ x < 2.
(1)
(c) The area of the shaded region is k. Find the value of k, giving your answer in terms of π.
(6)
 π
Let g(x) = 6 + 6sin  x   . The graph of f is transformed to the graph of g.
 2
(d) Give a full geometric description of this transformation.
(2)

p
(e) Given that p
2 g ( x)dx = k and 0 ≤ p < 2π, write down the two values of p.
(3)
(Total 17 marks)
19. The diagram below shows a quadrilateral ABCD with obtuse angles AB̂C and AD̂C .

diagram not to scale


AB = 5 cm, BC = 4 cm, CD = 4 cm, AD = 4 cm, BÂC = 30°, AB̂C = x°, AD̂C = y°.
(a) Use the cosine rule to show that AC = 41 40 cos x .
(1)
(b) Use the sine rule in triangle ABC to find another expression for AC.
(2)
(c) (i) Hence, find x, giving your answer to two decimal places.
(ii) Find AC.
(6)
(d) (i) Find y.
(ii) Hence, or otherwise, find the area of triangle ACD.
(5)
(Total 14 marks)
20. Let f(x) = Aekx + 3. Part of the graph of f is shown below.

The y-intercept is at (0, 13).


(a) Show that A =10.
(2)
(b) Given that f(15) = 3.49 (correct to 3 significant figures), find the value of k.
(3)
(c) (i) Using your value of k, find f′(x).
(ii) Hence, explain why f is a decreasing function.
(iii) Write down the equation of the horizontal asymptote of the graph f.
(5)
2
Let g(x) = –x + 12x – 24.
(d) Find the area enclosed by the graphs of f and g.
(6)
(Total 16 marks)
21. Consider f(x) = 2 – x2, for –2 ≤ x ≤ 2 and g(x) = sin ex, for –2 ≤ x ≤ 2. The graph of f is given
below.

(a) On the diagram above, sketch the graph of g.


(3)
(b) Solve f(x) = g(x).
(2)
(c) Write down the set of values of x such that f(x) > g(x).
(2)
(Total 7 marks)
22. Let f(x) = ex sin 2x + 10, for 0 ≤ x ≤ 4. Part of the graph of f is given below.
There is an x-intercept at the point A, a local maximum point at M, where x = p and a local
minimum point at N, where x = q.
(a) Write down the x-coordinate of A.
(1)
(b) Find the value of
(i) p;
(ii) q.
(2)
q
(c) Find p
f ( x)dx . Explain why this is not the area of the shaded region.
(3)
(Total 6 marks)
23. The number of bacteria, n, in a dish, after t minutes is given by n = 800e013t.
(a) Find the value of n when t = 0.
(2)
(b) Find the rate at which n is increasing when t = 15.
(2)
(c) After k minutes, the rate of increase in n is greater than 10 000 bacteria per minute. Find
the least value of k, where k  .
(4)
(Total 8 marks)
24. Consider f(x) = x ln(4 – x2), for –2 < x < 2. The graph of f is given below.

(a) Let P and Q be points on the curve of f where the tangent to the graph of f is parallel to
the x-axis.
(i) Find the x-coordinate of P and of Q.
(ii) Consider f(x) = k. Write down all values of k for which there are exactly two
solutions.
(5)
Let g(x) = x3 ln(4 – x2), for –2 < x < 2.
 2x 4
(b) Show that g′(x) =  3x 2 ln( 4  x 2 ) .
4 x 2

(4)
(c) Sketch the graph of g′.
(2)
(d) Consider g′(x) = w. Write down all values of w for which there are exactly two solutions.
(3)
(Total 14 marks)
25. Let f(x) = x2 + 4 and g(x) = x – 1.
(a) Find (f ° g)(x).
(2)
 3
The vector   translates the graph of (f ° g) to the graph of h.
 1
(b) Find the coordinates of the vertex of the graph of h.
(3)
2
(c) Show that h(x) = x – 8x + 19.
(2)
(d) The line y = 2x – 6 is a tangent to the graph of h at the point P. Find the x-coordinate of P.
(5)
(Total 12 marks)
26. Consider the function f(x) = px3 + qx2 + rx. Part of the graph of f is shown below.

The graph passes through the origin O and the points A(–2, –8), B(1, –2) and C(2, 0).
(a) Find three linear equations in p, q and r.
(4)
(b) Hence find the value of p, of q and of r.
(3)
(Total 7 marks)
27. Let f(x) = x ln(4 – x2), for –2 < x < 2. The graph of f is shown below.
The graph of f crosses the x-axis at x = a, x = 0 and x = b.
(a) Find the value of a and of b.
(3)
The graph of f has a maximum value when x = c.
(b) Find the value of c.
(2)
(c) The region under the graph of f from x = 0 to x = c is rotated 360° about the x-axis. Find
the volume of the solid formed.
(3)
(d) Let R be the region enclosed by the curve, the x-axis and the line x = c, between x = a and
x = c.
Find the area of R.
(4)
(Total 12 marks)
28. Let f(x) = x2 and g(x) = 2(x – 1)2.
(a) The graph of g can be obtained from the graph of f using two transformations.
Give a full geometric description of each of the two transformations.
(2)
 3 
(b) The graph of g is translated by the vector   to give the graph of h.
  2
The point (–1, 1) on the graph of f is translated to the point P on the graph of h.
Find the coordinates of P.
(4)
(Total 6 marks)
29. Let f(x) = ex+3.
(a) (i) Show that f–1(x) = ln x – 3.
(ii) Write down the domain of f–1.
(3)
1
(b) Solve the equation f–1(x) = ln   .
 x
(4)
(Total 7 marks)
ax
30. Let f(x) = , –8 ≤ x ≤ 8, a  . The graph of f is shown below.
x 1
2

The region between x = 3 and x = 7 is shaded.


(a) Show that f(–x) = –f(x).
(2)
2ax( x  3)
2
(b) Given that f′′(x) = , find the coordinates of all points of inflexion.
( x 2  1) 3
(7)
a
 f ( x)dx  2 ln( x  1)  C .
2
(c) It is given that
(i) Find the area of the shaded region, giving your answer in the form p ln q.
8
(ii) Find the value of  2 f ( x  1)dx .
4
(7)
(Total 16 marks)
31. Let f(x) = x2 and g(x) = 2x – 3.
(a) Find g–1(x).
(2)
(b) Find (f ° g)(4).
(3)
(Total 5 marks)
π 1
32. Let f(x) = 3e 2 x sin x + e2x cos x, for 0 ≤ x ≤ π. Given that tan  , solve the equation
6 3
f(x) = 0.
(Total 6 marks)
3x  x
33. Let f(x) = +1, g(x) = 4cos   – 1. Let h(x) = (g ° f)(x).
2 3
(a) Find an expression for h(x).
(3)
(b) Write down the period of h.
(1)
(c) Write down the range of h.
(2)
(Total 6 marks)
34. Let f(x) = ax2 + bx + c where a, b and c are rational numbers.
(a) The point P(–4, 3) lies on the curve of f. Show that 16a –4b + c = 3.
(b) The points Q(6, 3) and R(–2, –1) also lie on the curve of f. Write down two other linear
equations in a, b and c.
(2)
(c) These three equations may be written as a matrix equation in the form AX = B,
a
 
where X =  b  .
c
 
(i) Write down the matrices A and B.
(ii) Write down A–1.
(iii) Hence or otherwise, find f(x).
(8)
2
(d) Write f(x) in the form f(x) = a(x – h) + k, where a, h and k are rational numbers.
(3)
(Total 15 marks)
35. Consider the graph of f shown below.

(a) On the same grid sketch the graph of y = f(–x).


(2)
The following four diagrams show images of f under different transformations.

(b) Complete the following table.


Description of transformation Diagram letter
Horizontal stretch with scale factor 1.5
Maps f to f(x) + 1
(2)
(c) Give a full geometric description of the transformation that gives the image in
Diagram A.
(2)
(Total 6 marks)
36. Solve the equation ex = 4 sin x, for 0 ≤ x ≤ 2π.
(Total 5 marks)
37. The quadratic equation kx2 + (k – 3)x + 1 = 0 has two equal real roots.
(a) Find the possible values of k.
(5)
(b) Write down the values of k for which x2 + (k – 3)x + k = 0 has two equal real roots.
(2)
(Total 7 marks)
38. Let f(x) = 2x3 + 3 and g(x) = e3x – 2.
(a) (i) Find g(0).
(ii) Find (f ° g)(0).
(5)
(b) Find f–1(x).
(3)
(Total 8 marks)
39. The diagram below shows the graph of a function f(x), for –2 ≤ x ≤ 4.
(a) Let h(x) = f(–x). Sketch the graph of h on the grid below.

(2)
1
(b) Let g(x) = f(x – 1). The point A(3, 2) on the graph of f is transformed to the point P on
2
the graph of g. Find the coordinates of P.
(3)
(Total 5 marks)
40. Let f(x) = k log2 x.
(a) Given that f–1(1) = 8, find the value of k.
(3)
2
(b) Find f–1   .
3
(4)
(Total 7 marks)
20
41. Let f(x) = 3 + , for x ≠ ±2. The graph of f is given below.
x 4
2
diagram not to scale
The y-intercept is at the point A.
(a) (i) Find the coordinates of A.
(ii) Show that f′(x) = 0 at A.
(7)
40(3x  4)
2
(b) The second derivative f′′(x) = . Use this to
( x 2  4) 3
(i) justify that the graph of f has a local maximum at A;
(ii) explain why the graph of f does not have a point of inflexion.
(6)
(c) Describe the behaviour of the graph of f for large │x│.
(1)
(d) Write down the range of f.
(2)
(Total 16 marks)
π
42. Let f(x) = 5 cos x and g(x) = –0.5x2 + 5x – 8, for 0 ≤ x ≤ 9.
4
(a) On the same diagram, sketch the graphs of f and g.
(3)
(b) Consider the graph of f. Write down
(i) the x-intercept that lies between x = 0 and x =3;
(ii) the period;
(iii) the amplitude.
(4)
(c) Consider the graph of g. Write down
(i) the two x-intercepts;
(ii) the equation of the axis of symmetry.
(3)
(d) Let R be the region enclosed by the graphs of f and g. Find the area of R.
(5)
(Total 15 marks)
43. Let f (x) = ln (x + 5) + ln 2, for x  –5.
(a) Find f −1(x).
(4)
x
Let g (x) = e .
(b) Find (g ◦ f) (x), giving your answer in the form ax + b, where a, b, .
(3)
(Total 7 marks)
44. Let f (x) = 3(x + 1)2 – 12.
(a) Show that f (x) = 3x2 + 6x – 9.
(2)
(b) For the graph of f
(i) write down the coordinates of the vertex;
(ii) write down the equation of the axis of symmetry;
(iii) write down the y-intercept;
(iv) find both x-intercepts.
(8)
(c) Hence sketch the graph of f.
(2)
2
(d) Let g (x) = x . The graph of f may be obtained from the graph of g by the two
transformations:
a stretch of scale factor t in the y-direction
followed by
 p
a translation of  .
q
 p
Find   and the value of t.
q
(3)
(Total 15 marks)
45. The following diagram shows part of the graph of f, where f (x) = x2 − x − 2.

(a) Find both x-intercepts.


(4)
(b) Find the x-coordinate of the vertex.
(2)
(Total 6 marks)
46. Part of the graph of a function f is shown in the diagram below.
y

–2 –1 0 1 2 3 4 x

–1

–2

–3

–4

(a) On the same diagram sketch the graph of y = − f (x).


(2)
(b) Let g (x) = f (x + 3).
(i) Find g (−3).
(ii) Describe fully the transformation that maps the graph of f to the graph of g.
(4)
(Total 6 marks)
π π
47. Let f (x) = 4 tan2 x – 4 sin x,  x .
3 3
(a) On the grid below, sketch the graph of y = f (x).

(3)
(b) Solve the equation f (x) = 1.
(3)
(Total 6 marks)
48. Let f (x) = 3x – ex–2 – 4, for –1  x  5.
(a) Find the x-intercepts of the graph of f.
(3)
(b) On the grid below, sketch the graph of f.
y
3
2
1

x
–2 –1 0 1 2 3 4 5 6
–1
–2
–3
–4
–5
–6
–7
–8
–9
–10

(3)
(c) Write down the gradient of the graph of f at x = 2.
(1)
(Total 7 marks)
49. A city is concerned about pollution, and decides to look at the number of people using taxis. At
the end of the year 2000, there were 280 taxis in the city. After n years the number of taxis, T, in
the city is given by
T = 280  1.12n.
(a) (i) Find the number of taxis in the city at the end of 2005.
(ii) Find the year in which the number of taxis is double the number of taxis there were
at the end of 2000.
(6)
(b) At the end of 2000 there were 25 600 people in the city who used taxis. After n years the
number of people, P, in the city who used taxis is given by
2 560 000
P= .
10  90e – 0.1n
(i) Find the value of P at the end of 2005, giving your answer to the nearest whole
number.
(ii) After seven complete years, will the value of P be double its value at the end of
2000? Justify your answer.
(6)
(c) Let R be the ratio of the number of people using taxis in the city to the number of taxis.
The city will reduce the number of taxis if R  70.
(i) Find the value of R at the end of 2000.
(ii) After how many complete years will the city first reduce the number of taxis?
(5)
(Total 17 marks)
50. Let f be the function given by f(x) = e0.5x, 0 ≤ x ≤ 3.5. The diagram shows the graph of f.

(a) On the same diagram, sketch the graph of f–1.


(3)
–1
(b) Write down the range of f .
(1)
(c) Find f–1(x).
(3)
(Total 7 marks)
51. Let f(t) = a cos b (t – c) + d, t ≥ 0. Part of the graph of y = f(t) is given below.
When t = 3, there is a maximum value of 29, at M.
When t = 9 , there is a minimum value of 15.
(a) (i) Find the value of a.
π
(ii) Show that b = .
6
(iii) Find the value of d.
(iv) Write down a value for c.
(7)
1
The transformation P is given by a horizontal stretch of a scale factor of , followed by a
2
 3 
translation of   .
 10 
(b) Let M′ be the image of M under P. Find the coordinates of M′.
(2)
The graph of g is the image of the graph of f under P.
(c) Find g(t) in the form g(t) = 7 cos B(t – C) + D.
(4)
(d) Give a full geometric description of the transformation that maps the graph of g to the
graph of f.
(3)
(Total 16 marks)
52. Let f(x) = 2x2 + 4x – 6.
(a) Express f(x) in the form f(x) = 2(x – h)2 + k.
(3)
(b) Write down the equation of the axis of symmetry of the graph of f.
(1)
(c) Express f(x) in the form f(x) = 2(x – p)(x – q).
(2)
(Total 6 marks)
53. Let f(x) = x cos (x – sin x), 0 ≤ x ≤ 3.
(a) Sketch the graph of f on the following set of axes.

(3)
(b) The graph of f intersects the x-axis when x = a, a ≠ 0. Write down the value of a.
(1)
(c) The graph of f is revolved 360° about the x-axis from x = 0 to x = a.
Find the volume of the solid formed.
(4)
(Total 8 marks)
54. Consider f(x) = x  5 .
(a) Find
(i) f(11);
(ii) f(86);
(iii) f(5).
(3)
(b) Find the values of x for which f is undefined.
(2)
(c) Let g(x) = x2. Find (g ° f)(x).
(2)
(Total 7 marks)
55. The quadratic function f is defined by f(x) = 3x2 – 12x + 11.
(a) Write f in the form f(x) = 3(x – h)2 – k.
(3)
(b) The graph of f is translated 3 units in the positive x-direction and 5 units in the positive
y-direction. Find the function g for the translated graph, giving your answer in the form
g(x) = 3(x – p)2 + q.
(3)
(Total 6 marks)
 2  1 0 0
56. Let M =   , and O =   . Given that M2 – 6M + kI = O, find k.
3 4  0 0
(Total 6 marks)
57. Solve the following equations.
(a) logx 49 = 2
(3)
(b) log2 8 = x
(2)
1
(c) log25 x = 
2
(3)
(d) log2 x + log2(x – 7) = 3
(5)
2
58. Let f (x) = 2x – 12x + 5.
(a) Express f(x) in the form f(x) = 2(x – h)2 – k.
(3)
(b) Write down the vertex of the graph of f.
(2)
(c) Write down the equation of the axis of symmetry of the graph of f.
(1)
(d) Find the y-intercept of the graph of f.
(2)
p q
(e) The x-intercepts of f can be written as , where p, q, r  .
r
Find the value of p, of q, and of r.
(7)
(Total 15 marks)
1
59. Let f(x) = , x ≠ 0.
x
(a) Sketch the graph of f.
(2)
 2
The graph of f is transformed to the graph of g by a translation of   .
 3
(b) Find an expression for g(x).
(2)
(c) (i) Find the intercepts of g.
(ii) Write down the equations of the asymptotes of g.
(iii) Sketch the graph of g.
(10)
(Total 14 marks)
3
60. The function f is defined by f(x) = , for –3 < x < 3.
9  x2
(a) On the grid below, sketch the graph of f.
(2)
(b) Write down the equation of each vertical asymptote.
(2)
c) Write down the range of the function f.
(2)
(Total 6 marks)
61. The functions f and g are defined by f : x  3x, g : x  x + 2.
(a) Find an expression for (f ° g)(x).
(2)
–1 –1
(b) Find f (18) + g (18).
(4)
(Total 6 marks)
62. A farmer owns a triangular field ABC. One side of the triangle, [AC], is 104 m, a second side,
[AB], is 65 m and the angle between these two sides is 60°.
(a) Use the cosine rule to calculate the length of the third side of the field.
(3)
3
(b) Given that sin 60°= , find the area of the field in the form 3 p 3 where p is an
2
integer.
(3)
Let D be a point on [BC] such that [AD] bisects the 60° angle. The farmer divides the field into
two parts A1 and A2 by constructing a straight fence [AD] of length x metres, as shown on the
diagram below.
65x
(c) (i) Show that the area of A1 is given by .
4
(ii) Find a similar expression for the area of A2.
(iii) Hence, find the value of x in the form q 3 , where q is an integer.
(7)
(d) (i) Explain why sinA D̂C  sinA D̂B .
BD 5
(ii) Use the result of part (i) and the sine rule to show that  .
DC 8
(5)
(Total 18 marks)

63. The functions f (x) and g (x) are defined by f (x) = ex and g (x) = ln (1+ 2x).
(a) Write down f −1(x).
(b) (i) Find ( f ◦ g) (x).
(ii) Find ( f ◦ g)−1 (x).
(Total 6 marks)
64. The graph of the function y = f (x), 0  x  4, is shown below.

(a) Write down the value of


(i) f ′ (1);
(ii) f ′ (3).
(b) On the diagram below, draw the graph of y = 3 f (−x).

(c) On the diagram below, draw the graph of y = f (2x).

(Total 6 marks)

65. (a) Given that (2x)2 + (2x) −12 can be written as (2x + a)(2x + b), where a, b  , find the
value of a and of b.
(b) Hence find the exact solution of the equation (2x)2 + (2x) −12 = 0, and explain why there
is only one solution.
(Total 6 marks)

66. The population of a city at the end of 1972 was 250 000. The population increases by 1.3 per
year.
(a) Write down the population at the end of 1973.
(b) Find the population at the end of 2002.
(Total 6 marks)
67. Let f (x) = x  4 , x  − 4 and g (x) = x2, x  .
(a) Find (g ◦ f ) (3).
(b) Find f −1(x).
(c) Write down the domain of f −1.
(Total 6 marks)

68. Consider two different quadratic functions of the form f (x) = 4x2 − qx + 25. The graph of each
function has its vertex on the x-axis.
(a) Find both values of q.
(b) For the greater value of q, solve f (x) = 0.
(c) Find the coordinates of the point of intersection of the two graphs.
(Total 6 marks)
69. Let f (x) = ln (x + 2), x  −2 and g (x) = e(x−4), x  0.
(a) Write down the x-intercept of the graph of f.
(b) (i) Write down f (−1.999).
(ii) Find the range of f.
(c) Find the coordinates of the point of intersection of the graphs of f and g.
(Total 6 marks)

70. The graph of a function f is shown in the diagram below. The point A (–1, 1) is on the graph,
and y = −1 is a horizontal asymptote.

(a) Let g (x) = f (x −1) + 2. On the diagram, sketch the graph of g.


(b) Write down the equation of the horizontal asymptote of g.
(c) Let A′ be the point on the graph of g corresponding to point A. Write down the
coordinates of A′.
(Total 6 marks)
71. The following diagram shows part of the graph of a quadratic function, with equation in the
form y = (x − p)(x − q), where p, q  .

(a) Write down


(i) the value of p and of q;
(ii) the equation of the axis of symmetry of the curve.
(3)

(b) Find the equation of the function in the form y = (x − h)2 + k, where h, k  .
(3)

dy
(c) Find .
dx
(2)

(d) Let T be the tangent to the curve at the point (0, 5). Find the equation of T.
(2)
(Total 10 marks)

1 5
72. The function f (x) is defined as f (x) = 3 + ,x  .
2x  5 2
(a) Sketch the curve of f for −5  x  5, showing the asymptotes.
(3)

(b) Using your sketch, write down


(i) the equation of each asymptote;
(ii) the value of the x-intercept;
(iii) the value of the y-intercept.
(4)

(c) The region enclosed by the curve of f, the x-axis, and the lines x = 3 and x = a, is revolved
through 360 about the x-axis. Let V be the volume of the solid formed.
 6 1 
(i)


Find  9    dx.
2 
2 x  5 2 x  5 
 28 
(ii) Hence, given that V = π  3 ln 3  , find the value of a.
 3 
(10)
(Total 17 marks)
3x
73. Let f (x) = p  , where p, q +.
x  q2
2

Part of the graph of f, including the asymptotes, is shown below.

(a) The equations of the asymptotes are x =1, x = −1, y = 2. Write down the value of
(i) p;
(ii) q.
(2)

(b) Let R be the region bounded by the graph of f, the x-axis, and the y-axis.
(i) Find the negative x-intercept of f.
(ii) Hence find the volume obtained when R is revolved through 360 about the x-axis.
(7)

Show that f ′ (x) =



3 x 2 1 .
x 
(c) (i) 2
1 2

(ii) Hence, show that there are no maximum or minimum points on the graph of f.
(8)

(d) Let g (x) = f ′ (x). Let A be the area of the region enclosed by the graph of g and the x-
axis, between x = 0 and x = a, where a  0. Given that A = 2, find the value of a.
(7)
(Total 24 marks)

74. Let f (x) = loga x, x  0.


(a) Write down the value of
(i) f (a);
(ii) f (1);
(iii) f (a4 ).
(3)

(b) The diagram below shows part of the graph of f.


y

1
f

–2 –1 0 1 2 x

–1

–2

On the same diagram, sketch the graph of f−1.


(3)
(Total 6 marks)
75. The following diagram shows part of the graph of f (x) = 5 − x2 with vertex V (0, 5).
h
Its image y = g (x) after a translation with vector   has vertex T (3, 6).
k 

(a) Write down the value of


(i) h;
(ii) k.
(2)

(b) Write down an expression for g (x).


(2)

(c) On the same diagram, sketch the graph of y = g (−x).


(2)
(Total 6 marks)
76. The area A km2 affected by a forest fire at time t hours is given by A = A0 ekt.
When t = 5, the area affected is 1 km2 and the rate of change of the area is 0.2 km2 h−1.
(a) Show that k = 0.2.
(4)
1
(b) Given that A0 = , find the value of t when 100 km2 are affected.
e
(2)
(Total 6 marks)

 5  1
77. Consider the function f (x) e(2x–1) +   , x  .
 2 x 1  2
(a) Sketch the curve of f for −2  x  2, including any asymptotes.
(3)

(b) (i) Write down the equation of the vertical asymptote of f.


(ii) Write down which one of the following expressions does not represent an area
between the curve of f and the x-axis.
2
1
f (x)dx
2
0
f (x)dx
(iii) Justify your answer.
(3)

(c) The region between the curve and the x-axis between x = 1 and x = 1.5 is rotated through
360 about the x-axis. Let V be the volume formed.
(i) Write down an expression to represent V.
(ii) Hence write down the value of V.
(4)

(d) Find f ′ (x).


(4)

(e) (i) Write down the value of x at the minimum point on the curve of f.
(ii) The equation f (x) = k has no solutions for p  k  q. Write down the value of p and
of q.
(3)
(Total 17 marks)

78. (a) Consider the equation 4x2 + kx + 1 = 0. For what values of k does this equation have two
equal roots?
(3)

Let f be the function f ( ) = 2 cos 2 + 4 cos  + 3, for −360    360.


(b) Show that this function may be written as f ( ) = 4 cos2  + 4 cos  + 1.
(1)

(c) Consider the equation f ( ) = 0, for −360    360.


(i) How many distinct values of cos  satisfy this equation?
(ii) Find all values of  which satisfy this equation.
(5)

(d) Given that f ( ) = c is satisfied by only three values of , find the value of c.
(2)
(Total 11 marks)
79. The following diagram shows part of the graph of f (x).

Consider the five graphs in the diagrams labelled A, B, C, D, E below.

(a) Which diagram is the graph of f (x + 2) ?

(b) Which diagram is the graph of – f (x) ?

(c) Which diagram is the graph of f (–x)


(Total 6 marks)

80. Let f (x) = a (x − 4)2 + 8.


(a) Write down the coordinates of the vertex of the curve of f.
(b) Given that f (7) = −10, find the value of a.
(c) Hence find the y-intercept of the curve of f.
(Total 6 marks)

81. Let f (x) = x3 − 4 and g (x) = 2x.


(a) Find (g ◦ f ) (−2).
(b) Find f −1 (x).
(Total 6 marks)

82. Let ƒ (x) = 3 sin 2x, for 1  x  4 and g (x) = −5x2 + 27x − 35 for 1  x  4. The graph of ƒ is
shown below.

(a) On the same diagram, sketch the graph of g.


(b) One solution of ƒ (x) = g (x) is 1.89. Write down the other solution.

(c) Let h (x) = g (x) − ƒ (x). Given that h (x)  0 for p  x  q, write down the value of p and
of q.
(Total 6 marks)

83. Solve the following equations.


(a) ln (x + 2) = 3.
(b) 102x = 500.
(Total 6 marks)

84. (a) Express y = 2x2 – 12x + 23 in the form y = 2(x – c)2 + d.


The graph of y = x2 is transformed into the graph of y = 2x2 – 12x + 23 by the transformations
a vertical stretch with scale factor k followed by
a horizontal translation of p units followed by
a vertical translation of q units.
(b) Write down the value of
(i) k;
(ii) p;
(iii) q.
(Total 6 marks)

85. Consider the functions f and g where f (x) = 3x – 5 and g (x) = x – 2.


(a) Find the inverse function, f −1.
(3)

(b) Given that g–1 (x) = x + 2, find (g–1 ◦ f) (x).


(2)

x3
(c) Given also that (f −1 ◦ g) (x) , solve (f −1 ◦ g) (x) = (g–1 ◦ f) (x).
3
(2)

f ( x)
Let h (x) = , x  2.
g ( x)
(d) (i) Sketch the graph of h for −3  x  7 and −2  y  8, including any asymptotes.
(ii) Write down the equations of the asymptotes.
(5)

3x  5 1
(e) The expression may also be written as 3 + . Use this to answer the
x 3 x2
following.
(i) Find  h (x) dx.
5
(ii) Hence, calculate the exact value of  3
h (x)dx.
(5)
5
(f) On your sketch, shade the region whose area is represented by  3
h (x)dx.
(1)
(Total 18 marks)
86. (a) Let y = –16x2 + 160x –256. Given that y has a maximum value, find
(i) the value of x giving the maximum value of y;
(ii) this maximum value of y.
The triangle XYZ has XZ = 6, YZ = x, XY = z as shown below. The perimeter of triangle XYZ
is 16.
(4)

(b) (i) Express z in terms of x.


(ii) Using the cosine rule, express z2 in terms of x and cos Z.
5 x 16
(iii) Hence, show that cos Z = .
3x
(7)
Let the area of triangle XYZ be A.

(c) Show that A2 = 9x2 sin2 Z.


(2)

(d) Hence, show that A2 = –16x2 + 160x – 256.


(4)
(e) (i) Hence, write down the maximum area for triangle XYZ.
(ii) What type of triangle is the triangle with maximum area?
(3)
(Total 20 marks)

87. Part of the graph of the function y = d (x −m)2 + p is given in the diagram below.
The x-intercepts are (1, 0) and (5, 0). The vertex is V(m, 2).

(a) Write down the value of


(i) m;
(ii) p.
(b) Find d.
(Total 6 marks)

88. Find the exact value of x in each of the following equations.


(a) 5x+1 = 625

(b) loga (3x + 5) = 2


(Total 6 marks)
5x
89. Let g (x) = 3x – 2, h (x) = , x  4.
x4
(a) Find an expression for (h ◦ g) (x). Simplify your answer.

(b) Solve the equation (h ◦ g) (x) = 0.


(Total 6 marks)

90. The function f is given by f (x) = mx3 + nx2 + px + q, where m, n, p, q are integers.
The graph of f passes through the point (0, 0).
(a) Write down the value of q.
(1)

The graph of f also passes through the point (3, 18).


(b) Show that 27 m+ 9n + 3p =18.
The graph of f also passes through the points (1, 0) and (–1, –10).
(2)

(c) Write down the other two linear equations in m, n and p.


(2)

(d) (i) Write down these three equations as a matrix equation.


(ii) Solve this matrix equation.
(6)

(e) The function f can also be written f (x) = x (x −1)(rx − s) where r and s are integers. Find r
and s.
(3)
(Total 14 marks)
91. The function f is defined as f (x) = (2x +1) e−x, 0  x  3. The point P(0, 1) lies on the graph of f
(x), and there is a maximum point at Q.
(a) Sketch the graph of y = f (x), labelling the points P and Q.
(3)

(b) (i) Show that f ′ (x) = (1− 2x) e−x.


(ii) Find the exact coordinates of Q.
(7)

(c) The equation f (x) = k, where k  , has two solutions. Write down the range of values
of k.
(2)

(d) Given that f (x) = e−x (−3 + 2x), show that the curve of f has only one point of inflexion.
(2)

(e) Let R be the point on the curve of f with x-coordinate 3. Find the area of the region
enclosed by the curve and the line (PR).
(7)
(Total 21 marks)

92. The functions f and g are defined by f :  3x, g : x  x  2 .


(a) Find an expression for (f  g) (x).
(b) Show that f –l (18) + g–l (18) = 22.
(Total 6 marks)
3
93. The function f is defined by f ( x)  , for –3 < x < 3.
9 – x2
(a) On the grid below, sketch the graph of f.

(b) Write down the equation of each vertical asymptote.


(c) Write down the range of the function f.
(Total 6 marks)

94. The quadratic function f is defined by f (x) = 3x2 – 12x + 11.


(a) Write f in the form f (x) = 3(x – h)2 – k.
(b) The graph of f is translated 3 units in the positive x-direction and 5 units in the positive
y-direction. Find the function g for the translated graph, giving your answer in the form
g (x) = 3(x – p)2 + q.
(Total 6 marks)
95. The diagram below shows the graphs of f (x) = 1 + e2x, g (x) = 10x + 2, 0  x  1.5.
y
f

16 g

12

p
8

x
0.5 1 1.5

(a) (i) Write down an expression for the vertical distance p between the graphs of f and g.
(ii) Given that p has a maximum value for 0  x  1.5, find the value of x at which this
occurs.
(6)

The graph of y = f (x) only is shown in the diagram below. When x = a, y = 5.


y

16

12

8
5
4

x
0.5 a 1 1.5

(b) (i) Find f –1(x).


(ii) Hence show that a = ln 2.
(5)

(c) The region shaded in the diagram is rotated through 360° about the x-axis. Write down an
expression for the volume obtained.
(3)
(Total 14 marks)

96. Consider the line L with equation y + 2x = 3. The line L1 is parallel to L and passes through the
point (6, –4).
(a) Find the gradient of L1.
(b) Find the equation of L1 in the form y = mx + b.
(c) Find the x-coordinate of the point where line L1 crosses the x-axis.
Working:
Answers:
(a) ..................................................................
(b) ..................................................................
(c) ..................................................................
(Total 6 marks)
97. The function f is given by f (x) = e(x–11) –8.
(a) Find f –1(x).
(b) Write down the domain of f –l(x).
Working:
Answers:
(a) ..................................................................
(b) ..................................................................
(Total 6 marks)

98. The graph of y = f (x) is shown in the diagram.


y
2

–2 –1 0 1 2 3 4 5 6 7 8 x
–1

–2
(a) On each of the following diagrams draw the required graph,
(i) y = 2 f (x);
y
2

–2 –1 0 1 2 3 4 5 6 7 8 x
–1

–2

(ii) y = f (x – 3).
y
2

–2 –1 0 1 2 3 4 5 6 7 8 x
–1

–2
(b) The point A (3, –1) is on the graph of f. The point A is the corresponding point on the
graph of y = –f (x) + 1. Find the coordinates of A.
Working:
Answer:
(b) ..................................................................
(Total 6 marks)

99. The equation of a curve may be written in the form y = a(x – p)(x – q). The curve intersects the
x-axis at A(–2, 0) and B(4, 0). The curve of y = f (x) is shown in the diagram below.
y
4

A B
–4 –2 0 2 4 6x

–2

–4

–6

(a) (i) Write down the value of p and of q.


(ii) Given that the point (6, 8) is on the curve, find the value of a.
(iii) Write the equation of the curve in the form y = ax2 + bx + c.
(5)

dy
(b) (i) Find .
dx
(ii) A tangent is drawn to the curve at a point P. The gradient of this tangent is 7.
Find the coordinates of P.
(4)

(c) The line L passes through B(4, 0), and is perpendicular to the tangent to the curve at
point B.
(i) Find the equation of L.
(ii) Find the x-coordinate of the point where L intersects the curve again.
(6)
(Total 15 marks)
100. Part of the graph of f (x) = (x – p) (x – q) is shown below.

The vertex is at C. The graph crosses the y-axis at B.


(a) Write down the value of p and of q.
(b) Find the coordinates of C.
(c) Write down the y-coordinate of B.
Working:

Answers:

(a) .....................................................
(b) .....................................................
(c) .....................................................
(Total 6 marks)
1
101. Consider the functions f (x) = 2x and g (x) = , x  3.
x 3
(a) Calculate (f ◦ g) (4).
(b) Find g−1(x).
(c) Write down the domain of g−1.
Working:

Answers:

(a) .....................................................
(b) .....................................................
(c) .....................................................
(Total 6 marks)
102. A machine was purchased for $10000. Its value V after t years is given by V =100000e−0.3t. The
machine must be replaced at the end of the year in which its value drops below $1500.
Determine in how many years the machine will need to be replaced.
Working:

Answers:

........................................................
(Total 6 marks)
103. Let f (x) = 6 sin x , and g (x) = 6e–x – 3 , for 0  x  2. The graph of f is shown on the diagram
below. There is a maximum value at B (0.5, b).
y
B

x
0 1 2

(a) Write down the value of b.


(b) On the same diagram, sketch the graph of g.
(c) Solve f (x) = g (x) , 0.5  x  1.5.
Working:

Answers:

(a) .................................................
(b) .................................................
(Total 6 marks)
104. Two weeks after its birth, an animal weighed 13 kg. At 10 weeks this animal weighed 53 kg.
The increase in weight each week is constant.
(a) Show that the relation between y, the weight in kg, and x, the time in weeks, can be
written as y = 5x + 3
(2)
(b) Write down the weight of the animal at birth.
(1)
(c) Write down the weekly increase in weight of the animal.
(1)
(d) Calculate how many weeks it will take for the animal to reach 98 kg.
(2)
(Total 6 marks)

16
105. Consider the function f (x) = + 8, x  10.
x 10
(a) Write down the equation of
(i) the vertical asymptote;
(ii) the horizontal asymptote.
(2)

(b) Find the


(i) y-intercept;
(ii) x-intercept.
(2)

(c) Sketch the graph of f , clearly showing the above information.


(4)
16
(d) Let g (x) = , x  0.
x
The graph of g is transformed into the graph of f using two transformations.
10 
The first is a translation with vector  . Give a full geometric description of the
0
second transformation.
(2)
(Total 10 marks)

106. Consider the graph of the function, f , defined by


f (x) = 3x4 – 4x3 – 30x2 – 36x + 112, – 2  x  4.5.
(a) Given that f (x) = 0 has one solution at x = 4, find the other solution.
(2)

(b) The tangent to the graph of f is horizontal at x = 3 and at one other value of x. Find this
other value.
(3)

(c) Find the x-coordinates of both points of inflexion on the graph of f .


(4)
(d) Write down both coordinates of the point of inflexion on the graph of f where the tangent
is horizontal.
(2)
1
A sketch of the graph of is given below.
f

(e) Write down the equations of the two vertical asymptotes.


(2)

1
(f) The tangent to the graph of is horizontal at P. Write down the x-coordinate of P.
f
(2)
(Total 15 marks)
107. Let f (x) = 2x + 1.
(a) On the grid below draw the graph of f (x) for 0  x  2.
(b) Let g (x) = f (x +3) –2. On the grid below draw the graph of g (x) for –3  x  –1.
y
6

–6 –5 –4 –3 –2 –1 0 1 2 3 4 5 6 x

–1

–2

–3

–4

–5

–6
Working:
(Total 6 marks)

108. The equation x2 – 2kx + 1 = 0 has two distinct real roots. Find the set of all possible values of k.
Working:
Answer:
…………………………………………..
(Total 6 marks)

109. There were 1420 doctors working in a city on 1 January 1994. After n years the number of
doctors, D, working in the city is given by
D = 1420 + 100n.
(a) (i) How many doctors were there working in the city at the start of 2004?
(ii) In what year were there first more than 2000 doctors working in the city?
(3)
At the beginning of 1994 the city had a population of 1.2 million. After n years, the population,
P, of the city is given by
P = 1 200 000 (1.025)n.
(b) (i) Find the population P at the beginning of 2004.
(ii) Calculate the percentage growth in population between 1 January 1994 and 1
January 2004.
(iii) In what year will the population first become greater than 2 million?
(7)

(c) (i) What was the average number of people per doctor at the beginning of 1994?
(ii) After how many complete years will the number of people per doctor first fall
below 600?
(5)
(Total 15 marks)
110. Let f (x) = 2x + 1 and g (x) = 3x2 – 4.
Find
(a) f –1(x);
(b) (g  f ) (–2);
(c) ( f  g) (x).
Working:
Answers:
(a) …………………………………………..
(b) …………………………………………..
(c) …………………………………………..
(Total 6 marks)

111. Let f (x) = 2 + cos (2x) – 2 sin (0.5x) for 0  x  3, where x is in radians.
(a) On the grid below, sketch the curve of y = f (x), indicating clearly the point P on the curve
where the derivative is zero.
y
4
3
2
1
0
0.5 1 1.5 2 2.5 3 x
–1
–2
–3
–4

(b) Write down the solutions of f (x) = 0.


Working:
Answer:
(b) …………………………………………..
(Total 6 marks)

112. The population p of bacteria at time t is given by p = 100e0.05t.


Calculate
(a) the value of p when t = 0;
(b) the rate of increase of the population when t = 10.
Working:
Answers:
(a) …………………………………………..
(b) …………………………………………..
(Total 6 marks)
113. Part of the graph of the periodic function f is shown below. The domain of f is 0  x  15 and
the period is 3.
f(x)
4

0 x
0 1 2 3 4 5 6 7 8 9 10
(a) Find
(i) f (2);
(ii) f (6.5);
(iii) f (14).
(b) How many solutions are there to the equation f (x) = 1 over the given domain?
Working:
Answers:
(a) (i) ………………………………………
(ii) ………………………………………
(iii) ………………………………………
(b) ……………………………………………
(Total 6 marks)
114. The function f (x) is defined as f (x) = –(x – h)2 + k. The diagram below shows part of the graph
of f (x). The maximum point on the curve is P (3, 2).
y
4
P(3, 2)
2

x
–1 1 2 3 4 5 6
–2

–4

–6

–8

–10

–12

(a) Write down the value of


(i) h;
(ii) k.
(2)

(b) Show that f (x) can be written as f (x) = –x2 + 6x – 7.


(1)

(c) Find f (x).


(2)

The point Q lies on the curve and has coordinates (4, 1). A straight line L, through Q, is
perpendicular to the tangent at Q.
(d) (i) Calculate the gradient of L.
(ii) Find the equation of L.
(iii) The line L intersects the curve again at R. Find the x-coordinate of R.
(8)
(Total 13 marks)

115. Let f (x) = 1 + 3 cos (2x) for 0  x  π, and x is in radians.


(a) (i) Find f (x).
(ii) Find the values for x for which f (x) = 0, giving your answers in terms of .
(6)

π
The function g (x) is defined as g (x) = f (2x) – 1, 0  x  .
2
(b) (i) The graph of f may be transformed to the graph of g by a stretch in the x-direction
with scale factor 12 followed by another transformation. Describe fully this other
transformation.
(ii) Find the solution to the equation g (x) = f (x)
(4)
(Total 10 marks)

116. Let h (x) = (x – 2) sin (x – 1) for –5  x  5. The curve of h (x) is shown below. There is a
minimum point at R and a maximum point at S. The curve intersects the x-axis at the points
(a, 0) (1, 0) (2, 0) and (b, 0).
y
4

1
S
(a, 0) (b, 0)
–5 –4 –3 –2 –1 1 2 3 4 5 x
R
–1

–2

–3

–4

–5

–6

–7
(a) Find the exact value of
(i) a;
(ii) b.
(2)

The regions between the curve and the x-axis are shaded for a  x  2 as shown.
(b) (i) Write down an expression which represents the total area of the shaded regions.
(ii) Calculate this total area.
(5)

(c) (i) The y-coordinate of R is –0.240. Find the y-coordinate of S.


(ii) Hence or otherwise, find the range of values of k for which the equation
(x – 2) sin (x – 1) = k has four distinct solutions.
(4)
(Total 11 marks)
x
117. Let f (x) = e–x, and g (x) = , x  –1. Find
1 x
(a) f –1 (x);
(b) (g ° f ) (x).
Working:
Answers:
(a) ..................................................................
(b) ..................................................................
(Total 6 marks)

118. A family of functions is given by


f (x) = x2 + 3x + k, where k  {1, 2, 3, 4, 5, 6, 7}.
One of these functions is chosen at random. Calculate the probability that the curve of this
function crosses the x-axis.
Working:
Answer:
..................................................................
(Total 6 marks)

119. Consider the following relations between two variables x and y.


A. y = sin x
B. y is directly proportional to x
C. y = 1 + tan x
D. speed y as a function of time x, constant acceleration
E. y = 2x
F. distance y as a function of time x, velocity decreasing
Each sketch below could represent exactly two of the above relations on a certain interval.
(i) (ii) (iii)
y y y

x x x
Complete the table below, by writing the letter for the two relations that each sketch could
represent.
sketch relation letters
(i)
(ii)
(iii)
120. The diagram shows part of the graph of the curve y = a (x – h)2 + k, where a, h, k  .
y

20

15

10 P(5, 9)

0 x
1 2 3 4 5 6 .
(a) The vertex is at the point (3, 1). Write down the value of h and of k.
(2)
(b) The point P (5, 9) is on the graph. Show that a = 2.
(3)
(c) Hence show that the equation of the curve can be written as
y = 2x2 – 12x + 19.
(1)
dy
(d) (i) Find .
dx
A tangent is drawn to the curve at P (5, 9).
(ii) Calculate the gradient of this tangent.
(iii) Find the equation of this tangent.
(4)
(Total 10 marks)
121. The equation kx2 + 3x + 1 = 0 has exactly one solution. Find the value of k.
Working:
Answer:
..................................................................
(Total 6 marks)
q
122. (a) The diagram shows part of the graph of the function f (x) = . The curve passes
x– p
through the point A (3, 10). The line (CD) is an asymptote.
y
C
15

10 A

–15 –10 –5 0 5 10 15 x

-5

-10

-15
D
Find the value of
(i) p;
(ii) q.
(b) The graph of f (x) is transformed as shown in the following diagram. The point A is
transformed to A (3, –10).
y
C
15

10

–15 –10 –5 0 5 10 15 x

–5

–10
A

–15
D
Give a full geometric description of the transformation.
Working:
Answers:
(a) (i) ...........................................................
(ii) ...........................................................
(b) ..................................................................
..................................................................
(Total 6 marks)

123. The mass m kg of a radio-active substance at time t hours is given by


m = 4e–0.2t.
(a) Write down the initial mass.
(b) The mass is reduced to 1.5 kg. How long does this take?
Working:
Answers:
(a) ..................................................................
(b) ..................................................................
(Total 6 marks)

124. The function f is given by f (x) = x2 – 6x + 13, for x  3.


(a) Write f (x) in the form (x – a)2 + b.
(b) Find the inverse function f –1.
(c) State the domain of f –1.
Working:
Answers:
(a) ..................................................................
(b) ..................................................................
(c) ..................................................................
(Total 6 marks)
x
125. Let f (x) = 2x, and g (x) = , (x  2).
x–2
Find
(a) (g  f ) (3);
(b) g–1 (5).
Working:
Answers:
(a) ..................................................................
(b) ..................................................................
(Total 6 marks)

126. Solve the equation log27 x = 1 – log27 (x – 0.4).


Working:
Answer:
......................................................................
(Total 6 marks)
127. The diagram shows part of the graph of y = a (x – h)2 + k. The graph has its vertex at P, and
passes through the point A with coordinates (1, 0).
y

P
2

A
–1 0 1 x
(a) Write down the value of
(i) h;
(ii) k.
(b) Calculate the value of a.
Working:
Answers:
(a) (i) ..........................................................
(ii) ..........................................................
(b) .................................................................
(Total 6 marks)

128. Consider functions of the form y = e–kx


1 1
(a) Show that  0
e – kx dx =
k
(1 – e–k).
(3)

(b) Let k = 0.5


(i) Sketch the graph of y = e–0.5x, for –1  x  3, indicating the coordinates of the
y-intercept.
(ii) Shade the region enclosed by this graph, the x-axis, y-axis and the line x = 1.
(iii) Find the area of this region.
(5)

dy
(c) (i) Find in terms of k, where y = e–kx.
dx
The point P(1, 0.8) lies on the graph of the function y = e–kx.
(ii) Find the value of k in this case.
(iii) Find the gradient of the tangent to the curve at P.
(5)
(Total 13 marks)

129. Consider the function f (x) = 2x2 – 8x + 5.


(a) Express f (x) in the form a (x – p)2 + q, where a, p, q  .
(b) Find the minimum value of f (x).
Working:
Answers:
(a) ..................................................................
(b) ..................................................................
(Total 6 marks)

130. Solve the equation ex = 5 – 2x, giving your answer correct to four significant figures.
Working:
Answer:
......................................................................
(Total 6 marks)

6– x
131. Consider the functions f : x  4(x – 1) and g : x  .
2
(a) Find g–1.
(b) Solve the equation ( f ° g–1) (x) = 4.
Working:
Answers:
(a) ..................................................................
(b) ..................................................................
(Total 6 marks)
132. $1000 is invested at 15% per annum interest, compounded monthly. Calculate the minimum
number of months required for the value of the investment to exceed $3000.
Working:
Answer:
......................................................................
(Total 6 marks)
133. The sketch shows part of the graph of y = f (x) which passes through the points A(–1, 3), B(0,
2), C(l, 0), D(2, 1) and E(3, 5).

6
E
5

4
A
3
B
2
D
1
C
–4 –3 –2 –1 0 1 2 3 4 5

–1

–2
A second function is defined by g (x) = 2f (x – 1).
(a) Calculate g (0), g (1), g (2) and g (3).
(b) On the same axes, sketch the graph of the function g (x).
Working:
Answers:
(a) ..................................................................
..................................................................
(Total 6 marks)

134. The diagram below shows a sketch of the graph of the function y = sin (ex) where –1  x  2,
and x is in radians. The graph cuts the y-axis at A, and the x-axis at C and D. It has a maximum
point at B.
y
B
A

–1 0 1 C D 2 x

(a) Find the coordinates of A.


(2)

(b) The coordinates of C may be written as (ln k, 0). Find the exact value of k.
(2)

(c) (i) Write down the y-coordinate of B.


dy
(ii) Find .
dx
π
(iii) Hence, show that at B, x = ln .
2
(6)

(d) (i) Write down the integral which represents the shaded area.
(ii) Evaluate this integral.
(5)
(e) (i) Copy the above diagram into your answer booklet. (There is no need to copy the
shading.) On your diagram, sketch the graph of y = x3.
(ii) The two graphs intersect at the point P. Find the x-coordinate of P.
(3)
(Total 18 marks)

135. Let g (x) = x4 – 2x3 + x2 – 2.


(a) Solve g (x) = 0.
(2)
2x3
Let f (x) =  1 . A part of the graph of f (x) is shown below.
g ( x)
y

A 0 B x

(b) The graph has vertical asymptotes with equations x = a and x = b where a < b. Write
down the values of
(i) a;
(ii) b.
(2)

(c) The graph has a horizontal asymptote with equation y = l. Explain why the value of f (x)
approaches 1 as x becomes very large.
(2)
(d) The graph intersects the x-axis at the points A and B. Write down the exact value of the
x-coordinate at
(i) A;
(ii) B.
(2)

(e) The curve intersects the y-axis at C. Use the graph to explain why the values of f (x) and
f (x) are zero at C.
(2)
(Total 10 marks)

136. The diagram shows part of the graph with equation y = x2 + px + q. The graph cuts the x-axis at
–2 and 3.
y

x
–3 –2 –1 0 1 2 3 4
–2

–4

–6
Find the value of
(a) p;
(b) q.
Working:
Answers:
(a) ..................................................................
(b) ..................................................................
(Total 4 marks)

137. Each year for the past five years the population of a certain country has increased at a steady
rate of 2.7% per annum. The present population is 15.2 million.
(a) What was the population one year ago?
(b) What was the population five years ago?
Working:
Answers:
(a) ..................................................................
(b) ..................................................................
(Total 4 marks)

138. (a) On the following diagram, sketch the graphs of y = ex and y = cos x
for –2  x  1.
y
2

x
–2 –1 0 1
–1

–2

(b) The equation ex = cos x has a solution between –2 and –1.


Find this solution.
Working:
Answers:
(a) ..................................................................
(b) ..................................................................
(Total 4 marks)
139. The function f is defined by
3
f : x a 3 – 2x , x .
2
Evaluate f –1(5).
Working:
Answer:
.......................................................................
(Total 4 marks)
140. The following diagram shows the graph of y = f (x). It has minimum and maximum points at
1
(0, 0) and ( 1, ).
2
y

3.5

2.5

1.5

0.5

–2 –1 0 1 2 3 x
–0.5

–1

–1.5

–2

–2.5
3
(a) On the same diagram, draw the graph of y  f ( x – 1)  .
2
(b) What are the coordinates of the minimum and maximum points of
3
y  f ( x – 1)  ?
2
Working:
Answer:
(b) ................................................................
(Total 4 marks)

141. Michele invested 1500 francs at an annual rate of interest of 5.25 percent,
compounded annually.
(a) Find the value of Michele’s investment after 3 years. Give your answer to the nearest
franc.
(3)
(b) How many complete years will it take for Michele’s initial investment to double in value?
(3)
(c) What should the interest rate be if Michele’s initial investment were to double in value in
10 years?
(4)
(Total 10 marks)

142. Note: Radians are used throughout this question.


Let f (x) = sin (1 + sin x).
(a) (i) Sketch the graph of y = f (x), for 0  x  6.
(ii) Write down the x-coordinates of all minimum and maximum points of f, for
0  x  6. Give your answers correct to four significant figures.
(9)

(b) Let S be the region in the first quadrant completely enclosed by the graph of f and both
coordinate axes.
(i) Shade S on your diagram.
(ii) Write down the integral which represents the area of S.
(iii) Evaluate the area of S to four significant figures.
(5)
(c) Give reasons why f (x) ≥ 0 for all values of x.
(2)
(Total 16 marks)

143. In the diagram below, the points O(0, 0) and A(8, 6) are fixed. The angle OP̂A
varies as the point P(x, 10) moves along the horizontal line y = 10.
y
P(x, 10)
y=10

A(8, 6)

O(0, 0) x
Diagram to scale

(a) (i) Show that AP  x 2 – 16 x  80.


(ii) Write down a similar expression for OP in terms of x.
(2)

(b) Hence, show that


x 2 – 8 x  40
cos OP̂A  ,
 {( x 2 – 16 x  80)( x 2  100 )}
(3)

(c) Find, in degrees, the angle OP̂A when x = 8.


(2)
(d) Find the positive value of x such that OP̂A  60 .
(4)
Let the function f be defined by
x 2 – 8 x  40
f ( x)  cos OP̂A  , 0  x  15.
 {( x 2 – 16 x  80)( x 2  100 )}

(e) Consider the equation f (x) = 1.


(i) Explain, in terms of the position of the points O, A, and P, why this
equation has a solution.
(ii) Find the exact solution to the equation.
(5)
(Total 16 marks)

144. The diagram shows parts of the graphs of y = x2 and y = 5 – 3(x – 4)2.
y

y = x2
8

6 2
y = 5 – 3(x–4)

x
–2 0 2 4 6
The graph of y = x2 may be transformed into the graph of y = 5 – 3(x – 4)2 by these
transformations.
A reflection in the line y = 0 followed by
a vertical stretch with scale factor k followed by
a horizontal translation of p units followed by
a vertical translation of q units.
Write down the value of
(a) k;
(b) p;
(c) q.
Working:
Answers:
(a) ..................................................................
(b) ..................................................................
(c) ..................................................................
(Total 4 marks)
1
145. Solve the equation log9 81 + log9 + log9 3 = log9 x.
9
Working:
Answer:
.......................................................................
(Total 4 marks)
 x
146. The diagram below shows the graph of y = x sin   , for 0  x < m, and 0  y < n, where x is
3
in radians and m and n are integers.
y
n

n–1

0 m–1 m x

Find the value of


(a) m;
(b) n.
Working:
Answers:
(a) ..................................................................
(b) ..................................................................
(Total 4 marks)
147. Given that f (x) = 2e3x, find the inverse function f –1(x).
Working:
Answer:
.......................................................................
(Total 4 marks)

148. The diagram below shows part of the graph of the function
f : x  – x3  2 x 2  15 x .
y

40
Q
35
30
25
20
15
10
5 B
A
–3 –2 –1 –5 1 2 3 4 5 x
–10
–15
P
–20

The graph intercepts the x-axis at A(–3, 0), B(5, 0) and the origin, O. There is a minimum point
at P and a maximum point at Q.
(a) The function may also be written in the form f : x  – x( x – a) ( x – b),
where a < b. Write down the value of
(i) a;
(ii) b.
(2)

(b) Find
(i) f (x);
(ii) the exact values of x at which f '(x) = 0;
(iii) the value of the function at Q.
(7)

(c) (i) Find the equation of the tangent to the graph of f at O.


(ii) This tangent cuts the graph of f at another point. Give the x-coordinate of this point.
(4)
(d) Determine the area of the shaded region.
(2)
(Total 15 marks)
149. (a) Express f (x) = x2 – 6x + 14 in the form f (x) = (x – h)2 + k, where h and k are to be
determined.
(b) Hence, or otherwise, write down the coordinates of the vertex of the parabola with
equation y – x2 – 6x + 14.
Working:
Answers:
(a) ..................................................................
(b) ..................................................................
(Total 4 marks)
150. A group of ten leopards is introduced into a game park. After t years the number of leopards, N,
is modelled by N = 10 e0.4t.
(a) How many leopards are there after 2 years?
(b) How long will it take for the number of leopards to reach 100? Give your answers to an
appropriate degree of accuracy.
Give your answers to an appropriate degree of accuracy.
Working:
Answers:
(a) ..................................................................
(b) ..................................................................
(Total 4 marks)
151. Consider the function f : x  x  1, x  – 1
(a) Determine the inverse function f –1.
(b) What is the domain of f –1?
Working:
Answers:
(a) ..................................................................
(b) ..................................................................
(Total 4 marks)

152. The diagram shows the graph of y = f (x), with the x-axis as an asymptote.
y

B(5, 4)

A(–5, –4)

(a) On the same axes, draw the graph of y =f (x + 2) – 3, indicating the coordinates of the
images of the points A and B.

(b) Write down the equation of the asymptote to the graph of y = f (x + 2) – 3.


Working:
Answer:
(b)
...................................................................
.
(Total 4 marks)

153. (a) Sketch, on the given axes, the graphs of y = x2 and y – sin x for –1  x  2.
y

–1 –0.5 0 0.5 1 1.5 2 x


(b) Find the positive solution of the equation
x2 = sin x,
giving your answer correct to 6 significant figures.
Working:
Answer:
(b)
...................................................................
.
(Total 4 marks)

154. Note: Radians are used throughout this question.


(a) Draw the graph of y =  + x cos x, 0  x  5, on millimetre square graph paper, using a
scale of 2 cm per unit. Make clear
(i) the integer values of x and y on each axis;
(ii) the approximate positions of the x-intercepts and the turning points.
(5)

(b) Without the use of a calculator, show that  is a solution of the equation
 + x cos x = 0.
(3)

(c) Find another solution of the equation  + x cos x = 0 for 0  x  5, giving your answer to
six significant figures.
(2)

(d) Let R be the region enclosed by the graph and the axes for 0  x  . Shade R on your
diagram, and write down an integral which represents the area of R .
(2)

(e) Evaluate the integral in part (d) to an accuracy of six significant figures. (If you consider
d
it necessary, you can make use of the result ( x sin x  cos x )  x cos x .)
dx
(3)
(Total 15 marks)

155. A ball is thrown vertically upwards into the air. The height, h metres, of the ball above the
ground after t seconds is given by
h = 2 + 20t – 5t2, t  0
(a) Find the initial height above the ground of the ball (that is, its height at the instant when
it is released).
(2)

(b) Show that the height of the ball after one second is 17 metres.
(2)

(c) At a later time the ball is again at a height of 17 metres.


(i) Write down an equation that t must satisfy when the ball is at a height of 17 metres.
(ii) Solve the equation algebraically.
(4)

dh
(d) (i) Find .
dt
(ii) Find the initial velocity of the ball (that is, its velocity at the instant when it is
released).
(iii) Find when the ball reaches its maximum height.
(iv) Find the maximum height of the ball.
(7)
(Total 15 marks)
156. Two functions f, g are defined as follows:
f : x  3x + 5
g : x  2(1 – x)
Find
(a) f –1(2);
(b) (g  f )(–4).
Working:
Answers:
(a) ..................................................................
(b) ..................................................................
(Total 4 marks)
157. The quadratic equation 4x2 + 4kx + 9 = 0, k > 0 has exactly one solution for x.
Find the value of k.
Working:
Answer:
......................................................................
(Total 4 marks)

158. The diagram shows three graphs.


B
y
A

A is part of the graph of y = x.


B is part of the graph of y = 2x.
C is the reflection of graph B in line A.
Write down
(a) the equation of C in the form y =f (x);
(b) the coordinates of the point where C cuts the x-axis.
Working:
Answers:
(a) ..................................................................
(b) ..................................................................
(Total 4 marks)
159. The diagrams show how the graph of y = x2 is transformed to the graph of y = f (x) in three
steps.
For each diagram give the equation of the curve.
y y

(a)
1

0 x 0 x
y=x2 1

(b) (c) 7
y

0 1 x 0 1 x
Working:
Answers:
(a) ..................................................................
(b) ..................................................................
(c) ..................................................................
(Total 4 marks)

160. f (x) = 4 sin  3 x    .


 2
For what values of k will the equation f (x) = k have no solutions?
Working:
Answer:
......................................................................
(Total 4 marks)

161. The diagram shows the graph of the function y = ax2 + bx + c.


y

x
Complete the table below to show whether each expression is positive, negative or zero.
Expression positive negative zero
a
c
b2 – 4ac
b

Working:
(Total 4 marks)

162. Initially a tank contains 10 000 litres of liquid. At the time t = 0 minutes a tap is opened, and
liquid then flows out of the tank. The volume of liquid, V litres, which remains in the tank after t
minutes is given by
V = 10 000 (0.933t).

(a) Find the value of V after 5 minutes.


(1)

(b) Find how long, to the nearest second, it takes for half of the initial amount of liquid to
flow out of the tank.
(3)
(c) The tank is regarded as effectively empty when 95% of the liquid has flowed out.
Show that it takes almost three-quarters of an hour for this to happen.
(3)

(d) (i) Find the value of 10 000 – V when t = 0.001 minutes.


(ii) Hence or otherwise, estimate the initial flow rate of the liquid.
Give your answer in litres per minute, correct to two significant figures.
(3)
(Total 10 marks)

163. (a) Factorize x2 – 3x – 10.


(b) Solve the equation x2 – 3x – 10 = 0.
Working:
Answers:
(a) ..................................................................
(b) ..................................................................
(Total 4 marks)
164. The diagram represents the graph of the function
f : x  (x – p)(x – q).
y

– 12 2 x

(a) Write down the values of p and q.

(b) The function has a minimum value at the point C. Find the x-coordinate of C.
Working:
Answers:
(a) ..................................................................
(b) ..................................................................
(Total 4 marks)
165. The graph represents the function
f : x  p cos x, p  .
y
3

 x

–3

Find
(a) the value of p;
(b) the area of the shaded region.
Working:
Answers:
(a) ..................................................................
(b) ..................................................................
(Total 4 marks)
166. Two functions f and g are defined as follows:
f (x) = cos x, 0  x  2;
g (x) = 2x + 1, x .
Solve the equation (g  f)(x) = 0.
Working:
Answer:
......................................................................
(Total 4 marks)

167. The function f is given by


f (x) = 2 x  1 , x  , x  3.
x3
(a) (i) Show that y = 2 is an asymptote of the graph of y = f (x).
(2)
(ii) Find the vertical asymptote of the graph.
(1)
(iii) Write down the coordinates of the point P at which the asymptotes intersect.
(1)

(b) Find the points of intersection of the graph and the axes.
(4)
(c) Hence sketch the graph of y = f (x), showing the asymptotes by dotted lines.
(4)
(d) Show that f (x) = 7 and hence find the equation of the tangent at
( x  3) 2
the point S where x = 4.
(6)

(e) The tangent at the point T on the graph is parallel to the tangent at S.
Find the coordinates of T.
(5)
(f) Show that P is the midpoint of [ST].
(l)
(Total 24 marks)

168. The diagram shows the parabola y = (7 – x)(l + x). The points A and C are the x-intercepts and
the point B is the maximum point.
y

A 0 C x
Find the coordinates of A, B and C.
Working:
Answer:
......................................................................
(Total 4 marks)

169. Three of the following diagrams I, II, III, IV represent the graphs of
(a) y = 3 + cos 2x
(b) y = 3 cos (x + 2)
(c) y = 2 cos x + 3.
Identify which diagram represents which graph.
I y II y

4 2

1
2
x
– 
– – 
–  
–
 –1  
x
– 
– – 
–  
– –2
  

y y
III IV
3
5
2
4
1
3
x
– 
– – 
–  
–
2
  
1

x
– 
– – 
–  
–
–3   
Working:
Answers:
(a) ..................................................................
(b) ..................................................................
(c) ..................................................................
(Total 4 marks)
170. The function f is given by f (x) = 1n ( x  2) . Find the domain of the function.
Working:
Answer:
......................................................................
(Total 4 marks)

171. A population of bacteria is growing at the rate of 2.3% per minute. How long will it take for the
size of the population to double? Give your answer to the nearest minute.
Working:
Answer:
......................................................................
(Total 4 marks)
172. Let f (x) = x , and g (x) = 2x. Solve the equation
(f –1  g)(x) = 0.25.
Working:
Answer:
......................................................................
(Total 4 marks)

173. Solve the equation 2cos x = sin 2x, for 0 ≤ x ≤ 3π.


(Total 7 marks)
174. The following diagram represents a large Ferris wheel, with a diameter of 100 metres.

Let P be a point on the wheel. The wheel starts with P at the lowest point, at ground level. The
wheel rotates at a constant rate, in an anticlockwise (counterclockwise) direction. One
revolution takes 20 minutes.

(a) Write down the height of P above ground level after


(i) 10 minutes;
(ii) 15 minutes.
(2)

Let h(t) metres be the height of P above ground level after t minutes. Some values of h(t) are
given in the table below.

t h(t)
0 0.0
1 2.4
2 9.5
3 20.6
4 34.5
5 50.0

(b) (i) Show that h(8) = 90.5.


(ii) Find h(21).
(4)
(c) Sketch the graph of h, for 0 ≤ t ≤ 40.
(3)

(d) Given that h can be expressed in the form h(t) = a cos bt + c, find a, b and c.
(5)
(Total 14 marks)

175. The following diagram shows triangle ABC.

diagram not to scale


AB = 7 cm, BC = 9 cm and AB̂C = 120°.

(a) Find AC.


(3)
(b) Find BÂC .
(3)
(Total 6 marks)
176. The following diagram shows a waterwheel with a bucket. The wheel rotates at a constant rate
in an anticlockwise (counterclockwise) direction.

diagram not to scale


The diameter of the wheel is 8 metres. The centre of the wheel, A, is 2 metres above the water
level. After t seconds, the height of the bucket above the water level is given by h = a sin bt + 2.

(a) Show that a = 4.


(2)

The wheel turns at a rate of one rotation every 30 seconds.


π
(b) Show that b = .
15
(2)

In the first rotation, there are two values of t when the bucket is descending at a rate of
0.5 m s–1.
(c) Find these values of t.
(6)

(d) Determine whether the bucket is underwater at the second value of t.


(4)
(Total 14 marks)
177. There is a vertical tower TA of height 36 m at the base A of a hill. A straight path goes up the
hill from A to a point U. This information is represented by the following diagram.

The path makes a 4° angle with the horizontal.


The point U on the path is 25 m away from the base of the tower.
The top of the tower is fixed to U by a wire of length x m.

(a) Complete the diagram, showing clearly all the information above.
(3)

(b) Find x.
(4)
(Total 7 marks)

178. The diagram below shows a plan for a window in the shape of a trapezium.

Three sides of the window are 2 m long. The angle between the sloping sides of the window and
π
the base is θ, where 0 < θ < .
2
(a) Show that the area of the window is given by y = 4 sin θ + 2 sin 2θ.
(5)
(b) Zoe wants a window to have an area of 5 m2. Find the two possible values of θ.
(4)

(c) John wants two windows which have the same area A but different values of θ.
Find all possible values for A.
(7)
(Total 16 marks)

3
179. The straight line with equation y = x makes an acute angle θ with the x-axis.
4
(a) Write down the value of tan θ.
(1)

(b) Find the value of


(i) sin 2θ;
(ii) cos 2θ.
(6)
(Total 7 marks)
180. The graph of y = p cos qx + r, for –5 ≤ x ≤ 14, is shown below.

There is a minimum point at (0, –3) and a maximum point at (4, 7).
(a) Find the value of
(i) p;
(ii) q;
(iii) r.
(6)

(b) The equation y = k has exactly two solutions. Write down the value of k.
(1)
(Total 7 marks)
181. The diagram below shows a circle with centre O and radius 8 cm.

diagram not to scale


The points A, B, C, D, E and F are on the circle, and [AF] is a diameter. The length of arc ABC
is 6 cm.

(a) Find the size of angle AOC.


(2)

(b) Hence find the area of the shaded region.


(6)

The area of sector OCDE is 45 cm2.


(c) Find the size of angle COE.
(2)

(d) Find EF.


(5)
(Total 15 marks)
182. The diagram shows two concentric circles with centre O.

diagram not to scale


The radius of the smaller circle is 8 cm and the radius of the larger circle is 10 cm.
π
Points A, B and C are on the circumference of the larger circle such that AÔB is radians.
3

(a) Find the length of the arc ACB.


(2)

(b) Find the area of the shaded region.


(4)
(Total 6 marks)

183. (a) Show that 4 – cos 2θ + 5 sin θ = 2 sin2 θ + 5 sin θ + 3.


(2)
(b) Hence, solve the equation 4 – cos 2θ + 5 sin θ = 0 for 0 ≤ θ ≤ 2π.
(5)
(Total 7 marks)
184. The diagram shows quadrilateral ABCD with vertices A(1, 0), B(1, 5), C(5, 2) and D(4, –1).

diagram not to scale

(a) (i) Show that AC   42  .


 
(ii) Find BD .
(iii) Show that AC is perpendicular to BD .
(5)

The line (AC) has equation r = u + sv.


(b) (i) Write down vector u and vector v.
(ii) Find a vector equation for the line (BD).
(4)

The lines (AC) and (BD) intersect at the point P(3, k).
(c) Show that k = 1.
(3)
(d) Hence find the area of triangle ACD.
(5)
(Total 17 marks)

185. The following diagram shows the triangle ABC.

diagram not to scale


The angle at C is obtuse, AC = 5 cm, BC = 13.6 cm and the area is 20 cm2.

(a) Find AĈB .


(4)

(b) Find AB.


(3)
(Total 7 marks)
186. The following diagram represents a large Ferris wheel at an amusement park.
The points P, Q and R represent different positions of a seat on the wheel.

The wheel has a radius of 50 metres and rotates clockwise at a rate of one revolution every 30
minutes.
A seat starts at the lowest point P, when its height is one metre above the ground.

(a) Find the height of a seat above the ground after 15 minutes.
(2)

(b) After six minutes, the seat is at point Q. Find its height above the ground at Q.
(5)

The height of the seat above ground after t minutes can be modelled by the function
h(t) = 50 sin (b(t – c)) + 51.
(c) Find the value of b and of c.
(6)

(d) Hence find the value of t the first time the seat is 96 m above the ground.
(3)
(Total 16 marks)
187. A rectangle is inscribed in a circle of radius 3 cm and centre O, as shown below.

The point P(x, y) is a vertex of the rectangle and also lies on the circle. The angle between (OP)
π
and the x-axis is θ radians, where 0 ≤ θ ≤ .
2

(a) Write down an expression in terms of θ for


(i) x;
(ii) y.
(2)

Let the area of the rectangle be A.


(b) Show that A = 18 sin 2θ.
(3)
dA
(c) (i) Find .
d
(ii) Hence, find the exact value of θ which maximizes the area of the rectangle.
(iii) Use the second derivative to justify that this value of θ does give a maximum.
(8)
(Total 13 marks)

188. The vertices of the triangle PQR are defined by the position vectors
 4  3 6
     
OP    3 , OQ    1 and OR    1 .
 1  2 5
     
(a) Find
(i) PQ ;
(ii) PR .
(3)

1
(b) Show that cos RP̂Q  .
2
(7)

(c) (i) Find sin RP̂Q .


(ii) Hence, find the area of triangle PQR, giving your answer in the form a 3 .
(6)
(Total 16 marks)
189. The circle shown has centre O and radius 3.9 cm.

diagram not to scale


Points A and B lie on the circle and angle AOB is 1.8 radians.

(a) Find AB.


(3)

(b) Find the area of the shaded region.


(4)
(Total 7 marks)

190. The diagram below shows a triangle ABD with AB = 13 cm and AD = 6.5 cm.
Let C be a point on the line BD such that BC = AC = 7 cm.

diagram not to scale

(a) Find the size of angle ACB.


(3)

(b) Find the size of angle CAD.


(5)
(Total 8 marks)

191. Let f(x) = 3sinx + 4 cos x, for –2π ≤ x ≤ 2π.


(a) Sketch the graph of f.
(3)

(b) Write down


(i) the amplitude;
(ii) the period;
π
(iii) the x-intercept that lies between  and 0.
2
(3)

(c) Hence write f(x) in the form p sin (qx + r).


(3)
(d) Write down one value of x such that f′(x) = 0.
(2)

(e) Write down the two values of k for which the equation f(x) = k has exactly two solutions.
(2)

(f) Let g(x) = ln(x + 1), for 0 ≤ x ≤ π. There is a value of x, between 0 and 1, for which the
gradient of f is equal to the gradient of g. Find this value of x.
(5)
(Total 18 marks)

192. Solve cos 2x – 3 cos x – 3 – cos2 x = sin2 x, for 0 ≤ x ≤ 2π.


(Total 7 marks)

193. The following diagram shows a circle with centre O and radius 4 cm.

diagram not to scale


The points A, B and C lie on the circle. The point D is outside the circle, on (OC).
Angle ADC = 0.3 radians and angle AOC = 0.8 radians.

(a) Find AD.


(3)

(b) Find OD.


(4)

(c) Find the area of sector OABC.


(2)

(d) Find the area of region ABCD.


(4)
(Total 13 marks)

194. Let p = sin40 and q = cos110. Give your answers to the following in terms of p and/or q.
(a) Write down an expression for
(i) sin140;
(ii) cos70.
(2)
(b) Find an expression for cos140.
(3)
(c) Find an expression for tan140.
(1)
(Total 6 marks)

195. Consider g (x) = 3 sin 2x.


(a) Write down the period of g.
(1)
(b) On the diagram below, sketch the curve of g, for 0  x  2.
y
4
3
2
1
0
π π 3π 2π
x
–1 2 2
–2
–3
–4
(3)

(c) Write down the number of solutions to the equation g (x) = 2, for 0  x  2.
(2)
(Total 6 marks)

1 π
196. (a) Given that cos A = and 0  A  , find cos 2A.
3 2
(3)
2 π
(b) Given that sin B = and  B  , find cos B.
3 2
(3)
(Total 6 marks)
197. Let f : x  sin3 x.
(a) (i) Write down the range of the function f.
(ii) Consider f (x) =1, 0  x  2. Write down the number of solutions to this equation.
Justify your answer.
(5)
(b) Find f ′ (x), giving your answer in the form a sin x cos x where a, p, q 
p q
.
(2)
1
π
(c) Let g (x) = 3 sin x (cos x) 2 for 0  x 
. Find the volume generated when the curve
2
of g is revolved through 2 about the x-axis.
(7)
(Total 14 marks)
198. The following diagram shows a semicircle centre O, diameter [AB], with radius 2.
Let P be a point on the circumference, with PÔB =  radians.

(a) Find the area of the triangle OPB, in terms of .


(2)
(b) Explain why the area of triangle OPA is the same as the area triangle OPB.
(3)
Let S be the total area of the two segments shaded in the diagram below.

(c) Show that S = 2( − 2 sin  ).


(3)
(d) Find the value of  when S is a local minimum, justifying that it is a minimum.
(8)
(e) Find a value of  for which S has its greatest value.
(2)
(Total 18 marks)
199. The diagram below shows triangle PQR. The length of [PQ] is 7 cm, the length of [PR] is
10 cm, and PQ̂R is 75.

(a) Find PQ̂R.


(3)
(b) Find the area of triangle PQR.
(3)
(Total 6 marks)

200. The diagram below shows a circle centre O, with radius r. The length of arc ABC is 3 cm and

AÔC = .
9

(a) Find the value of r.


(2)
(b) Find the perimeter of sector OABC.
(2)
(c) Find the area of sector OABC.
(2)
(Total 6 marks)
201. The following graph shows the depth of water, y metres, at a point P, during one day.
The time t is given in hours, from midnight to noon.

(a) Use the graph to write down an estimate of the value of t when
(i) the depth of water is minimum;
(ii) the depth of water is maximum;
(iii) the depth of the water is increasing most rapidly.
(3)
(b) The depth of water can be modelled by the function y = A cos (B (t – 1)) + C.
(i) Show that A = 8.
(ii) Write down the value of C.
(iii) Find the value of B.
(6)
(c) A sailor knows that he cannot sail past P when the depth of the water is less than 12 m.
Calculate the values of t between which he cannot sail past P.
(2)
(Total 11 marks)

π
202. Let f(x) = sin3 x + cos3 x tan x, < x < π.
2
(a) Show that f(x) = sin x.
(2)

2 4 5
(b) Let sin x = . Show that f(2x) =  .
3 9
(5)
(Total 7 marks)

203. A ship leaves port A on a bearing of 030°. It sails a distance of 25 km to point B.


At B, the ship changes direction to a bearing of 100°. It sails a distance of 40 km to reach point
C. This information is shown in the diagram below.
diagram not to scale
A second ship leaves port A and sails directly to C.

(a) Find the distance the second ship will travel.


(4)

(b) Find the bearing of the course taken by the second ship.
(3)
(Total 7 marks)
204. The graph of a function of the form y = p cos qx is given in the diagram below.

(a) Write down the value of p.


(2)

(b) Calculate the value of q.


(4)
(Total 6 marks)

π 12
205. Given that    π and that cosθ =  , find
2 13
(a) sin θ;
(3)
(b) cos 2θ;
(3)
(c) sin (θ + π).
(1)
(Total 7 marks)
206. (a) Given that 2 sin2 θ + sinθ – 1 = 0, find the two values for sin θ.
(4)

(b) Given that 0° ≤ θ ≤ 360° and that one solution for θ is 30°, find the other two possible
values for θ.
(2)
(Total 6 marks)

207. A spring is suspended from the ceiling. It is pulled down and released, and then oscillates up
and down. Its length, l centimetres, is modelled by the function l = 33 + 5cos((720t)°), where t is
time in seconds after release.
(a) Find the length of the spring after 1 second.
(2)

(b) Find the minimum length of the spring.


(3)

(c) Find the first time at which the length is 33 cm.


(3)

(d) What is the period of the motion?


(2)
(Total 10 marks)
208. The diagram below shows part of the graph of y = sin 2x. The shaded region is between x = 0
and x = m.

(a) Write down the period of this function.


(2)

(b) Hence or otherwise write down the value of m.


(2)

(c) Find the area of the shaded region.


(6)
(Total 10 marks)

209. In the triangle PQR, PR = 5 cm, QR = 4 cm and PQ = 6 cm.


Calculate
(a) the size of PQ̂R ;
(b) the area of triangle PQR.
(Total 6 marks)
210. The following diagram shows a triangle ABC, where AĈB is 90, AB = 3, AC = 2 and BÂC
is .

5
(a) Show that sin  = .
3
4 5
(b) Show that sin 2 = .
9
(c) Find the exact value of cos 2.
(Total 6 marks)

211. The following diagram shows a sector of a circle of radius r cm, and angle  at the centre. The
perimeter of the sector is 20 cm.

20  2r
(a) Show that  = .
r

(b) The area of the sector is 25 cm2. Find the value of r.


(Total 6 marks)
212. The following diagram shows the triangle AOP, where OP = 2 cm, AP = 4 cm and AO = 3 cm.
A

diagram not to
scale

O P
(a) Calculate AÔP , giving your answer in radians.
(3)

The following diagram shows two circles which intersect at the points A and B. The smaller
circle C1 has centre O and radius 3 cm, the larger circle C2 has centre P and radius 4 cm, and OP
= 2 cm. The point D lies on the circumference of C1 and E on the circumference of C2.Triangle
AOP is the same as triangle AOP in the diagram above.

A
C2

C1

O P diagram not to
D E
scale

(b) Find AÔB , giving your answer in radians.


(2)
(c) Given that AP̂B is 1.63 radians, calculate the area of
(i) sector PAEB;
(ii) sector OADB.
(5)

(d) The area of the quadrilateral AOBP is 5.81 cm2.


(i) Find the area of AOBE.
(ii) Hence find the area of the shaded region AEBD.
(4)
(Total 14 marks)

213. The following diagram shows a pentagon ABCDE, with AB = 9.2 cm, BC = 3.2 cm, BD = 7.1
cm, AÊD =110, AD̂E = 52 and AB̂D = 60.

(a) Find AD.


(4)

(b) Find DE.


(4)

(c) The area of triangle BCD is 5.68 cm2. Find DB̂C .


(4)

(d) Find AC.


(4)
(e) Find the area of quadrilateral ABCD.
(5)
(Total 21 marks)

 x
214. The diagram below shows the graph of f (x) = 1 + tan   for −360  x  360.
2

(a) On the same diagram, draw the asymptotes.


(2)

(b) Write down


(i) the period of the function;
(ii) the value of f (90).
(2)

(c) Solve f (x) = 0 for −360  x  360.


(2)
(Total 6 marks)
215. A Ferris wheel with centre O and a radius of 15 metres is represented in the diagram below.
π
Initially seat A is at ground level. The next seat is B, where AÔB = .
6

(a) Find the length of the arc AB.


(2)

(b) Find the area of the sector AOB.


(2)


(c) The wheel turns clockwise through an angle of . Find the height of A above the
3
ground.
(3)

The height, h metres, of seat C above the ground after t minutes, can be modelled by the
function
 π
h (t) = 15 − 15 cos  2t   .
 4
π
(d) (i) Find the height of seat C when t = .
4
(ii) Find the initial height of seat C.
(iii) Find the time at which seat C first reaches its highest point.
(8)
(e) Find h′ (t).
(2)

(f) For 0  t  ,
(i) sketch the graph of h′;
(ii) find the time at which the height is changing most rapidly.
(5)
(Total 22 marks)

216. The following diagram shows a circle with radius r and centre O. The points A, B and C are on
the circle and AÔC =.

4 2
The area of sector OABC is  and the length of arc ABC is .
3 3
Find the value of r and of .
(Total 6 marks)
217. Let ƒ (x) = a sin b (x − c). Part of the graph of ƒ is given below.

Given that a, b and c are positive, find the value of a, of b and of c.


(Total 6 marks)

218. The points P(−2, 4), Q (3, 1) and R (1, 6) are shown in the diagram below.

(a) Find the vector PQ .


(b) Find a vector equation for the line through R parallel to the line (PQ).
(Total 6 marks)
219. The diagram below shows a circle of radius r and centre O. The angle AÔB = .

The length of the arc AB is 24 cm. The area of the sector OAB is 180 cm2.
Find the value of r and of .
(Total 6 marks)
220. The diagram below shows a quadrilateral ABCD. AB = 4, AD = 8, CD =12, B Ĉ D = 25,
BÂD =.

(a) Use the cosine rule to show that BD = 4 5  4 cos  .


(2)
Let  = 40.
(b) (i) Find the value of sin CB̂D .
(ii) Find the two possible values for the size of CB̂D .
(iii) Given that CB̂D is an acute angle, find the perimeter of ABCD.
(12)
(c) Find the area of triangle ABD.
(2)
(Total 16 marks)

π
221. The function f is defined by f : x  30 sin 3x cos 3x, 0  x  .
3
(a) Write down an expression for f (x) in the form a sin 6x, where a is an integer.
(b) Solve f (x) = 0, giving your answers in terms of .
(Total 6 marks)
222. The following diagram shows two semi-circles. The larger one has centre O and radius 4 cm.
The smaller one has centre P, radius 3 cm, and passes through O. The line (OP) meets the larger
semi-circle at S. The semi-circles intersect at Q.

(a) (i) Explain why OPQ is an isosceles triangle.


1
(ii) Use the cosine rule to show that cos OP̂Q = .
9
80
(iii) Hence show that sin OP̂Q = .
9
(iv) Find the area of the triangle OPQ.
(7)

(b) Consider the smaller semi-circle, with centre P.


(i) Write down the size of OP̂Q.
(ii) Calculate the area of the sector OPQ.
(3)

(c) Consider the larger semi-circle, with centre O. Calculate the area of the sector QOS.
(3)

(d) Hence calculate the area of the shaded region.


(4)
(Total 17 marks)
223. The graph of a function of the form y = p cos qx is given in the diagram below.
y
40
30
20
10
x
/2 
–10
–20
–30
–40

(a) Write down the value of p.


(b) Calculate the value of q.
(Total 6 marks)

224. A farmer owns a triangular field ABC. One side of the triangle, [AC], is 104 m, a second side,
[AB], is 65 m and the angle between these two sides is 60°.
(a) Use the cosine rule to calculate the length of the third side of the field.
(3)

3
(b) Given that sin 60° = , find the area of the field in the form p 3 where p is an integer.
2
(3)
Let D be a point on [BC] such that [AD] bisects the 60° angle. The farmer divides the field into
two parts A1 and A2 by constructing a straight fence [AD] of length x metres, as shown on the
diagram below.
C

104 m

A2
30°
A D
x
30°
A1

65 m

65x
(c) (i) Show that the area of Al is given by .
4
(ii) Find a similar expression for the area of A2.
(iii) Hence, find the value of x in the form q 3 , where q is an integer.
(7)

(d) (i) Explain why sin AD̂C  sin AD̂B .


(ii) Use the result of part (i) and the sine rule to show that
BD 5
 .
DC 8
(5)
(Total 18 marks)
225. The following diagram shows a circle of centre O, and radius r. The shaded sector OACB has
an area of 27 cm2. Angle AÔB = θ = 1.5 radians.
A
C

B
r
O

(a) Find the radius.


(b) Calculate the length of the minor arc ACB.
Working:
Answers:
(a) ..................................................................
(b) ..................................................................
(Total 6 marks)
226. Consider y = sin  x    .
 9
(a) The graph of y intersects the x-axis at point A. Find the x-coordinate of A, where
0  x  π.
(b) Solve the equation sin  x    = – 1 , for 0  x  2.
 9 2
Working:
Answers:
(a) ..................................................................
(b) ..................................................................
(Total 6 marks)

227. The diagram shows a triangular region formed by a hedge [AB], a part of a river bank [AC] and
a fence [BC]. The hedge is 17 m long and BÂC is 29°. The end of the fence, point C, can be
positioned anywhere along the river bank.
(a) Given that point C is 15 m from A, find the length of the fence [BC].
A 15 m C river bank
29°

17 m

B
(3)
(b) The farmer has another, longer fence. It is possible for him to enclose two different
triangular regions with this fence. He places the fence so that AB̂C is 85°.
(i) Find the distance from A to C.
(ii) Find the area of the region ABC with the fence in this position.
(5)

(c) To form the second region, he moves the fencing so that point C is closer to point A.
Find the new distance from A to C.
(4)

(d) Find the minimum length of fence [BC] needed to enclose a triangular region ABC.
(2)
(Total 14 marks)

228. Let f (x) = 1 sin 2x + cos x for 0  x  2.


2
(a) (i) Find f (x).
One way of writing f (x) is –2 sin2 x – sin x + 1.
(ii) Factorize 2 sin2 x + sin x – 1.
(iii) Hence or otherwise, solve f (x) = 0.
(6)

The graph of y = f (x) is shown below.


y
A

b
0 a 2 x

B
There is a maximum point at A and a minimum point at B.
(b) Write down the x-coordinate of point A.
(1)

(c) The region bounded by the graph, the x-axis and the lines x = a and x = b is shaded in the
diagram above.
(i) Write down an expression that represents the area of this shaded region.
(ii) Calculate the area of this shaded region.
(5)
(Total 12 marks)

229. In triangle PQR, PQ is 10 cm, QR is 8 cm and angle PQR is acute. The area of the triangle is 20
cm2. Find the size of angle PQ̂R.
Working:

Answers:

........................................................
(Total 6 marks)
230. Consider the equation 3 cos 2x + sin x = 1
(a) Write this equation in the form f (x) = 0 , where f (x) = p sin2 x + q sin x + r , and p , q , r
 .
(b) Factorize f (x).
(c) Write down the number of solutions of f (x) = 0, for 0  x  2.
Working:

Answers:

(a) .....................................................
(b) .....................................................
(c) .....................................................
(Total 6 marks)

231. The diagram below shows two circles which have the same centre O and radii 16 cm and 10 cm
respectively. The two arcs AB and CD have the same sector angle  = 1.5 radians.

A B

C D

O
Find the area of the shaded region.
Working:
Answer:
…………………………………………..
(Total 6 marks)
232. Let f (x) = sin (2x + 1), 0  x  π.
(a) Sketch the curve of y = f (x) on the grid below.
y
2

1.5

0.5

0
0.5 1 1.5 2 2.5 3 3.5 x

–0.5

–1

–1.5

–2
(b) Find the x-coordinates of the maximum and minimum points of f (x), giving your answers
correct to one decimal place.
Working:
Answer:
(b) …………………………………………..
(Total 6 marks)
233. In a triangle ABC, AB = 4 cm, AC = 3 cm and the area of the triangle is 4.5 cm2.
Find the two possible values of the angle BÂC .
Working:
Answer:
…………………………………………..
(Total 6 marks)

234. Solve the equation 2 cos2 x = sin 2x for 0  x  π, giving your answers in terms of π.
Working:
Answer:
…………………………………………..
(Total 6 marks)
235. The depth y metres of water in a harbour is given by the equation
t
y = 10 + 4 sin   ,
 2
where t is the number of hours after midnight.
(a) Calculate the depth of the water
(i) when t = 2;
(ii) at 2100.
(3)

The sketch below shows the depth y, of water, at time t, during one day (24 hours).
y
15
14
13
12
11
10
9
depth (metres)
8
7
6
5
4
3
2
1

1 2 3 4 5 6 7 8 9 10 11 12 13 14 15 16 17 18 19 20 21 22 23 24 t
time (hours)

(b) (i) Write down the maximum depth of water in the harbour.
(ii) Calculate the value of t when the water is first at its maximum depth during the
day.
(3)
The harbour gates are closed when the depth of the water is less than seven metres. An alarm
rings when the gates are opened or closed.
(c) (i) How many times does the alarm sound during the day?
(ii) Find the value of t when the alarm sounds first.
(iii) Use the graph to find the length of time during the day when the harbour gates are
closed. Give your answer in hours, to the nearest hour.
(7)
(Total 13 marks)

236. The following diagram shows a triangle ABC, where BC = 5 cm, B̂ = 60°, Ĉ = 40°.
A

60° 40°
B C
5 cm
(a) Calculate AB.
(b) Find the area of the triangle.
Working:
Answers:
(a) …………………………………………..
(b) …………………………………………..
(Total 6 marks)
237. The diagram below shows a circle of radius 5 cm with centre O. Points A and B are on the
circle, and AÔB is 0.8 radians. The point N is on [OB] such that [AN] is perpendicular to
[OB].
A

5 cm

0.8
O B
N

Find the area of the shaded region.


Working:
Answer:
…………………………………………........
(Total 6 marks)
238. Part of the graph of y = p + q cos x is shown below. The graph passes through the points (0, 3)
and (, –1).
y

0
 2 x

–1

Find the value of


(a) p;
(b) q.
Working:
Answers:
(a) ..................................................................
(b) ..................................................................
(Total 6 marks)
239. Find all solutions of the equation cos 3x = cos (0.5x), for 0  x  .
Working:
Answer:
..................................................................
(Total 6 marks)
240. The diagram below shows a triangle and two arcs of circles.
The triangle ABC is a right-angled isosceles triangle, with AB = AC = 2. The point P is the
midpoint of [BC].
The arc BDC is part of a circle with centre A.
The arc BEC is part of a circle with centre P.

E
B

P
2

A 2 C
(a) Calculate the area of the segment BDCP.
(b) Calculate the area of the shaded region BECD.
Working:
Answers:
(a) ..................................................................
(b) ..................................................................
(Total 6 marks)
7 10 
241. The diagram shows a parallelogram OPQR in which OP =   , OQ =  .
 3 1
y

Q
O x
R

(a) Find the vector OR .


(3)

15
(b) Use the scalar product of two vectors to show that cos OP̂Q = – .
754
(4)

(c) (i) Explain why cos PQ̂R = –cos OP̂Q.


23
(ii) Hence show that sin PQ̂R = .
754
(iii) Calculate the area of the parallelogram OPQR, giving your answer as an integer.
(7)
(Total 14 marks)
242. The points P, Q, R are three markers on level ground, joined by straight paths PQ, QR, PR as
shown in the diagram. QR = 9 km, PQ̂R = 35°, PR̂Q = 25°.
P
Diagram not to scale
35° 25°
Q R
9 km
(a) Find the length PR.
(3)
(b) Tom sets out to walk from Q to P at a steady speed of 8 km h–1. At the same time, Alan
sets out to jog from R to P at a steady speed of a km h–1. They reach P at the same time.
Calculate the value of a.
(7)
(c) The point S is on [PQ], such that RS = 2QS, as shown in the diagram.
P
S

Q R
Find the length QS.
(6)
(Total 16 marks)

243. Consider the function f (x) = cos x + sin x.


π
(a) (i) Show that f (– ) = 0.
4
(ii) Find in terms of , the smallest positive value of x which satisfies f (x) = 0.
(3)
The diagram shows the graph of y = ex (cos x + sin x), – 2  x  3. The graph has a maximum
turning point at C(a, b) and a point of inflexion at D.
y

6 C(a, b)

4
D
2

x
–2 –1 1 2 3

dy
(b) Find .
dx
(3)

(c) Find the exact value of a and of b.


(4)

π
(d) Show that at D, y = 2e 4 .
(5)

(e) Find the area of the shaded region.


(2)
(Total 17 marks)
244. The graph of the function f (x) = 3x – 4 intersects the x-axis at A and the y-axis at B.
(a) Find the coordinates of
(i) A;
(ii) B.
(b) Let O denote the origin. Find the area of triangle OAB.
Working:
Answers:
(a) (i) ...........................................................
(ii) ...........................................................
(b) ..................................................................
(Total 6 marks)
245. (a) Factorize the expression 3 sin2 x – 11 sin x + 6.
(b) Consider the equation 3 sin2 x – 11 sin x + 6 = 0.
(i) Find the two values of sin x which satisfy this equation,
(ii) Solve the equation, for 0°  x  180°.
Working:
Answers:
(a) ..................................................................
(b) (i) ...........................................................
(ii) ...........................................................

(Total 6 marks)

246. The diagram below shows a circle, centre O, with a radius 12 cm. The chord AB subtends at an
angle of 75° at the centre. The tangents to the circle at A and at B meet at P.

A
12 cm

P diagram not to
O 75º scale

(a) Using the cosine rule, show that the length of AB is 12 21 – cos 75 .
(2)

(b) Find the length of BP.


(3)

(c) Hence find


(i) the area of triangle OBP;
(ii) the area of triangle ABP.
(4)

(d) Find the area of sector OAB.


(2)

(e) Find the area of the shaded region.


(2)
(Total 13 marks)

247. Note: Radians are used throughout this question.


A mass is suspended from the ceiling on a spring. It is pulled down to point P and then released.
It oscillates up and down.
diagram not to
scale

Its distance, s cm, from the ceiling, is modelled by the function s = 48 + 10 cos 2πt where t is
the time in seconds from release.
(a) (i) What is the distance of the point P from the ceiling?
(ii) How long is it until the mass is next at P?
(5)

ds
(b) (i) Find .
dt
(ii) Where is the mass when the velocity is zero?
(7)
A second mass is suspended on another spring. Its distance r cm from the ceiling is modelled by
the function r = 60 + 15 cos 4t. The two masses are released at the same instant.
(c) Find the value of t when they are first at the same distance below the ceiling.
(2)

(d) In the first three seconds, how many times are the two masses at the same height?
(2)
(Total 16 marks)
248. The following diagram shows a circle of centre O, and radius 15 cm. The arc ACB subtends an
angle of 2 radians at the centre O.
C
A B

15
Diagram not to scale

cm
2 rad

O AÔB = 2 radians
OA = 15 cm

Find
(a) the length of the arc ACB;
(b) the area of the shaded region.
Working:
Answers:
(a) ..................................................................
(b) ..................................................................
(Total 6 marks)
249. Two boats A and B start moving from the same point P. Boat A moves in a straight line at
20 km h–1 and boat B moves in a straight line at 32 km h–1. The angle between their paths is
70°.
Find the distance between the boats after 2.5 hours.
Working:
Answer:
......................................................................
(Total 6 marks)
250. Let f (x) = sin 2x and g (x) = sin (0.5x).
(a) Write down
(i) the minimum value of the function f ;
(ii) the period of the function g.
(b) Consider the equation f (x) = g (x).

Find the number of solutions to this equation, for 0  x  .
2
Working:
Answers:
(a) (i) ..........................................................
(ii) ..........................................................
(b) .................................................................
(Total 6 marks)
251. Consider the following statements
A: log10 (10x) > 0.
B: –0.5  cos (0.5x)  0.5.
π π
C: –  arctan x  .
2 2
(a) Determine which statements are true for all real numbers x. Write your answers (yes or
no) in the table below.
Statement (a) Is the statement true for all (b) If not true, example
real numbers x? (Yes/No)
A
B
C

(b) If a statement is not true for all x, complete the last column by giving an example of one
value of x for which the statement is false.
Working:
(Total 6 marks)
2
252. The diagram shows a triangle ABC in which AC = 7 , BC = 6, AB̂C = 45°.
2
A
Diagram 2
7 2
not to scale

45°
B 6 C
2 6
(a) Use the fact that sin 45° = to show that sin BÂC = .
2 7
(2)
6
The point D is on (AB), between A and B, such that sin BD̂C = .
7
(b) (i) Write down the value of BD̂C + BÂC .
(ii) Calculate the angle BCD.
(iii) Find the length of [BD].
(6)
Area of BDC BD
(c) Show that = .
Area of BAC BA
(2)
(Total 10 marks)
253. In triangle ABC, AC = 5, BC = 7, Â = 48°, as shown in the diagram.
C

5 7 diagram not to scale

48°
A B
Find B̂, giving your answer correct to the nearest degree.
Working:
Answer:
......................................................................
(Total 6 marks)
1
254. Given that sin x = , where x is an acute angle, find the exact value of
3
(a) cos x;
(b) cos 2x.
Working:
Answers:
(a) ..................................................................
(b) ..................................................................
(Total 6 marks)
255. Consider the trigonometric equation 2 sin2 x = 1 + cos x.
(a) Write this equation in the form f (x) = 0, where f (x) = a cos2 x + b cos x + c,
and a, b, c  .
(b) Factorize f (x).
(c) Solve f (x) = 0 for 0°  x  360°.
Working:
Answers:
(a) ..................................................................
(b) ..................................................................
(c) ..................................................................
(Total 6 marks)
256. The following diagram shows a triangle with sides 5 cm, 7 cm, 8 cm.

5 7
Diagram not to scale

8
Find
(a) the size of the smallest angle, in degrees;
(b) the area of the triangle.
Working:
Answers:
(a) ..................................................................
(b) ..................................................................
(Total 4 marks)
257. (a) Write the expression 3 sin2 x + 4 cos x in the form a cos2 x + b cos x + c.
(b) Hence or otherwise, solve the equation
3 sin2 x + 4 cos x – 4 = 0, 0  x  90.
Working:
Answers:
(a) ..................................................................
(b) ..................................................................
(Total 4 marks)
258. In the following diagram, O is the centre of the circle and (AT) is the tangent to the circle at T.
T

O A

Diagram not to scale


If OA = 12 cm, and the circle has a radius of 6 cm, find the area of the shaded region.
Working:
Answer:
.......................................................................
(Total 4 marks)

259. The diagram below shows a sector AOB of a circle of radius 15 cm and centre O. The angle  at
the centre of the circle is 2 radians.
Diagram not to scale

A B

O
(a) Calculate the area of the sector AOB.
(b) Calculate the area of the shaded region.
Working:
Answers:
(a) ..................................................................
(b) ..................................................................
(Total 4 marks)

260. The diagrams below show two triangles both satisfying the conditions
AB = 20 cm, AC = 17 cm, AB̂C = 50°.
Diagrams not
to scale
Triangle 1 Triangle 2
A A

B C B C
(a) Calculate the size of AĈB in Triangle 2.
(b) Calculate the area of Triangle 1.
Working:
Answers:
(a) ..................................................................
(b) ..................................................................
(Total 4 marks)
261. The depth, y metres, of sea water in a bay t hours after midnight may be represented by the
function
 2 
y  a  b cos  t  , where a, b and k are constants.
 k 
The water is at a maximum depth of 14.3 m at midnight and noon, and is at a minimum depth of
10.3 m at 06:00 and at 18:00.
Write down the value of
(a) a;
(b) b;
(c) k.
Working:
Answers:
(a) ..................................................................
(b) ..................................................................
(c) ..................................................................
(Total 4 marks)

262. Town A is 48 km from town B and 32 km from town C as shown in the diagram.
C

32km

A
48km B
Given that town B is 56 km from town C, find the size of angle CÂB to the nearest degree.
Working:
Answer:
....................................................................
(Total 4 marks)

263. (a) Express 2 cos2 x + sin x in terms of sin x only.


(b) Solve the equation 2 cos2 x + sin x = 2 for x in the interval 0  x  , giving your answers
exactly.
Working:
Answers:
(a) ..................................................................
(b) ..................................................................
(Total 4 marks)
264. A formula for the depth d metres of water in a harbour at a time t hours after midnight is
 
d  P  Q cos  t , 0  t  24,
6 
where P and Q are positive constants. In the following graph the point (6, 8.2) is a minimum
point and the point (12, 14.6) is a maximum point.
d
15 (12, 14.6)

10.

(6, 8.2)
5

0 6 12 18 24 t

(a) Find the value of


(i) Q;
(ii) P.
(3)

(b) Find the first time in the 24-hour period when the depth of the water is 10 metres.
(3)

(c) (i) Use the symmetry of the graph to find the next time when the depth of the water is
10 metres.
(ii) Hence find the time intervals in the 24-hour period during which the water is less
than 10 metres deep.
(4)
265. Solve the equation 3 cos x = 5 sin x, for x in the interval 0°  x  360°, giving your answers to
the nearest degree.
Working:
Answer:
......................................................................
(Total 4 marks)

266. If A is an obtuse angle in a triangle and sin A = 5 , calculate the exact value of sin 2A.
13
Working:
Answer:
......................................................................
(Total 4 marks)

267. (a) Sketch the graph of y =  sin x – x, –3  x  3, on millimetre square paper, using a scale
of 2 cm per unit on each axis.
Label and number both axes and indicate clearly the approximate positions of the
x-intercepts and the local maximum and minimum points.
(5)

(b) Find the solution of the equation


 sin x – x = 0, x > 0.
(1)

(c) Find the indefinite integral


 ( sin x  x)dx
and hence, or otherwise, calculate the area of the region enclosed by the graph, the
x-axis and the line x = 1.
(4)
(Total 10 marks)

3
268. Given that sin θ = 1 , cos θ = – and 0° ≤ θ ≤ 360°,
2 2
(a) find the value of θ;

(b) write down the exact value of tan θ.


Working:
Answers:
(a) ..................................................................
(b) ..................................................................
(Total 4 marks)
269. The diagram shows a vertical pole PQ, which is supported by two wires fixed to the horizontal
ground at A and B.
P

36 B
30
Q 70
A
BQ = 40 m
PB̂Q = 36°
BÂQ = 70°
AB̂Q = 30°

Find
(a) the height of the pole, PQ;
(b) the distance between A and B.
Working:
Answers:
(a) ..................................................................
(b) ..................................................................
(Total 4 marks)
270. The diagram shows a circle of radius 5 cm.
1 radian

Find the perimeter of the shaded region.


Working:
Answer:
......................................................................
(Total 4 marks)
271. In this question you should note that radians are used throughout.
(a) (i) Sketch the graph of y = x2 cos x, for 0  x  2 making clear the approximate
positions of the positive x-intercept, the maximum point and the end-points.
(ii) Write down the approximate coordinates of the positive x-intercept, the maximum
point and the end-points.
(7)
(b) Find the exact value of the positive x-intercept for 0  x  2.
(2)
Let R be the region in the first quadrant enclosed by the graph and the x-axis.
(c) (i) Shade R on your diagram.
(ii) Write down an integral which represents the area of R.
(3)
(d) Evaluate the integral in part (c)(ii), either by using a graphic display calculator, or by
using the following information.
d (x2 sin x + 2x cos x – 2 sin x) = x2 cos x.
dx
(3)
(Total 15 marks)
272. In this part of the question, radians are used throughout.
The function f is given by
f (x) = (sin x)2 cos x.
The following diagram shows part of the graph of y = f (x).
y

B x
O

The point A is a maximum point, the point B lies on the x-axis, and the point C is a point of
inflexion.

(a) Give the period of f.


(1)

(b) From consideration of the graph of y = f (x), find to an accuracy of one significant
figure the range of f.
(1)
(c) (i) Find f (x).
(ii) Hence show that at the point A, cos x = 1.
3
(iii) Find the exact maximum value.
(9)

(d) Find the exact value of the x-coordinate at the point B.


(1)

(e) (i) Find  f (x) dx.


(ii) Find the area of the shaded region in the diagram.
(4)

(f) Given that f (x) = 9(cos x)3 – 7 cos x, find the x-coordinate at the point C.
(4)
(Total 20 marks)

273. A triangle has sides of length 4, 5, 7 units. Find, to the nearest tenth of a degree, the size of the
largest angle.
Working:
Answer:
......................................................................
(Total 4 marks)

274. O is the centre of the circle which has a radius of 5.4 cm.

A B

The area of the shaded sector OAB is 21.6 cm2. Find the length of the minor arc AB.
Working:
Answer:
......................................................................
(Total 4 marks)
6   6
275. The circle shown has centre O and radius 6. OA is the vector   , OB is the vector  
0  0 
 5 
and OC is the vector   .
 11 
y

B O A x

(a) Verify that A, B and C lie on the circle.


(3)
(b) Find the vector AC .
(2)

(c) Using an appropriate scalar product, or otherwise, find the cosine of angle OAˆ C .
(3)
(d) Find the area of triangle ABC, giving your answer in the form a 11 , where a  .
(4)
(Total 12 marks)
276. Solve the equation 3 sin2 x = cos2 x, for 0°  x  180°.
Working:
Answer:
......................................................................
(Total 4 marks)

277. The diagrams show a circular sector of radius 10 cm and angle θ radians which is formed into a
cone of slant height 10 cm. The vertical height h of the cone is equal to the radius r of its base.
Find the angle θ radians.

10cm

10cm
h

r
Working:
Answer:
......................................................................
(Total 4 marks)

278. The diagram shows the graph of the function f given by


f (x) = A sin   x  + B,
2 
for 0  x  5, where A and B are constants, and x is measured in radians.
y (1,3) (5, 3)

(0, 1)
x
0 1 2 3 4 5

(3, –1)

The graph includes the points (1, 3) and (5, 3), which are maximum points of the graph.
(a) Write down the values of f (1) and f (5).
(2)
(b) Show that the period of f is 4.
(2)
The point (3, –1) is a minimum point of the graph.
(c) Show that A = 2, and find the value of B.
(5)
(d) Show that f (x) =  cos   x  .
2 
(4)

The line y = k – x is a tangent line to the graph for 0  x  5.


(e) Find
(i) the point where this tangent meets the curve;
(ii) the value of k.
(6)
(f) Solve the equation f (x) = 2 for 0  x  5.
(5)
(Total 24 marks)

3
279. A scientist has 100 female fish and 100 male fish. She measures their lengths to the nearest cm.
These are shown in the following box and whisker diagrams.

(a) Find the range of the lengths of all 200 fish.


(3)

(b) Four cumulative frequency graphs are shown below.

Which graph is the best representation of the lengths of the female fish?
(2)
(Total 5 marks)
280. The following frequency distribution of marks has mean 4.5.

Mark 1 2 3 4 5 6 7
Frequency 2 4 6 9 x 9 4

(a) Find the value of x.


(4)

(b) Write down the standard deviation.


(2)
(Total 6 marks)

281. The following table gives the examination grades for 120 students.

Grade Number of students Cumulative frequency


1 9 9
2 25 34
3 35 p
4 q 109
5 11 120

(a) Find the value of


(i) p;
(ii) q.
(4)

(b) Find the mean grade.


(2)

(c) Write down the standard deviation.


(1)
(Total 7 marks)

282. A standard die is rolled 36 times. The results are shown in the following table.

Score 1 2 3 4 5 6
Frequency 3 5 4 6 10 8

(a) Write down the standard deviation.


(2)

(b) Write down the median score.


(1)

(c) Find the interquartile range.


(3)
(Total 6 marks)

283. A fisherman catches 200 fish to sell. He measures the lengths, l cm of these fish, and the results
are shown in the frequency table below.

Length l cm 0 ≤ l < 10 10 ≤ l < 20 20 ≤ l < 30 30 ≤ l < 40 40 ≤ l < 60 60 ≤ l < 75 75 ≤ l < 100


Frequency 30 40 50 30 33 11 6

(a) Calculate an estimate for the standard deviation of the lengths of the fish.
(3)
(b) A cumulative frequency diagram is given below for the lengths of the fish.

Use the graph to answer the following.


(i) Estimate the interquartile range.
(ii) Given that 40 % of the fish have a length more than k cm, find the value of k.
(6)

In order to sell the fish, the fisherman classifies them as small, medium or large.
Small fish have a length less than 20 cm.
Medium fish have a length greater than or equal to 20 cm but less than 60 cm.
Large fish have a length greater than or equal to 60 cm.
(c) Write down the probability that a fish is small.
(2)
The cost of a small fish is $4, a medium fish $10, and a large fish $12.
(d) Copy and complete the following table, which gives a probability distribution for the cost
$X.

Cost $X 4 10 12
P(X = x) 0.565
(2)

(e) Find E(X).


(2)
(Total 15 marks)

284. The following diagram is a box and whisker plot for a set of data.

The interquartile range is 20 and the range is 40.

(a) Write down the median value.


(1)

(b) Find the value of


(i) a;
(ii) b.
(4)
(Total 5 marks)
285. A box contains 100 cards. Each card has a number between one and six written on it.
The following table shows the frequencies for each number.
Number 1 2 3 4 5 6
Frequency 26 10 20 k 29 11
(a) Calculate the value of k.
(2)
(b) Find
(i) the median;
(ii) the interquartile range.
(5)
(Total 7 marks)

286. In a school with 125 girls, each student is tested to see how many sit-up exercises (sit-ups) she
can do in one minute. The results are given in the table below.
Cumulative
Number of sit-ups Number of students
number of students
15 11 11
16 21 32
17 33 p
18 q 99
19 18 117
20 8 125
(a) (i) Write down the value of p.
(ii) Find the value of q.
(3)
(b) Find the median number of sit-ups.
(2)
(c) Find the mean number of sit-ups.
(2)
(Total 7 marks)
287. The following is a cumulative frequency diagram for the time t, in minutes, taken by 80 students
to complete a task.

(a) Write down the median.


(1)

(b) Find the interquartile range.


(3)
(c) Complete the frequency table below.

Time Number of
(minutes) students
0 ≤ t < 10 5
10 ≤ t < 20
20 ≤ t < 30 20
30 ≤ t < 40 24
40 ≤ t < 50
50 ≤ t < 60 6
(2)
(Total 6 marks)

288. A test marked out of 100 is written by 800 students. The cumulative frequency graph for the
marks is given below.
(a) Write down the number of students who scored 40 marks or less on the test.
(2)

(b) The middle 50 % of test results lie between marks a and b, where a < b.
Find a and b.
(4)
(Total 6 marks)

289. The cumulative frequency graph below shows the heights of 120 girls in a school.
130

120

110

100

90
Cumulative frequency

80

70

60

50

40

30

20

10

0
150 155 160 165 170 175 180 185
Height in centimetres
(a) Using the graph
(i) write down the median;
(ii) find the interquartile range.
(b) Given that 60 of the girls are taller than a cm, find the value of a.
(Total 6 marks)

290. A set of data is


18, 18, 19, 19, 20, 22, 22, 23, 27, 28, 28, 31, 34, 34, 36.
The box and whisker plot for this data is shown below.

(a) Write down the values of A, B, C, D and E.


A = ...... B = ...... C= ...... D = ...... E = ......
(b) Find the interquartile range.
(Total 6 marks)
291. There are 50 boxes in a factory. Their weights, w kg, are divided into 5 classes, as shown in the
following table.
Class Weight (kg) Number of boxes
A 9.5  w 18.5 7
B 18.5  w  27.5 12
C 27.5  w  36.5 13
D 36.5  w  45.5 10
E 45.5  w  54.5 8
(a) Show that the estimated mean weight of the boxes is 32 kg.
(3)
(b) There are x boxes in the factory marked “Fragile”. They are all in class E. The estimated
mean weight of all the other boxes in the factory is 30 kg. Calculate the value of x.
(4)
(c) An additional y boxes, all with a weight in class D, are delivered to the factory. The total
estimated mean weight of all of the boxes in the factory is less than 33 kg. Find the
largest possible value of y.
(5)
(Total 12 marks)
292. The histogram below represents the ages of 270 people in a village.

(a) Use the histogram to complete the table below.


Mid-interval
Age range Frequency
value
0  age  20 40 10
20 ≤ age  40
40 ≤ age  60
60 ≤ age  80
80 ≤ age ≤100
(2)

(b) Hence, calculate an estimate of the mean age.


(4)
(Total 6 marks)
293. Consider the four numbers a, b, c, d with a  b  c ≤ d, where a, b, c, d  .
The mean of the four numbers is 4.
The mode is 3.
The median is 3.
The range is 6.
Find the value of a, of b, of c and of d.
(Total 6 marks)

294. The population below is listed in ascending order.


5, 6, 7, 7, 9, 9, r, 10, s, 13, 13, t
The median of the population is 9.5. The upper quartile Q3 is 13.
(a) Write down the value of
(i) r;
(ii) s.
(b) The mean of the population is 10. Find the value of t.
(Total 6 marks)
295. The four populations A, B, C and D are the same size and have the same range.
Frequency histograms for the four populations are given below.

(a) Each of the three box and whisker plots below corresponds to one of the four populations.
Write the letter of the correct population under each plot.

...... ...... ......


(b) Each of the three cumulative frequency diagrams below corresponds to one of the four
populations. Write the letter of the correct population under each diagram.

(Total 6 marks)
296. The box and whisker diagram shown below represents the marks received by 32 students.

(a) Write down the value of the median mark.


(b) Write down the value of the upper quartile.
(c) Estimate the number of students who received a mark greater than 6.
(Total 6 marks)
297. The following is the cumulative frequency curve for the time, t minutes, spent by 150 people in
a store on a particular day.

150

140

130

120

110

100
cumulative frequency

90

80

70

60

50

40

30

20

10

0
1 2 3 4 5 6 7 8 9 10 11 12

time (t)
(a) (i) How many people spent less than 5 minutes in the store?
(ii) Find the number of people who spent between 5 and 7 minutes in the store.
(iii) Find the median time spent in the store.
(6)
(b) Given that 40 of the people spent longer than k minutes, find the value of k.
(3)

(c) (i) On your answer sheet, copy and complete the following frequency table.
t (minutes) 0t2 2t4 4t6 6t8 8  t  10 10  t  12
Frequency 10 23 15
(ii) Hence, calculate an estimate for the mean time spent in the store.
(5)
(Total 14 marks)

298. A test marked out of 100 is written by 800 students. The cumulative frequency graph for the
marks is given below.

800

700

600
Number
of 500
candidates
400

300

200

100

10 20 30 40 50 60 70 80 90 100

Mark
(a) Write down the number of students who scored 40 marks or less on the test.
(b) The middle 50% of test results lie between marks a and b, where a < b. Find a and b.
(Total 6 marks)

299. The 45 students in a class each recorded the number of whole minutes, x, spent doing
experiments on Monday. The results are x = 2230.
(a) Find the mean number of minutes the students spent doing experiments on Monday.
Two new students joined the class and reported that they spent 37 minutes and 30 minutes
respectively.
(b) Calculate the new mean including these two students.
Working:
Answers:
(a) ..................................................................
(b) ..................................................................
(Total 6 marks)
300. The following table shows the mathematics marks scored by students.
Mark 1 2 3 4 5 6 7
Frequency 0 4 6 k 8 6 6
The mean mark is 4.6.
(a) Find the value of k.
(b) Write down the mode.
Working:

Answers:

(a) .................................................
(b) .................................................
(Total 6 marks)

301. In the research department of a university, 300 mice were timed as they each ran through a
maze. The results are shown in the cumulative frequency diagram opposite.
(a) How many mice complete the maze in less than 10 seconds?
(1)
(b) Estimate the median time.
(1)
(c) Another way of showing the results is the frequency table below.
Time t (seconds) Number of mice
t7 0
7t8 16
8t9 22
9  t  10 p
10  t  11 q
11  t  12 70
12  t  13 44
13  t  14 31
14  t  15 23
(i) Find the value of p and the value of q.
(ii) Calculate an estimate of the mean time.
(4)
(Total 6 marks)
302. The cumulative frequency curve below shows the marks obtained in an examination by a group
of 200 students.
200

190

180

170

160

150

140
Number
of 130
students
120

110

100

90

80

70

60

50

40

30

20

10

0
10 20 30 40 50 60 70 80 90 100
Mark obtained

(a) Use the cumulative frequency curve to complete the frequency table below.
Mark (x) 0  x < 20 20  x < 40 40  x < 60 60  x < 80 80  x < 100
Number of 22 20
students
(b) Forty percent of the students fail. Find the pass mark.
Working:
Answer:
(b) …………………………………………..
(Total 6 marks)
303. The table below shows the marks gained in a test by a group of students.
Mark 1 2 3 4 5
Number of students 5 10 p 6 2
The median is 3 and the mode is 2. Find the two possible values of p.
Working:
Answer:
…………………………………………..
(Total 6 marks)

304. The cumulative frequency curve below shows the heights of 120 basketball players in
centimetres.
120

110

100

90

80

70

60
Number of players
50

40

30

20

10

0
160 165 170 175 180 185 190 195 200
Height in centimetres
Use the curve to estimate
(a) the median height;
(b) the interquartile range.
Working:
Answers:
(a) …………………………………………..
(b) …………………………………………..
(Total 6 marks)

305. Let a, b, c and d be integers such that a < b, b < c and c = d.


The mode of these four numbers is 11.
The range of these four numbers is 8.
The mean of these four numbers is 8.
Calculate the value of each of the integers a, b, c, d.
Working:
Answers:
a = ............................., b = .............................
c = ............................., d = .............................
(Total 6 marks)

306. The number of hours of sleep of 21 students are shown in the frequency table below.
Hours of sleep Number of students
4 2
5 5
6 4
7 3
8 4
10 2
12 1
Find
(a) the median;
(b) the lower quartile;
(c) the interquartile range.
Working:
Answers:
(a) ..................................................................
(b) ..................................................................
(c) ..................................................................
(Total 6 marks)
307. A student measured the diameters of 80 snail shells. His results are shown in the following
cumulative frequency graph. The lower quartile (LQ) is 14 mm and is marked clearly on the
graph.
90
80
70
Cumulative frequency

60
50
40
30
20
10
0
0 5 10 15 20 25 30 35 40 45
LQ = 14
Diameter (mm)
(a) On the graph, mark clearly in the same way and write down the value of
(i) the median;
(ii) the upper quartile.
(b) Write down the interquartile range.
Working:
Answer:
(b) ..................................................................
(Total 6 marks)
308. From January to September, the mean number of car accidents per month was 630. From
October to December, the mean was 810 accidents per month.
What was the mean number of car accidents per month for the whole year?
Working:
Answer:
......................................................................
(Total 6 marks)
309. A taxi company has 200 taxi cabs. The cumulative frequency curve below shows the fares in
dollars ($) taken by the cabs on a particular morning.
200

180

160

140

120
Number of cabs

100

80

60

40

20

10 20 30 40 50 60 70 80
Fares ($)
(a) Use the curve to estimate
(i) the median fare;
(ii) the number of cabs in which the fare taken is $35 or less.
(2)

The company charges 55 cents per kilometre for distance travelled. There are no other charges.
Use the curve to answer the following.
(b) On that morning, 40% of the cabs travel less than a km. Find the value of a.
(4)

(c) What percentage of the cabs travel more than 90 km on that morning?
(4)
(Total 10 marks)

310. Three positive integers a, b, and c, where a < b < c, are such that their median is 11, their mean
is 9 and their range is 10. Find the value of a.
Working:
Answer:
......................................................................
(Total 6 marks)
311. In a suburb of a large city, 100 houses were sold in a three-month period. The following
cumulative frequency table shows the distribution of selling prices (in thousands of dollars).
Selling price P P  100 P  200 P  300 P  400 P  500
($1000)
Total number 12 58 87 94 100
of houses
(a) Represent this information on a cumulative frequency curve, using a scale of 1 cm to
represent $50000 on the horizontal axis and 1 cm to represent 5 houses on the vertical
axis.
(4)
(b) Use your curve to find the interquartile range.
(3)
The information above is represented in the following frequency distribution.
Selling price P 0 < P  100 100 < P  200 200 < P  300 300 < P  400 400 < P  500
($1000)
Number of 12 46 29 a b
houses
(c) Find the value of a and of b.
(2)
(d) Use mid-interval values to calculate an estimate for the mean selling price.
(2)
(e) Houses which sell for more than $350000 are described as De Luxe.
(i) Use your graph to estimate the number of De Luxe houses sold.
Give your answer to the nearest integer.
(ii) Two De Luxe houses are selected at random. Find the probability
that both have a selling price of more than $400000.
(4)
(Total 15 marks)
312. Given the following frequency distribution, find
(a) the median;
(b) the mean.
Number (x) 1 2 3 4 5 6
Frequency (f ) 5 9 16 18 20 7

Working:
Answers:
(a) ..................................................................
(b) ..................................................................
(Total 4 marks)

313. The table below represents the weights, W, in grams, of 80 packets of roasted
peanuts.
Weight (W) 80 < W  85 85 < W  90 90 < W  95 95 < W  100 100 < W  105 105 < W  110 110 < W  115
Number of
5 10 15 26 13 7 4
packets

(a) Use the midpoint of each interval to find an estimate for the standard deviation of the
weights.
(3)
(b) Copy and complete the following cumulative frequency table for the above data.
Weight (W) W  85 W  90 W  95 W  100 W  105 W 110 W  115
Number of
5 15 80
packets
(1)

(c) A cumulative frequency graph of the distribution is shown below, with a scale 2 cm for
10 packets on the vertical axis and 2 cm for 5 grams on the horizontal axis.

80

70

60

50
Number
of
packets
40

30

20

10

80 85 90 95 100 105 110 115


Weight (grams)
Use the graph to estimate
(i) the median;
(ii) the upper quartile (that is, the third quartile).
Give your answers to the nearest gram.
(4)

(d) Let W1, W2, ..., W80 be the individual weights of the packets, and let W be their mean.
What is the value of the sum
(W1 – W )  (W2 – W )  (W3 – W )  ...  (W79 – W )  (W80 – W ) ?
(2)

(e) One of the 80 packets is selected at random. Given that its weight satisfies
85 < W  110, find the probability that its weight is greater than 100 grams.
(4)
(Total 14 marks)

314. The speeds in km h–1 of cars passing a point on a highway are recorded in the following table.
Speed v Number of cars
v  60 0
60 < v  70 7
70 < v  80 25
80 < v  90 63
90 < v  100 70
100 < v  110 71
110 < v  120 39
120 < v  130 20
130 < v  140 5
v > 140 0
(a) Calculate an estimate of the mean speed of the cars.
(2)

(b) The following table gives some of the cumulative frequencies for the information above.
Speed v Cumulative frequency
v  60 0
v  70 7
v  80 32
v  90 95
v  100 a
v  110 236
v  120 b
v  130 295
v  140 300

(i) Write down the values of a and b.


(ii) On graph paper, construct a cumulative frequency curve to represent this
information. Use a scale of 1 cm for 10 km h–1 on the horizontal axis and a scale of
1 cm for 20 cars on the vertical axis.
(5)

(c) Use your graph to determine


(i) the percentage of cars travelling at a speed in excess of 105 km h–1;
(ii) the speed which is exceeded by 15% of the cars.
(4)
(Total 11 marks)
315. The following diagram represents the lengths, in cm, of 80 plants grown in a laboratory.
20

15
frequency
10

0
0 10 20 30 40 50 60 70 80 90 100
length (cm)

(a) How many plants have lengths in cm between


(i) 50 and 60?
(ii) 70 and 90?
(2)

(b) Calculate estimates for the mean and the standard deviation of the lengths of the plants.
(4)

(c) Explain what feature of the diagram suggests that the median is different from the mean.
(1)
(d) The following is an extract from the cumulative frequency table.
length in cm cumulative
less than frequency
. .
50 22
60 32
70 48
80 62
. .
Use the information in the table to estimate the median. Give your answer to two
significant figures.
(3)
(Total 10 marks)

316. A supermarket records the amount of money d spent by customers in their store during a busy
period. The results are as follows:
Money in $ (d) 0–20 20–40 40–60 60–80 80–100 100–120 120–140
Number of customers (n) 24 16 22 40 18 10 4

(a) Find an estimate for the mean amount of money spent by the customers, giving your
answer to the nearest dollar ($).
(2)

(b) Copy and complete the following cumulative frequency table and use it to draw a
cumulative frequency graph. Use a scale of 2 cm to represent $20 on the horizontal axis,
and 2 cm to represent 20 customers on the vertical axis.
(5)
Money in $ (d) <20 <40 <60 <80 < 100 < 120 < 140
Number of customers (n) 24 40
(c) The time t (minutes), spent by customers in the store may be represented by the equation
2
t = 2d 3 + 3.
(i) Use this equation and your answer to part (a) to estimate the mean time in minutes
spent by customers in the store.
(3)
(ii) Use the equation and the cumulative frequency graph to estimate the number of
customers who spent more than 37 minutes in the store.
(5)
(Total 15 marks)

317. The table shows the scores of competitors in a competition.


Score 10 20 30 40 50
Number of competitors 1 2 5 k 3
with this score

The mean score is 34. Find the value of k.


Working:
Answer:
......................................................................
(Total 4 marks)
318. A survey is carried out to find the waiting times for 100 customers at a supermarket.
waiting time number of
(seconds) customers
0–30 5
30– 60 15
60– 90 33
90 –120 21
120–150 11
150–180 7
180–210 5
210–240 3
(a) Calculate an estimate for the mean of the waiting times, by using an appropriate
approximation to represent each interval.
(2)
(b) Construct a cumulative frequency table for these data.
(1)
(c) Use the cumulative frequency table to draw, on graph paper, a cumulative frequency
graph, using a scale of 1 cm per 20 seconds waiting time for the horizontal axis and 1 cm
per 10 customers for the vertical axis.
(4)
(d) Use the cumulative frequency graph to find estimates for the median and the lower
and upper quartiles.
(3)
(Total 10 marks)

319. One thousand candidates sit an examination. The distribution of marks is shown in the
following grouped frequency table.
Marks 1–10 11–20 21–30 31–40 41–50 51–60 61–70 71–80 81–90 91–100
Number of 15 50 100 170 260 220 90 45 30 20
candidates
(a) Copy and complete the following table, which presents the above data as a cumulative
frequency distribution.
(3)
Mark 10 20 30 40 50 60 70 80 90 100
Number of 15 65 905
candidates
(b) Draw a cumulative frequency graph of the distribution, using a scale of 1 cm for 100
candidates on the vertical axis and 1 cm for 10 marks on the horizontal axis.
(5)

(c) Use your graph to answer parts (i)–(iii) below,


(i) Find an estimate for the median score.
(2)
(ii) Candidates who scored less than 35 were required to retake the examination.
How many candidates had to retake?
(3)
(iii) The highest-scoring 15% of candidates were awarded a distinction.
Find the mark above which a distinction was awarded.
(3)
(Total 16 marks)

320. At a conference of 100 mathematicians there are 72 men and 28 women. The men have a mean
height of 1.79 m and the women have a mean height of 1.62 m. Find the mean height of the 100
mathematicians.
Working:
Answer:
......................................................................
(Total 4 marks)

25
321. The mean of the population x1, x2, ........ , x25 is m. Given that x
i 1
i = 300 and
25

 (x
i 1
i – m) 2 = 625, find

(a) the value of m;


(b) the standard deviation of the population.
Working:
Answers:
(a) ..................................................................
(b) ..................................................................
(Total 4 marks)

MARKSCHEME
1. (a) attempt to form composite (M1)
e.g. g 7  2 x , 7  2 x  3
(g ° f)(x) = 10 – 2x A1 N2 2

(b) g–1(x) = x – 3 A1 N1 1

(c) METHOD 1
valid approach (M1)
e.g. g–1(5), 2, f (5)
f (2) = 3 A1 N2 2

METHOD 2
attempt to form composite of f and g–1 (M1)
e.g. (f ° g–1)(x) = 7 – 2(x – 3), 13 – 2x
(f ° g–1)(5) = 3 A1 N2 2
[5]

2. (a) valid approach (M1)


e.g. b 2  4ac,   0,  4k   42k 1
2

correct equation A1
e.g. (–4k)2 – 4(2k)(1) = 0, 16k2 = 8k, 2k2 – k = 0
correct manipulation A1
8  64
e.g. 8k 2k  1,
32
1
k A2 N3 5
2

(b) recognizing vertex is on the x-axis M1


e.g. (1, 0), sketch of parabola opening upward from the x-axis
P0 A1 N1 2
[7]

3. (a) v=1 A1 N1 1

(b) (i)
d
2t   2 A1
dt
d
cos 2t   2 sin 2t A1A1
dt
Note: Award A1 for coefficient 2 and A1 for –sin 2t.
evidence of considering acceleration = 0 (M1)
dv
e.g.  0, 2  2 sin 2t  0
dt
correct manipulation A1
e.g. sin 2k  1, sin 2t  1
π π
2k   accept 2t   A1
2 2
π
k AG N0
4


(ii) attempt to substitute t  into v (M1)
4
  2 
e.g. 2   cos 
4  4 

v A1 N2 8
2
(c)

A1A1A2 N4 4
Notes: Award A1 for y-intercept at (0, 1), A1 for curve having
π
zero gradient at t  , A2 for shape that is concave down to
4
π π
the left of and concave up to the right of . If a correct
4 4
π
curve is drawn without indicating t = , do not award the
4
second A1 for the zero gradient, but award the final A2 if
appropriate. Sketch need not be drawn to scale. Only essential
features need to be clear.

(d) (i) correct expression A2


1
 sin 2t 
 2t  cos 2t  dt, t
1 sin 2 1
 , 1 
2
e.g.  , vdt
0 2 0 2 0

(ii)

A1 3
π
Note: The line at t = 1 needs to be clearly after t  .
4
[16]
4. (a) attempt to apply rules of logarithms (M1)
e.g. ln ab = bln a, ln ab = ln a + ln b
correct application of ln ab = bln a (seen anywhere) A1
e.g. 3ln x = ln x3
correct application of ln ab = ln a + ln b (seen anywhere) A1
e.g. ln 5x3 = ln 5 + ln x3
so ln 5x3 = ln 5 + 3ln x
g (x) = f (x) + ln5 (accept g (x) = 3ln x + ln 5) A1 N1 4

(b) transformation with correct name, direction, and value A3


0 
e.g. translation by   , shift up by ln 5, vertical translation of ln 5 3
 ln 5 
[7]

5. (a) f (x) = –10(x + 4)(x – 6) A1A1 N2 2

(b) METHOD 1
attempting to find the x-coordinate of maximum point (M1)
e.g. averaging the x-intercepts, sketch, y′ = 0, axis of symmetry
attempting to find the y-coordinate of maximum point (M1)
e.g. k = –10(1+ 4)(1– 6)
f (x) = –10(x –1)2 + 250 A1A1 N4 4
METHOD 2
attempt to expand f (x) (M1)
e.g. –10(x2 – 2x – 24)
attempt to complete the square (M1)
e.g. –10((x –1)2 –1– 24)
f (x) = –10(x –1)2 + 250 A1A1 N4 4

(c) attempt to simplify (M1)


e.g. distributive property, –10(x –1)(x –1) + 250
correct simplification A1
e.g. –10(x2 – 6x + 4x – 24), –10(x2 – 2x +1) + 250
f (x) = 240 + 20x –10x2 AG N0 2

(d) (i) valid approach (M1)


e.g. vertex of parabola, v′(t) = 0
t =1 A1 N2

(ii) recognizing a(t) = v′(t) (M1)


a(t) = 20 – 20t A1A1
speed is zero  t = 6 (A1)
a(6) = –100 (m s–2) A1 N3 7
[15]

6. (a) (1, – 2) A1A1 N2 2

(b) g (x) = 3(x – 1)2 – 2 (accept p =1, q = –2) A1A1 N2 2

(c) (1, 2) A1A1 N2 2


[6]

7. (a) evidence of valid approach involving A and B (M1)


e.g. P(A ∩ pass) + P(B ∩ pass), tree diagram
correct expression (A1)
e.g. P(pass) = 0.6 × 0.8 + 0.4 × 0.9
P(pass) = 0.84 A1 N2 3

(b) evidence of recognizing complement (seen anywhere) (M1)


e.g. P(B) = x, P(A) = 1 – x, 1 – P(B), 100 – x, x + y =1
evidence of valid approach (M1)
e.g. 0.8(1 – x) + 0.9x, 0.8x + 0.9y
correct expression A1
e.g. 0.87 = 0.8(1 – x) + 0.9x, 0.8 × 0.3 + 0.9 × 0.7 = 0.87, 0.8x + 0.9y = 0.87
70 % from B A1 N2 4
[7]

8. (a) B, D A1A1 N2 2
x 2
(b) (i) f′(x) =  2 xe A1A1 N2
 x2
Note: Award A1 for e and A1 for –2x.

(ii) finding the derivative of –2x, i.e. –2 (A1)


evidence of choosing the product rule (M1)
x 2
x 2
e.g.  2e  2 x  2xe
 2e  x  4x 2 e  x
2 2
A1
2  x2
f ′′(x) = (4x – 2) e AG N0 5

(c) valid reasoning R1


e.g. f ′′(x) = 0
attempting to solve the equation (M1)
e.g. (4x2 – 2) = 0, sketch of f ′′(x)
 1   1 
p = 0.707    , q  0.707     A1A1 N3 4
 2  2

(d) evidence of using second derivative to test values on either side of POI M1
e.g. finding values, reference to graph of f′′, sign table
correct working A1A1
e.g. finding any two correct values either side of POI,
checking sign of f ′′ on either side of POI
reference to sign change of f ′′(x) R1 N0 4
[15]

9. (a) combining 2 terms (A1)


1
e.g. log3 8x – log3 4, log3 x + log3 4
2
expression which clearly leads to answer given A1
8x 4x
e.g. log 3 , log 3
3 2
f(x) = log3 2x AG N0 2
(b) attempt to substitute either value into f (M1)
e.g. log3 1, log3 9
f(0.5) = 0, f(4.5) = 2 A1A1 N3 3

(c) (i) a = 2, b = 3 A1A1 N1N1

(ii)

A1A1A1 N3
Note: Award A1 for sketch approximately through
(0.5 ± 0.1, 0 ± 0.1)
A1 for approximately correct shape,
A1 for sketch asymptotic to the y-axis.

(iii) x = 0 (must be an equation) A1 N1


[6]

(d) f–1(0) = 0.5 A1 N1 1


(e)

A1A1A1A1 N4 4
Note: Award A1 for sketch approximately through (0 ± 0.1,
0.5 ± 0.1),
A1 for approximately correct shape of the graph
reflected over y = x,
A1 for sketch asymptotic to x-axis,
A1 for point (2 ± 0.1, 4.5 ± 0.1) clearly marked and
on curve.
[16]

10. (a) attempt to form composite (M1)


e.g. f(2x – 5)
h(x) = 6x – 15 A1 N2 2
(b) interchanging x and y (M1)
evidence of correct manipulation (A1)
x 5
e.g. y  15  6 x,  y 
6 2

h 1 x  
x 15
A1 N3 3
6
[5]

11. (a)

A1A1A1A1 N4 4
Note: Award A1 for approximately correct shape, A1 for left
end point in circle, A1 for local maximum in circle, A1 for right
end point in circle.

(b) attempting to solve g (x) = –1 (M1)


1
e.g. marking coordinate on graph, x sin x + 1 = 0
2
x = 3.71 A1 N2 2
[6]

12. (a) evidence of setting function to zero (M1)


e.g. f(x) = 0, 8x = 2x2
evidence of correct working A1
 8  64
e.g. 0 = 2x(4 – x),
4
x-intercepts are at 4 and 0 (accept (4, 0) and (0, 0), or x = 4, x = 0) A1A1 N1N1

(b) (i) x = 2 (must be equation) A1 N1


(ii) substituting x = 2 into f(x) (M1)
y=8 A1 N2
[7]

13. (a) interchanging x and y (seen anywhere) (M1)


e.g. x = log y (accept any base)
evidence of correct manipulation A1
1
1
e.g. 3x = y ,3y  x2,x  log3 y, 2y = log3 x
2
f–1(x) = 32x AG N0

(b) y > 0, f–1(x) > 0 A1 N1

(c) METHOD 1
finding g(2) = log3 2 (seen anywhere) A1
attempt to substitute (M1)
e.g. (f–1 ° g)(2) = 3 log3 2
evidence of using log or index rule (A1)
–1 log3 4 log3 2 2
e.g. (f ° g)(2) = 3 ,3
(f–1 ° g)(2) = 4 A1 N1

METHOD 2
attempt to form composite (in any order) (M1)
e.g. (f–1 ° g)(x) = 3 2 log3 x
evidence of using log or index rule (A1)
–1 log3 x 2 log3 x 2
e.g.(f ° g)(x) = 3 ,3
(f–1 ° g)(x) = x2 A1
–1
(f ° g)(2) = 4 A1 N1
[7]
14. (a) f′(x) = x2 – 2x – 3 A1A1A1
evidence of solving f′(x) = 0 (M1)
e.g. x2 – 2x – 3 = 0
evidence of correct working A1
2  16
e.g. (x + 1)(x – 3),
2
x = –1 (ignore x = 3) (A1)
evidence of substituting their negative x-value into f(x) (M1)
1 1
e.g. (1)  (1)  3(1),   1  3
3 2
3 3
5
y= A1
3
 5
coordinates are   1,  N3
 3

(b) (i) (–3, –9) A1 N1

(ii) (1, –4) A1A1 N2

(iii) reflection gives (3, 9) (A1)


3 
stretch gives  , 9  A1A1 N3
2 
[14]

15. (a) q = –2, r = 4 or q = 4, r = –2 A1A1 N2

(b) x = 1 (must be an equation) A1 N1

(c) substituting (0, –4) into the equation (M1)


e.g. –4 = p(0 – (–2))(0 – 4), –4 = p(–4)(2)
correct working towards solution (A1)
e.g. –4 = –8p
4 1
p =   A1 N2
8  2
[6]
π
16. (a) f    cosπ (A1)
2
= –1 A1 N2

π
(b) (g ° f)   = g(–1) (= 2(–1)2 – 1) (A1)
2
=1 A1 N2

(c) (g ° f)(x) = 2(cos (2x))2 – 1 (= 2 cos2(2x) – 1) A1


2
evidence of 2 cos θ – 1 = cos 2θ (seen anywhere) (M1)
(g ° f)(x) = cos 4x
k=4 A1 N2
[7]

17. recognizing log a + log b = log ab (seen anywhere) (A1)


e.g. log2(x(x – 2)), x2 – 2x
recognizing loga b = x  ax = b (seen anywhere) (A1)
e.g. 23 = 8
correct simplification A1
e.g. x(x – 2) = 23, x2 – 2x – 8
evidence of correct approach to solve (M1)
e.g. factorizing, quadratic formula
correct working A1
2  36
e.g. (x – 4)(x + 2),
2
x=4 A2 N3
[7]

18. (a) (i) sin x = 0 A1


x = 0, x = π A1A1 N2

(ii) sin x = –1 A1

x= A1 N1
2

(b) A1 N1
2

(c) evidence of using anti-differentiation (M1)



e.g. 
0
2 (6  6 sin x)dx
correct integral 6x – 6 cos x (seen anywhere) A1A1
correct substitution (A1)
 3π   3π 
e.g. 6   6cos   (6 cos 0), 9π – 0 + 6
 2  2
k = 9π + 6 A1A1 N3

π
(d)
 
translation of  2  A1A1 N2
0

(e) recognizing that the area under g is the same as the shaded region in f (M1)
π
p= ,p=0 A1A1 N3
2
[17]

19. (a) correct substitution A1


e.g. 25 + 16 – 40cos x, 52 + 42 – 2 × 4 × 5 cosx
AC = 41 40 cos x AG

(b) correct substitution A1


AC 4 1
e.g.  , AC = 4 sin x
sin x sin 30 2
 4 sin x 
AC = 8 sin x  accept  A1 N1
 sin 30 

(c) (i) evidence of appropriate approach using AC M1


e.g. 8 sin x = 41 40 cos x , sketch showing intersection
correct solution 8.682..., 111.317... (A1)
obtuse value 111.317... (A1)
x = 111.32 to 2 dp (do not accept the radian answer 1.94) A1 N2
(ii) substituting value of x into either expression for AC (M1)
e.g. AC = 8 sin 111.32
AC = 7.45 A1 N2

(d) (i) evidence of choosing cosine rule (M1)


a2  c2  b2
e.g. cos B =
2ac
correct substitution A1
4 2  4 2  7.45 2
e.g. , 7.452 = 32 – 32 cos y, cos y = –0.734...
2 4 4
y = 137 A1 N2

(ii) correct substitution into area formula (A1)


1
e.g. × 4 × 4 × sin 137, 8 sin 137
2
area = 5.42 A1 N2
[14]

20. (a) substituting (0, 13) into function M1


e.g. 13 = Ae0 + 3
13 = A + 3 A1
A = 10 AG N0

(b) substituting into f(15) = 3.49 A1


e.g. 3.49 = 10e15k + 3, 0.049 = e15k
evidence of solving equation (M1)
e.g. sketch, using ln
 ln0.049 
k = –0.201  accept  A1 N2
 15 

(c) (i) f(x) = 10e–0.201x + 3


f′(x) = 10e–0.201x × –0.201 (= –2.01e–0.201x) A1A1A1 N3
Note: Award A1 for 10e–0.201x, A1 for × –0.201,
A1 for the derivative of 3 is zero.

(ii) valid reason with reference to derivative R1 N1


e.g. f′(x) < 0, derivative always negative

(iii) y=3 A1 N1

(d) finding limits 3.8953…, 8.6940… (seen anywhere) A1A1


evidence of integrating and subtracting functions (M1)
correct expression A1
8.69 8.69
e.g. 
3.90
g ( x)  f ( x)dx, 
3.90
[( x 2  12 x  24)  (10e 0.201x  3)]dx
area = 19.5 A2 N4
[16]

21. (a)
A1A1A1 N3

(b) x = –1.32, x = 1.68 (accept x = –1.41, x = 1.39 if working in degrees) A1A1 N2

(c) –1.32 < x < 1.68 (accept –1.41 < x < 1.39 if working in degrees) A2 N2
[7]

22. (a) 2.31 A1 N1

(b) (i) 1.02 A1 N1

(ii) 2.59 A1 N1

q
(c) p
f ( x)dx = 9.96 A1 N1
split into two regions, make the area below the x-axis positive R1R1 N2
[6]
23. (a) n = 800e0 (A1)
n = 800 A1 N2

(b) evidence of using the derivative (M1)


n′(15) = 731 A1 N2

(c) METHOD 1
setting up inequality (accept equation or reverse inequality) A1
e.g. n′(t) > 10 000
evidence of appropriate approach M1
e.g. sketch, finding derivative
k = 35.1226... (A1)
least value of k is 36 A1 N2

METHOD 2
n′(35) = 9842, and n′(36) = 11208 A2
least value of k is 36 A2 N2
[8]

24. (a) (i) –1.15, 1.15 A1A1 N2

(ii) recognizing that it occurs at P and Q (M1)


e.g. x = –1.15, x = 1.15
k = –1.13, k = 1.13 A1A1 N3

(b) evidence of choosing the product rule (M1)


e.g. uv′ + vu′
derivative of x3 is 3x2 (A1)
 2x
derivative of ln (4 – x2) is (A1)
4  x2
correct substitution A1
 2x
e.g. x 3   ln( 4  x 2 )  3x 2
4 x 2

 2x 4
g′(x) =  3x 2 ln( 4  x 2 ) AG N0
4 x 2
(c)

A1A1 N2

(d) w = 2.69, w < 0 A1A2 N2


[14]

25. (a) attempt to form composition (in any order) (M1)


(f ° g)(x) = (x –1)2 + 4 (x2 – 2x + 5) A1 N2

(b) METHOD 1
vertex of f ° g at (1, 4) (A1)
evidence of appropriate approach (M1)
 3
e.g. adding   to the coordinates of the vertex of f ° g
 1
vertex of h at (4, 3) A1 N3

METHOD 2
attempt to find h(x) (M1)
e.g. ((x – 3) – 1)2 + 4 – 1, h(x) = (f ° g)(x – 3) – 1
h(x) = (x – 4)2 + 3 (A1)
vertex of h at (4, 3) A1 N3

(c) evidence of appropriate approach (M1)


e.g. (x – 4)2 + 3, (x – 3)2 – 2(x – 3) + 5 – 1
simplifying A1
e.g. h(x) = x2 – 8x + 16 + 3, x2 – 6x + 9 – 2x + 6 + 4
h(x) = x2 – 8x + 19 AG N0
(d) METHOD 1
equating functions to find intersection point (M1)
e.g. x2 – 8x + 19 = 2x – 6, y = h(x)
x2 – 10x + 25 = 0 A1
evidence of appropriate approach to solve (M1)
e.g. factorizing, quadratic formula
appropriate working A1
e.g. (x – 5)2 = 0
x = 5 (p = 5) A1 N3

METHOD 2
attempt to find h′(x) (M1)
h′(x) = 2x – 8 A1
recognizing that the gradient of the tangent is the derivative (M1)
e.g. gradient at p = 2
2x – 8 = 2 (2x = 10) A1
x=5 A1 N3
[12]

26. (a) attempt to substitute points into the function (M1)


e.g. –8 = p(–2)3 + q(–2)2 + r(–2), one correct equation
–8 = –8p + 4q – 2r, –2 = p + q + r, 0 = 8p + 4q + 2r A1A1A1 N4

(b) attempt to solve system (M1)


e.g. inverse of a matrix, substitution
p = 1, q = –1, r = –2 A2 N3
Notes: Award A1 for two correct values.
If no working shown, award N0 for two correct values.
[7]

27. (a) evidence of valid approach (M1)


e.g. f(x) = 0, graph
a = –1.73, b = 1.73 (a   3 , b  3 ) A1A1 N3

(b) attempt to find max (M1)


e.g. setting f′(x) = 0, graph
c = 1.15 (accept (1.15, 1.13)) A1 N2

(c) attempt to substitute either limits or the function into formula M1

  f ( x) dx, π x ln( 4  x ) , π


c 2 2 1.149...
e.g. V = π 2
y 2 dx
0 0
V = 2.16 A2 N2

(d) valid approach recognizing 2 regions (M1)


e.g. finding 2 areas
correct working (A1)
1.73... 1.149... 0 1.149...
e.g. 
0
f ( x)dx  
0
f ( x)dx;  
1.73...
f ( x)dx  0
f ( x)dx
area = 2.07 (accept 2.06) A2 N3
[12]

28. (a) in any order


translated 1 unit to the right A1 N1
stretched vertically by factor 2 A1 N1

(b) METHOD 1
Finding coordinates of image on g (A1)(A1)
e.g. –1 + 1 = 0, 1 × 2 = 2, (–1, 1) → (–1 + 1, 2 × 1), (0, 2)
P is (3, 0) A1A1 N4
METHOD 2
h(x) = 2(x – 4)2 – 2 (A1)(A1)
P is (3, 0) A1A1 N4
[6]

29. (a) (i) interchanging x and y (seen anywhere) M1


e.g. x = ey+3
correct manipulation A1
e.g. ln x = y + 3, ln y = x + 3
f–1(x) = ln x – 3 AG N0

(ii) x>0 A1 N1
(b) collecting like terms; using laws of logs (A1)(A1)
 
 
1 x
e.g. ln x – ln    3, ln x  ln x  3; ln    3, ln x 2  3
 x 1
 
x
simplify (A1)
3
e.g. ln x = , x2 = e3
2
3
x = e 2 ( e 3 ) A1 N2
[7]

30. (a) METHOD 1


evidence of substituting –x for x (M1)
a( x)
f(–x) = A1
( x) 2  1
 ax
f(–x) = (= –f(x)) AG N0
x 2 1
METHOD 2
y = –f(x) is reflection of y = f(x) in x axis
and y = f(–x) is reflection of y = f(x) in y axis (M1)
sketch showing these are the same A1
 ax
f(–x) = 2 (= –f(x)) AG N0
x 1

(b) evidence of appropriate approach (M1)


e.g. f″(x) = 0
to set the numerator equal to 0 (A1)
e.g. 2ax(x2 – 3) = 0; (x2 – 3) = 0
 a 3   a 3 
(0, 0),  3 , 
,  3 ,
 (accept x = 0, y = 0 etc.) A1A1A1A1A1 N5
 4  4 

(c) (i) correct expression A2


7
a  a a a
e.g.  ln( x 2  1) , ln 50  ln 10, (ln 50  ln 10)
2 3 2 2 2
a
area = ln 5 A1A1 N2
2
(ii) METHOD 1
recognizing that the shift does not change the area (M1)
8 7 a
4  3 
e.g. f ( x  1)dx  f ( x)dx, ln 5
2
recognizing that the factor of 2 doubles the area (M1)
8 8
  2 7f ( x)dx 
4  4 
e.g. 2 f ( x  1)dx 2 f ( x  1)dx 
 3  

8
 2 f ( x 1)dx  a ln 5
4
(i.e. 2 × their answer to (c)(i)) A1 N3

METHOD 2
changing variable
dw
let w = x – 1, so =1
dx
2a

2 f ( w)dw 
2
ln( w 2  1)  c (M1)
substituting correct limits
  
8
7
e.g. a ln[( x  1) 2  1] 4 , a ln( w 2  1) 3 , a ln 50  a ln 10 (M1)
8
 2 f ( x 1)dx  a ln 5
4
A1 N3
[16]

31. (a) for interchanging x and y (may be done later) (M1)


e.g. x = 2y – 3
x3  x 3 x 3
g–1(x) =  accept y  ,  A1 N2
2  2 2 

(b) METHOD 1
g(4) = 5 (A1)
evidence of composition of functions (M1)
f(5) = 25 A1 N3
METHOD 2
f ° g(x) = (2x – 3)2 (M1)
2
f ° g(4) = (2 × 4 – 3) (A1)
= 25 A1 N3
[5]
32. e2x( 3 sin x + cos x) = 0 (A1)
e2x = 0 not possible (seen anywhere) (A1)
simplifying
sin x 1
e.g. 3 sin x  cos x  0, 3 sin x   cos x,  A1
 cos x 3
EITHER
1
tan x =  A1
3

x= A2 N4
6
OR
sketch of 30°, 60°, 90° triangle with sides 1, 2, 3 A1

work leading to x = A1
6

verifying satisfies equation A1 N4
6
[6]

33. (a) attempt to form any composition (even if order is reversed) (M1)
 3x 
correct composition h(x) = g  1 (A1)
 2 
 3x 
 1
h( x)  4 cos 2   1  4 cos 1 x  1   1, 4 cos 3x  2   1 A1 N3
 3   
 2 3  6  
 
 

(b) period is 4π(12.6) A1 N1

(c) range is –5 ≤ h(x) ≤ 3 ([–5, 3]) A1A1 N2


[6]

34. (a) evidence of substituting (–4, 3) (M1)


correct substitution 3 = a(–4)2 + b(–4) + c A1
16a – 4b + c = 3 AG N0

(b) 3 = 36a + 6b + c, –1 = 4a – 2b + c A1A1 N1N1

 16  4 1 3
   
(c) (i) A =  36 6 1; B   3  A1A1 N1N1
 4  2 1   1
   

  1 1 1 
   
 0.05 0.0125  0.0625    20 80 16  
–1     1 3 1 
(ii) A =   0.2 0.075 0.125    5 A2 N2
  0.6 8 
1.5 
40
 0.1   3 1 3 
   
  5 10 2  

(iii) evidence of appropriate method (M1)


e.g. X = A–1B, attempting to solve a system of three equations
 0.25 
 
X =   0.5  (accept fractions) A2
 3 
 
f(x) = 0.25x2 – 0.5x – 3 (accept a = 0.25, b = –0.5, c = –3, or fractions) A1 N2

(d) f(x) = 0.25(x – 1)2 – 3.25 (accept h = 1, k = –3.25, a = 0.25, or fractions) A1A1A1 N3
[15]

35. (a)

A2 N2

(b)
Description of transformation Diagram letter
Horizontal stretch with scale factor 1.5 C
Maps f to f(x) + 1 D
A1A1 N2

(c) translation (accept move/shift/slide etc.) with vector A1A1 N2


[6]

36. evidence of appropriate approach M1


e.g. a sketch, writing ex – 4 sin x = 0
x = 0.371, x = 1.36 A2A2 N2N2
[5]

37. (a) attempt to use discriminant (M1)


correct substitution, (k – 3)2 – 4 × k × 1 (A1)
setting their discriminant equal to zero M1
e.g. (k – 3)2 – 4 × k × 1 = 0, k2 – 10k + 9 = 0
k = 1, k = 9 A1A1 N3

(b) k = 1, k = 9 A2 N2
[7]

38. (a) (i) g(0) = e0 – 2 (A1)


= –1 A1 N2

(ii) METHOD 1
substituting answer from (i) (M1)
e.g. (f ° g)(0) = f(–1)
correct substitution f(–1) = 2(–1)3 + 3 (A1)
f(–1) = 1 A1 N3

METHOD 2
attempt to find (f ° g)(x) (M1)
e.g. (f ° g)(x) = f(e3x – 2) = 2(e3x – 2)3 + 3
correct expression for (f ° g)(x) (A1)
3x 3
e.g. 2(e – 2) + 3
(f ° g)(0) = 1 A1 N3
(b) interchanging x and y (seen anywhere) (M1)
e.g. x = 2y3 + 3
attempt to solve (M1)
x3
e.g. y3 =
2
x3
f–1(x) = 3 A1 N3
2
[8]

39. (a)

A2 N2

(b) evidence of appropriate approach (M1)


1
e.g. reference to any horizontal shift and/or stretch factor, x = 3 + 1, y =  2
2
P is (4, 1) (accept x = 4, y = 1) A1A1 N3
[5]

40. (a) METHOD 1


recognizing that f(8) = 1 (M1)
e.g. 1 = k log2 8
recognizing that log2 8 = 3 (A1)
e.g. 1 = 3k
1
k= A1 N2
3
METHOD 2
attempt to find the inverse of f(x) = k log2 x (M1)
x
e.g. x = k log2 y, y = 2 k

substituting 1 and 8 (M1)


1
e.g. 1 = k log2 8, 2k =8
1  1
k= k   A1 N2
log 2 8  3

(b) METHOD 1
2
recognizing that f(x) = (M1)
3
2 1
e.g.  log 2 x
3 3
log2 x = 2 (A1)
2
f–1   = 4 (accept x = 4) A2 N3
3

METHOD 2
1
attempt to find inverse of f(x) = log2 x (M1)
3
x
1
e.g. interchanging x and y , substituting k = into y = 2 k
3
correct inverse (A1)
e.g. f–1(x) = 23x, 23x
2
f–1   = 4 A2 N3
3
[7]

41. (a) (i) coordinates of A are (0, –2) A1A1 N2


(ii) derivative of x2 – 4 = 2x (seen anywhere) (A1)
evidence of correct approach (M1)
e.g. quotient rule, chain rule
finding f′(x) A2
( x 2  4)(0)  (20)(2 x)
e.g. f′(x) = 20 × (–1) × (x2 – 4)–2 × (2x),
( x 2  4) 2
substituting x = 0 into f′(x) (do not accept solving f′(x) = 0) M1
at A f′(x) = 0 AG N0

(b) (i) reference to f′(x) = 0 (seen anywhere) (R1)


reference to f″(0) is negative (seen anywhere) R1
evidence of substituting x = 0 into f″(x) M1
40  4   5 
finding f″(0) =   A1
(4) 3  2 
then the graph must have a local maximum AG

(ii) reference to f″(x) = 0 at point of inflexion, (R1)


recognizing that the second derivative is never 0 A1 N2
4
e.g. 40(3x2 + 4) ≠ 0, 3x2 + 4 ≠ 0, x2 ≠  , the numerator is
3
always positive
Note: Do not accept the use of the first derivative in part (b).

(c) correct (informal) statement, including reference to approaching y = 3 A1 N1


e.g. getting closer to the line y = 3, horizontal asymptote at y = 3

(d) correct inequalities, y ≤ –2, y > 3, FT from (a)(i) and (c) A1A1 N2
[16]
42. (a)

A1A1A1 N3
Note: Award A1 for f being of sinusoidal shape, with
2 maxima and one minimum,
A1 for g being a parabola opening down,
A1 for two intersection points in approximately
correct position.

(b) (i) (2,0) (accept x = 2) A1 N1

(ii) period = 8 A2 N2

(iii) amplitude = 5 A1 N1

(c) (i) (2, 0), (8, 0) (accept x = 2, x = 8) A1A1 N1N1

(ii) x = 5 (must be an equation) A1 N1

(d) METHOD 1
intersect when x = 2 and x = 6.79 (may be seen as limits of integration) A1A1
evidence of approach (M1)
6.79   π 
  
e.g. g  f , f ( x)dx  g ( x)dx,
2


 (0.5 x 2  5 x  8   5 cos x  
 4 
area = 27.6 A2 N3
METHOD 2
intersect when x = 2 and x = 6.79 (seen anywhere) A1A1
evidence of approach using a sketch of g and f, or g – f. (M1)

e.g. area A + B – C, 12.7324 + 16.0938 – 1.18129...


area = 27.6 A2 N3
[15]

43. (a) METHOD 1


ln (x + 5) + ln 2 = ln (2(x + 5)) (= ln (2x + 10)) (A1)
interchanging x and y (seen anywhere) (M1)
e.g. x = ln (2y + 10)
evidence of correct manipulation (A1)
e.g. ex = 2y + 10
e x  10
f 1  x   A1 N2
2
METHOD 2
y = ln (x + 5) + ln 2
y  ln 2 = ln (x + 5) (A1)
evidence of correct manipulation (A1)
e.g. ey  ln 2 = x + 5
interchanging x and y (seen anywhere) (M1)
e.g. ex  ln 2 = y + 5
f 1 (x) = ex  ln 2  5 A1 N2
(b) METHOD 1
evidence of composition in correct order (M1)
e.g. (g ◦ f) (x) = g (ln (x + 5) + ln 2)
= eln (2(x + 5)) = 2(x + 5)
(g ◦ f) (x) = 2x + 10 A1A1 N2
METHOD 2
evidence of composition in correct order (M1)
e.g. (g ◦ f) (x) = eln(x + 5) + ln 2
= eln (x + 5)  eln 2 = (x + 5) 2
(g ◦ f) (x) = 2x + 10 A1A1 N2
[7]

44. (a) f (x) = 3(x2 + 2x + 1)  12 A1


= 3x2 + 6x + 3  12 A1
= 3x2 + 6x  9 AG N0

(b) (i) vertex is (1, 12) A1A1 N2


(ii) x = 1 (must be an equation) A1 N1
(iii) (0,  9) A1 N1
(iv) evidence of solving f (x) = 0 (M1)
e.g. factorizing, formula,
correct working A1
 6  36  108
e.g. 3(x + 3)(x  1) = 0, x 
6
(3, 0), (1, 0) A1A1 N1N1
(c)
y

x
–3 1

–9

–12
A1A1 N2
Notes: Award A1 for a parabola opening upward,
A1 for vertex and intercepts in
approximately correct positions.

 p  1 
(d)      , t = 3 (accept p =  1, q = 12, t = 3) A1A1A1 N3
 q    12 
[15]

45. (a) evidence of attempting to solve f (x) = 0 (M1)


evidence of correct working A1
1 9
e.g.  x  1 x  2 ,
2
intercepts are (1, 0) and (2, 0) (accept x = 1, x = 2) A1A1 N1N1

(b) evidence of appropriate method (M1)


x x b
e.g. xv  1 2 , xv   , reference to symmetry
2 2a
xv = 0.5 A1 N2
[6]
46. (a)
y
4

–2 –1 0 1 2 3 4 x
–1

–2

–3

–4

M1A1 N2
Note: Award M1 for evidence of reflection in
x-axis, A1 for correct vertex and all
intercepts approximately correct.

(b) (i) g (3) = f (0) (A1)


f (0) =  1.5 A1 N2
  3
(ii) translation (accept shift, slide, etc.) of   A1A1 N2
 0 
[6]
47. (a)
y
15

10

π 0 π x

3 3

–5
A1A1A1 N3
Note: Award A1 for passing through (0, 0), A1
for correct shape, A1 for a range of
approximately 1 to 15.

(b) evidence of attempt to solve f (x) = 1 (M1)


sin x
e.g. line on sketch, using tan x 
cos x
x = 0.207 x = 0.772 A1A1 N3
[6]

48. (a) intercepts when f (x) = 0 (M1)


(1.54, 0) (4.13, 0) (accept x = 1.54 x = 4.13) A1A1 N3
(b)
y
3
2
1

–2 –1 0 1 2 3 4 5 6 x
–1
–2
–3
–4
–5
–6
–7
–8
–9
–10

A1A1A1 N3
Note: Award A1 for passing through
approximately (0,  4), A1 for correct
shape, A1 for a range of approximately
9 to 2.3.

(c) gradient is 2 A1 N1
[7]

49. (a) (i) n=5 (A1)


T = 280  1.125
T = 493 A1 N2
(ii) evidence of doubling (A1)
e.g. 560
setting up equation A1
e.g. 280  1.12n = 560, 1.12n = 2
n = 6.116... (A1)
in the year 2007 A1 N3

2 560 000
(b) (i) P (A1)
10  90 e 0.15 
P = 39 635.993... (A1)
P = 39 636 A1 N3

2 560 000
(ii) P
10  90 e 0.17 
P = 46 806.997... A1
not doubled A1 N0
valid reason for their answer R1
e.g. P < 51200

(c) (i) correct value A2 N2


25600
e.g. , 91.4 , 640 : 7
280
(ii) setting up an inequality (accept an equation, or reversed
inequality) M1
P 2 560 000
e.g.  70 ,  70
T  
10  90e 0.1n 280  1.12 n
finding the value 9.31.... (A1)
after 10 years A1 N2
[17]

50. (a)

A1A1A1 N3
Note: Award A1 for approximately correct (reflected) shape,
A1 for right end point in circle, A1 for through (1, 0).

(b) 0 ≤ y ≤ 3.5 A1 N1
(c) interchanging x and y (seen anywhere) M1
e.g. x = e0.5y
evidence of changing to log form A1
e.g. ln x = 0.5y, ln x = ln e0.5y (any base), ln x = 0.5 y ln e (any base)
f–1(x) = 2 ln x A1 N1
[7]

51. (a) (i) attempt to substitute (M1)


29  15
e.g. a =
2
a = 7 (accept a = –7) A1 N2

(ii) period = 12 (A1)



b= A1
12
π
b= AG N0
6

(iii) attempt to substitute (M1)


29  15
e.g. d =
2
d = 22 A1 N2

(iv) c = 3 (accept c = 9 from a = –7) A1 N1


Note: Other correct values for c can be found,
c = 3 ± 12k, k  .

(b) stretch takes 3 to 1.5 (A1)


translation maps (1.5, 29) to (4.5, 19) (so M′ is (4.5, 19)) A1 N2

π
(c) g(t) = 7 cos (t – 4.5) + 12 A1A2A1 N4
3
π
Note: Award A1 for , A2 for 4.5, A1 for 12.
3
Other correct values for c can be found
c = 4.5 ± 6k, k  .
  3
(d) translation   (A1)
 10 
horizontal stretch of a scale factor of 2 (A1)
completely correct description, in correct order A1 N3
  3
e.g. translation   then horizontal stretch of a scale factor of 2
 10 
[16]

52. (a) evidence of obtaining the vertex (M1)


b
e.g. a graph, x =  , completing the square
2a
f(x) = 2(x + 1)2 – 8 A2 N3

(b) x = –1 (equation must be seen) A1 N1

(c) f(x) = 2(x – 1)(x + 3) A1A1 N2


[6]

53. (a)

A1A2 N3
Notes: Award A1 for correct domain, 0 ≤ x ≤ 3.
Award A2 for approximately correct shape, with
local maximum in circle 1 and right endpoint
in circle 2.

(b) a = 2.31 A1 N1
  f ( x)
2
(c) evidence of using V = π dx (M1)
fully correct integral expression A2
2.31 2.31
e.g. V = π 
0
[ x cos(x  sin x)] 2 dx, V  π 
0
[ f ( x)] 2 dx
V = 5.90 A1 N2
[8]

54. (a) (i) 6 A1 N1

(ii) 9 A1 N1

(iii) 0 A1 N1

(b) x<5 A2 N2

(c) (g ° f)(x) = ( x  5 )2 (M1)


=x–5 A1 N2
[7]

55. (a) For a reasonable attempt to complete the square, (or expanding) (M1)
e.g. 3x2 – 12x + 11 = 3(x2 – 4x + 4) + 11 – 12
f(x) = 3(x – 2)2 – 1 (accept h = 2, k = 1) A1A1 N3

(b) METHOD 1
Vertex shifted to (2 + 3, –1 + 5) = (5, 4) M1
so the new function is 3(x – 5)2 + 4 (accept p = 5, q = 4) A1A1 N2
METHOD 2
g(x) = 3((x – 3) – h)2 + k + 5 = 3((x – 3) – 2)2 – 1 + 5 M1
= 3(x – 5)2 + 4 (accept p = 5, q = 4) A1A1 N2
[6]
 2  1 2  1  2  1  1 0   0 0 
56.     6   k      (A1)
  3 4   3 4    3 4   0 1   0 0 
 7  6
M2 =   A2
  18 19 
 12  6 
6M =   A1
  18 24 
  5 0   k 0   0 0
        A1
 0  5  0 k   0 0
k=5 A1 N2
[6]

57. (a) x2 = 49 (M1)


x = ±7 (A1)
x=7 A1 N3

(b) 2x = 8 (M1)
x=3 A1 N2

1

(c) x = 25 2 (M1)
1
x= (A1)
25
1
x= A1 N3
5

(d) log2 (x(x – 7)) = 3 (M1)


log2 (x2 – 7x) = 3
23 = 8 (8 = x2 – 7x) (A1)
2
x – 7x – 8 = 0 A1
(x – 8)(x + 1) = 0 (x = 8, x = –1) (A1)
x=8 A1 N3
[13]

58. (a) Evidence of completing the square (M1)


f(x) = 2(x2 – 6x + 9) + 5 – 18 (A1)
= 2(x – 3)2 – 13 (accept h = 3, k = 13) A1 N3

(b) Vertex is (3, –13) A1A1 N2

(c) x = 3 (must be an equation) A1 N1

(d) evidence of using fact that x = 0 at y-intercept (M1)


y-intercept is (0, 5) (accept 5) A1 N2

(e) METHOD 1
evidence of using y = 0 at x-intercept (M1)
e.g. 2(x – 3)2 – 13 = 0
evidence of solving this equation (M1)
13
e.g. (x – 3)2 = A1
2
13
(x – 3) = 
2
13 26
x=3±  3 A1
2 2
6  26
x=
2
p = 6, q = 26, r = 2 A1A1A1 N4

METHOD 2
evidence of using y = 0 at x-intercept (M1)
e.g. 2x2 – 12x + 5 = 0
evidence of using the quadratic formula (M1)
12  12  4  2  5
2
x= A1
2 2
12  104  6  26 
x=   A1
4  2 
 
p = 12, q = 104, r = 4 (or p = 6, q = 26, r = 2) A1A1A1 N4
[15]
59. (a)

A1A1 N2
Note: Award A1 for the left branch, and A1 for the right
branch.

1
(b) g(x) = +3 A1A1 N2
x2

 1 
(c) (i) Evidence of using x = 0  g (0)    3 (M1)
 2 
5
y= (= 2.5) A1
2
evidence of solving y = 0 (1 + 3(x – 2) = 0) M1
1 + 3x – 6 = 0 (A1)
3x = 5
5
x= A1
3
5 5 5   5
Intercepts are x = , y = (accept  , 0   0,  ) N3
3 2 3   2

(ii) x=2 A1 N1
y=3 A1 N1
(iii)

A1A1A1 N3
Note: Award A1 for the shape (both branches), A1 for the
correct behaviour close to the asymptotes, and A1
5   5
for the intercepts at approximately  , 0   0,  .
3   2
[14]

60. (a)

A1A1 N2
Note: Award A1 for the general shape and A1 for the
y-intercept at 1.

(b) x = 3, x = –3 A1A1 N1N1

(c) y≥1 A2 N2
[6]

61. (a) (f ° g): x  3(x + 2) (= 3x + 6) A2 N2


(b) METHOD 1
Evidence of finding inverse functions M1
x
e.g. f–1(x) = g–1(x) = x – 2
3
18
f–1(18) = (= 6) (A1)
3
g–1(18) = 18 – 2 (= 16) (A1)
f–1(18) + g–1(18) = 6 + 16 = 22 A1 N3

METHOD 2
Evidence of solving equations M1
e.g. 3x = 18, x + 2 = 18
x = 6, x = 16 (A1)(A1)
f–1(18) + g–1(18) = 6 + 16 = 22 A1 N3
[6]

62. (a) using the cosine rule a2 = b2 + c2 – 2bc cos  (M1)


substituting correctly BC2 = 652 + 1042 – 2(65)(104)cos60° A1
= 4225 + 10 816 – 6760 = 8281
 BC = 91m A1 N2

1
(b) finding the area, using bc sin Aˆ (M1)
2
1
substituting correctly, area = (65)(104)sin60° A1
2
= 1690 3 (accept p = 1690) A1 N2

1
(c) (i) A1 =   (65)(x)sin30° A1
2
65x
= AG N0
4

1
(ii) A2 =   (104)(x)sin30° M1
2
= 26x A1 N1

65x
(iii) stating A1 + A2 = A or substituting + 26x = 1690 3 (M1)
4
169 x
simplifying  1690 3 A1
4
4  1690 3
x= A1
169
 x = 40 3 (accept q = 40) A1 N2

(d) (i) Recognizing that supplementary angles have equal sines


e.g. AD̂C = 180° – AD̂B  sinA D̂C  sinA D̂B R1

(ii) using sin rule in ∆ADB and ∆ACD (M1)


BD 65 BD sin30 
substituting correctly    A1
sin30  sinA D̂B 65 sinA D̂B
DC 104 DC sin30 
and    M1
sin30  sinA D̂C 104 sinA D̂C
since sinA D̂B  sinA D̂C
BD DC BD 65
   A1
65 104 DC 104
BD 5
  AG N0
DC 8
[18]

63. (a) f 1
x   ln x A1 N1
(b) (i) Attempt to form composite (f ◦ g) (x) = f (ln (1 + 2x)) (M1)
(f ◦ g) (x) = eln (1 + 2x) = (= 1 + 2x) A1 N2
(ii) Simplifying y = eIn(1 + 2x) to y = 1 + 2x (may be seen in part
(i) or later) (A1)
Interchanging x and y (may happen any time) M1
eg x = 1 + 2y x  1 = 2y
x  1
(f ◦ g)1 (x) = A1 N2
2
[6]

64. (a) (i) 0 A1 N1


1
(ii)  A1 N1
2

(b)
y
5

3
2
1

x
–5 –4 –3 –2 –1 0 1 2 3 4 5
–1
–2
–3
–4
–5
A2 N2
(c)
y
5

3
2
1

x
–5 –4 –3 –2 –1 0 1 2 3 4 5
–1
–2
–3
–4
–5
A2 N2
[6]

65. (a) Two correct factors A1A1


eg y2 + y  12 = (y + 4)(y  3), (2x)2 + (2x)  12 = (2x + 4)(2x  3)
a = 4, b = 3 (or a = 3, b = 4) N2
(b) 2x  3 = 0 (M1)
2x = 3
ln 3  log 3 
x=  log 2 3 , etc. A1 N2
ln 2  log 2 
EITHER
Considering 2x + 4 = 0 (2x = 4) (may be seen earlier) A1
Valid reason R1 N1
eg this equation has no real solution, 2x > 0, graph does not cross the
x-axis
OR
Considering graph of y = 22x + 2x  12 (asymptote does not need to
be indicated) A1

There is only one point of intersection of the graph with x-axis. R1 N1


[6]

66. (a) 253250 (accept 253000) A1 N1

(b) 1972  2002 is 30 years, increase of 1.3%  1.013 (A1)(A1)


Evidence of any appropriate approach (M1)
Correct substitution 250000  1.01330 A1
368000 (accept 368318) A1 N3
[6]
67. (a) METHOD 1
f (3) = 7 (A1)
(g ◦ f) (3) = 7 A1 N2
METHOD 2
2
(g ◦ f) (x) = x  4 (= x + 4) (A1)
(g ◦ f) (3) = 7 A1 N2

(b) For interchanging x and y (seen anywhere) (M1)


Evidence of correct manipulation A1
eg x = y  4 , x2  y  4
f 1(x) = x2  4 A1 N2

(c) x0 A1 N1
[6]

68. (a) METHOD 1


Using the discriminant = 0 (q2  4(4)(25) = 0) M1
q2 = 400
q = 20, q = 20 A1A1 N2
METHOD 2
Using factorizing:
(2x  5)(2x  5) and/or (2x + 5) (2x + 5) M1
q = 20, q = 20 A1A1 N2

(b) x = 2.5 A1 N1

(c) (0, 25) A1A1 N2


[6]

69. (a) x = 1, (1, 0), 1 A1 N1

(b) (i) f (1.999) = ln (0.001) = 6.91 A1 N1


(ii) All real numbers. A2 N2

(c) (4.64, 1.89) A1A1 N2


[6]

70. (a)
y
6

5 2
4

2 3
1
A (–1, 1)
1

–5 –3 0 x
–4 –2 –1 1 2 3 4 5
–1

–2

–3

A1A1A1 N3
Notes: Award A1 for left end point in circle 1,
A1 for maximum point in circle 2,
A1 for right end point in circle 3.

(b) y = 1 (must be an equation) A1 N1

(c) (0, 3) A1A1 N2


[6]

71. (a) (i) p = 1, q = 5 (or p = 5, q = 1) A1A1 N2


(ii) x=3 (must be an equation) A1 N1

(b) y = (x  1)(x  5)
= x2  6x + 5 (A1)
= (x  3)2  4 (accept h = 3, k = 4) A1A1 N3

 2x  3  2 x  6
dy
(c) A1A1 N2
dx

dy
(d) When x = 0,  6 (A1)
dx
y  5 = 6(x  0) (y = 6x + 5 or equivalent) A1 N2
[10]

72. (a)

A1A1A1 N3
Notes: Award A1 for both asymptotes shown.
The asymptotes need not be labelled.
Award A1 for the left branch in
approximately correct position,
A1 for the right branch in
approximately correct position.

5
(b) (i) y = 3, x = (must be equations) A1A1 N2
2
14  7  14  
(ii) x=  or 2.33 , also accept , 0   A1 N1
6 3  6 
  14  
(iii) y=
14
 y  2.8  accept 0 ,  or 0 , 2.8 A1 N1
6   5 

 
9  6  1
(c) (i)   2 x  5 2 x  52 

 dx  9 x 

3 ln 2 x  5 
1
C A1A1A1
22 x  5
A1A1 N5
b
(ii) Evidence of using V =  a
y 2 dx (M1)
Correct expression A1
a 
2
 1  9  6  1
a
eg 
3
 3   dx , 
 2x  5 
3 
 2 x  5 2 x  52
 dx,


a
 
9 x  3 ln 2 x  5 
1

 22 x  5 3
   1
Substituting  9a  3 ln 2a  5 
1
   27  3ln 1   A1
 22a  5   2
Setting up an equation (M1)
 28 
 27   3 ln 2a  5  3 ln 1    3 ln 3 
1 1
9a 
22a  5 2  3 
Solving gives a = 4 A1 N2
[17]

73. (a) (i) p=2 A1 N1


(ii) q=1 A1 N1

(b) (i) f (x) = 0 (M1)


3x
2 2 =0 (2x2  3x  2 = 0) A1
x 1
1
x=  x=2
2
 1 
  , 0 A1 N2
 2 
b
(ii) Using V =  a
y2dx (limits not required) (M1)
2
0  3x 
V= ∫ 1
2
  2  2  dx
 x 1 
A2

V = 2.52 A1 N2
(c) (i) Evidence of appropriate method M1
eg Product or quotient rule
Correct derivatives of 3x and x2  1 A1A1
Correct substitution A1
 3 ( x 2  1)  (3x) (2 x)
eg
( x 2  1) 2
 3x 2  3  6 x 2
f ′ (x) = A1
( x 2  1) 2
3x 2  3 3( x 2  1)
f ′ (x) = = AG N0
( x 2 1) 2 ( x 2 1) 2

(ii) METHOD 1
Evidence of using f ′(x) = 0 at max/min (M1)
3 (x2 + 1) = 0 (3x2 + 3 = 0) A1
no (real) solution R1
Therefore, no maximum or minimum. AG N0
METHOD 2
Evidence of using f ′(x) = 0 at max/min (M1)
Sketch of f ′(x) with good asymptotic behaviour A1
Never crosses the x-axis R1
Therefore, no maximum or minimum. AG N0
METHOD 3
Evidence of using f ′ (x) = 0 at max/min (M1)
Evidence of considering the sign of f ′ (x) A1
f ′ (x) is an increasing function (f ′ (x)  0, always) R1
Therefore, no maximum or minimum. AG N0
(d) For using integral (M1)
 a a 3x  3
2

 
a
Area =  0
g ( x) dx  or
 0
f  ( x) dx or
0 ( x  1)
2
dx 
2

A1

a a
Recognizing that  0
g ( x) dx  f ( x)
0
A2
Setting up equation (seen anywhere) (M1)
Correct equation A1
a 3x 2  3  3a 
eg  dx = 2, 2  2   2  0 = 2, 2a2 + 3a  2 = 0
0 ( x 2  1) 2
 a  1
1
a= a=2
2
1
a= A1 N2
2
[24]

74. (a) (i) f (a) = 1 A1 N1


(ii) f (1) = 0 A1 N1
(iii) f (a4) = 4 A1 N1
(b)
y f –1

1
f

–2 –1 0 1 2 x

–1

–2

A1A1A1 N3
Note: Award A1 for approximate reflection of
f in y = x, A1 for y intercept at 1, and
A1 for curve asymptotic to x axis.
[6]

75. (a) (i) h=3 A1 N1


(ii) k=1 A1 N1

(b) g (x) = f (x  3) + 1, 5  (x  3)2 + 1, 6  (x  3)2,  x2 + 6x  3 A2 N2


(c)
y
T
V

–8 0 8 x

M1A1 N2
Note: Award M1 for attempt to reflect through
y-axis, A1 for vertex at approximately ( 3, 6).
[6]

76. (a) 1 = A0 e5k A1


dA
Attempt to find (M1)
dt
dA
eg  k A0 e kt
dt
Correct equation 0.2 = k A0 e5k A1
For any valid attempt to solve the system of equations M1
0.2 k A0 e 5k
eg 
1 A0 e 5k
k = 0.2 AG N0

1 0.2t
(b) 100 = e A1
e
ln 100  1
t  28.0 A1 N1
0.2
[6]
77. (a)
y

20

10

x
–2 –1 1 2

–10

–20

A1A1A1 N3
Note: Award A1 for the left branch asymptotic
to the x-axis and crossing the y-axis,
A1 for the right branch approximately
the correct shape,
A1 for a vertical asymptote at
1
approximately x = .
2

1
(b) (i) x (must be an equation) A1 N1
2
2
(ii)  0
f ( x ) dx A1 N1
(iii) Valid reason R1 N1
eg reference to area undefined or discontinuity
Note: GDC reason not acceptable.
f x  2 dx
1.5
(c) (i) V =  A2 N2
1

(ii) V = 105 (accept 33.3 ) A2 N2

(d) f (x) = 2e2x  1  10(2x  1)2 A1A1A1A1 N4

(e) (i) x = 1.11 (accept (1.11, 7.49)) A1 N1


(ii) p = 0, q = 7.49 (accept 0  k < 7.49) A1A1 N2
[17]

78. (a) METHOD 1


Using the discriminant  = 0 (M1)
k2 = 4  4  1
k = 4, k =  4 A1A1 N3
METHOD 2
Factorizing (M1)
(2x  1)2
k = 4, k =  4 A1A1 N3

(b) Evidence of using cos 2 = 2 cos2   1 M1


eg 2(2 cos2   1) + 4 cos  + 3
f () = 4 cos2  + 4 cos  + 1 AG N0
(c) (i) 1 A1 N1
(ii) METHOD 1
Attempting to solve for cos  M1
1
cos  =  (A1)
2
 = 240, 120,  240, 120 (correct four values only) A2 N3
METHOD 2
Sketch of y = 4 cos2  + 4 cos  + 1 M1
y
9

–360 –180 180 360 x

Indicating 4 zeros (A1)


 = 240, 120, 240, 120 (correct four values only) A2 N3

(d) Using sketch (M1)


c=9 A1 N2
[11]

79. (a) D A2 N2

(b) C A2 N2

(c) A A2 N2
[6]

80. (a) Vertex is (4, 8) A1A1 N2

(b) Substituting 10 = a(7  4)2 + 8 M1


a = 2 A1 N1

(c) For y-intercept, x = 0 (A1)


y = 24 A1 N2
[6]

81. (a) METHOD 1


For f (2) = 12 (A1)
(g ◦ f) (2) = g (12) = 24 A1 N2
METHOD 2
(g ◦ f) (x) = 2x3 8 (A1)
(g ◦ f) (2) = 24 A1 N2

(b) Interchanging x and y (may be done later) (M1)


x = y3  4 A1
f 1(x) = 3  x  4  A2 N3
[6]

82. (a)
y

x
0 1 2 3 4

–1

–2

–3

A1A1 N2
Note: Award A1 for approximate parabolic
shape with correct orientation, A1 for
maximum with 2.5 < x < 3, and 1 < y < 2.

(b) 3.19 A2 N2

(c) p = 1.89, q = 3.19 A2 N2


[6]

83. (a) eln(x + 2) = e3 (M1)


x + 2 = e3 (A1)
x = e3  2 (= 18.1) A1 N3

(b) log10 (102x) = log10 500 (accept lg and log for log10) (M1)
2x = log10 500 (A1)
1  log 500 
x= log10 500    1.35  A1 N3
2  log 100 
Note: In both parts (a) and (b), if candidates use
a graphical approach, award M1 for a
sketch, A1 for indicating appropriate points
of intersection, and A1 for the answer.
[6]

84. (a) For attempting to complete the square or expanding y = 2(x  c)2 + d,
or for showing the vertex is at (3, 5) M1
y = 2(x  3)2 + 5 (accept c = 3, d = 5) A1A1 N2

(b) (i) k=2 A1 N1


(ii) p=3 A1 N1
(iii) q=5 A1 N1
[6]
85. (a) METHOD 1
Attempting to interchange x and y (M1)
Correct expression x = 3y  5 (A1)
x5
f 1 ( x)  A1 N3
3
METHOD 2
Attempting to solve for x in terms of y (M1)
y5
Correct expression x  (A1)
3
x  5
f 1 ( x)  A1 N3
3

(b) For correct composition (g1◦ f) (x) = (3x  5) + 2 (A1)


(g1◦ f) (x) = 3x  3 A1 N2

x3
(c)  3x  3  x  3  9 x  9 (A1)
3
12
x A1 N2
8
(d) (i)

y=3

x=2

A1A1A1 N3
Note: Award A1 for approximately correct x
and y intervals, A1 for two branches of
correct shape, A1 for both asymptotes.
(ii) (Vertical asymptote) x = 2, (Horizontal asymptote) y = 3 A1A1 N2
(Must be equations)

(e) (i) 3x + ln (x  2) + C (3x + ln x  2 + C) A1A1 N2


(ii) 3x  ln x  253 (M1)
= (15 + ln 3)  (9 + ln1) A1
= 6 + ln 3 A1 N2

(f) Correct shading (see graph). A1 N1


[18]
86. (a) METHOD 1
Note: There are many valid algebraic approaches
to this problem (eg completing the square,
b
using x  ) . Use the following mark
2a
allocation as a guide.
dy
(i) Using 0 (M1)
dx
32x + 160 = 0 A1
x=5 A1 N2
(ii) ymax = 16(52) + 160(5)  256
ymax = 144 A1 N1
METHOD 2
(i) Sketch of the correct parabola (may be seen in part (ii)) M1
x=5 A2 N2
(ii) ymax = 144 A1 N1

(b) (i) z = 10  x (accept x + z = 10) A1 N1


(ii) z = x + 6 2  x  6  cos Z
2 2 2
A2 N2
(iii) Substituting for z into the expression in part (ii) (M1)
Expanding 100  20x + x2 = x2 + 36  12x cos Z A1
Simplifying 12x cos Z = 20x  64 A1
20 x  64
Isolating cos Z = A1
12 x
5 x 16
cos Z = AG N0
3x
Note: Expanding, simplifying and isolating may
be done in any order, with the final A1
being awarded for an expression that
clearly leads to the required answer.

(c) Evidence of using the formula for area of a triangle


 1 
 A   6  x  sin Z  M1
 2 
 1 
A  3x sin Z  A 2   3 6 x 2  sin 2 Z  A1
 4 
2 2 2
A = 9x sin Z AG N0

(d) Using sin2 Z = 1  cos2 Z (A1)


5 x 16
Substituting for cos Z A1
3x
 5 x 16 
2
 25 x 2 160 x  256 
for expanding   to   A1
 3x   9x 2 
 
for simplifying to an expression that clearly leads to the required answer A1
eg A2 = 9x2  (25x2  160x + 256)
A2 = 16x2 + 160x  256 AG

(e) (i) 144 (is maximum value of A2, from part (a)) A1
Amax = 12 A1 N1
(ii) Isosceles A1 N1
[20]

87. (a) (i) m=3 A2 N2


(ii) p=2 A2 N2
(b) Appropriate substitution M1
eg 0 = d(1  3)2 + 2, 0 = d(5  3)2 + 2, 2 = d(3  1)(3  5)
1
d  A1 N1
2
[6]

88. (a) METHOD 1


5x + 1 = 54 A1
x+1=4 (A1)
x=3 A1 N2
METHOD 2
Taking logs A1
eg x + 1 = log5 625, (x + 1)log 5 = log 625

x+1=
log 625
x  1  4 (A1)
log 5
x=3 A1 N2
(b) METHOD 1
Attempt to re-arrange equation (M1)
3x + 5 = a2 A1
a2 5
x A1 N2
3
METHOD 2
Change base to give log (3x + 5) = log a2 (M1)
3x + 5 = a2 A1
a2  5
x A1 N2
3
[6]

89. (a) Evidence of attempting to form composition (M1)


53x  2
Correct substitution (h ◦ g) (x) = A1
3x  2  4
53x  2  15 x 10   53x  2 
=       A1 N2
3x  6  3x  6   3x  2 

(b) Evidence of using numerator = 0 (M1)


eg 15x  10 = 0 (3x  2 = 0)
x   0.667 
2
A2 N3
3
[6]

90. (a) q=0 A1 N1

(b) Attempting to substitute (3, 18) (M1)


m33 + n32 + p3 = 18 A1
27m + 9n + 3p = 18 AG N0

(c) m+n+p=0 A1 N1
 m + n  p = 10 A1 N1
(d) (i) Evidence of attempting to set up a matrix equation (M1)
Correct matrix equation representing the given equations A2 N3
 27 9 3   m   18 
    
eg  1 1 1  n    0 
  1 1  1  p    10 
    
 2
 
(ii)   5 A1A1A1 N3
 3 
 

(e) Factorizing (M1)


eg f (x) = x(2x2  5x + 3), f (x) = (x2  x)(rx  s)
r=2 s=3 (accept f (x) = x(x  1)(2x  3)) A1A1 N3
[14]

91. (a)
y

Q
1
P
R

x
1 2 3

A1A1A1 N3
Note: Award A1 for the shape of the curve,
A1 for correct domain,
A1 for labelling both points P and
Q in approximately correct positions.
(b) (i) Correctly finding derivative of 2x + 1 ie 2 (A1)
Correctly finding derivative of ex ie ex (A1)
Evidence of using the product rule (M1)
f  (x) = 2ex + (2x + 1)(ex) A1
= (1  2x)ex AG N0
(ii) At Q, f (x) = 0 (M1)
x = 0.5, y = 2e0.5 A1A1
Q is (0.5, 2e0.5) N3

(c) 1  k < 2e0.5 A2 N2

(d) Using f  (x) = 0 at the point of inflexion M1


ex (3 + 2x) = 0
This equation has only one root. R1
So f has only one point of inflexion. AG N0

(e) At R, y = 7e3 (= 0.34850 ...) (A1)


7e 3 1
Gradient of (PR) is   0.2172  (A1)
3
 7e 3 1 
Equation of (PR) is g (x) =   x 1  0.2172 x  1 A1

 3 
Evidence of appropriate method, involving subtraction of integrals
or areas M2
Correct limits/endpoints A1

  f x   g x 
3
eg dx, area under curve  area under PR
0

3  3 
 2 x  1 e  x   7e 1 x  1  dx
Shaded area is 0

 3



= 0.529 A1 N4
[21]
92. (a) (f ° g): x  3(x + 2) (= 3x + 6) A2 2
(b) METHOD 1
f –1(x) = x g–1(x) = x – 2 (M1)
3
f –1(18) = 18 A1
3
g–1(18) = 18 – 2 A1
f –1(18) + g–1(18) = 6 +16 A1
f –1(18) + g–1(18) = 22 AG 4

METHOD 2
3x = 18, x + 2 = 18 (M1)
x = 6, x = 16 A1A1
f –1(18) + g–1(18) = 6 +16 A1
f –1(18) + g–1(18) = 22 AG 4
[6]

93. (a)
y
6

x
–3 –2 –1 1 2 3
A1A1 2
Note: Award (A1) for the general shape and (A1) for the j-
intercept at 1.

(b) x = 3, x = –3 A1A1 2
(c) y1 A2 2
Note: Award N1 for y > 1.
[6]
94. (a) For a reasonable attempt to complete the square, (or expanding)
3x2 – 12x + 11 = 3(x2 – 4x + 4)+ 11 – 12
= 3(x – 2)2 – 1 (Accept h = 2, k = l) A1A1 2

(b) METHOD 1
Vertex shifted to (2 + 3, –1 + 5) = (5, 4) M1
so the new function is 3 (x – 5)2 + 4 (Accept p = 5, q = 4) A1A1 2
METHOD 2
g (x) = 3((x – 3) – h)2 + k + 5 = 3((x – 3)–2)2 – 1 + 5 M1
= 3(x – 5)2 + 4 (Accept p = 5, q = 4) A1A1 2
[6]

95. (a) (i) p = (10x + 2) – (1 + e2x) A2 2


Note: Award (A1) for (l + e2x) – (10x + 2).
dp
(ii) = 10 – 2e2x A1A1
dx
dp
= 0 (10 – 2e2x = 0) M1
dx
1n 5
x= (= 0.805) A1 4
2

(b) (i) METHOD 1


x = 1 + e2x M1
1n(x – 1) = 2y A1
1n( x  1)  1n( x  1) 
f –1(x) =  Allow y   A1 3
2  2 
METHOD 2
y – 1 = e2x A1
ln( y  1)
=x M1
2
1n( x  1)  1n( x  1) 
f –1(x) =  Allow y   A1 3
2  2 
1n (5  1)  1
  1n 2 
2
(ii) a= M1
2  2 
= 1 × 21n2 A1
2
= 1n 2 AG 2

b
(c) Using V =  a
πy 2 d x (M1)

π(1  e 2 x ) 2 dx  or π(1  e 2 x ) 2 dx 
ln 2 0.805
Volume =  0  
0 
A2 3
[14]

96. (a) y  2 x  3
gradient of line L1  2 (A1) (C1)
Note: Award (A0) for 2x .

(b) METHOD 1
( y  y1 )  m ( x  x1 )   y  (4)   2( x  6) (M1)
y  4  2 x  12 (A1)
y  2 x  8 (A1) (C3)

METHOD 2
Substituting the point  6, 4  in y  mx  c , ie 4 = –2(6) + b (M1)
b8 (A1)
y  2 x  8 (A1) (C3)

(c) when line L1 cuts the x-axis, y = 0 (M1)


y  2 x  8
x4 (A1) (C2)
[6]
97. (a) interchanging x and y (may happen later) x  e y 11  8 (M1)
e y 11  x  8 (A1)
 
ln e y 11  ln ( x  8) (A1)
1
f ( x)  ln ( x  8)  11 (A1) (C4)

(b) Domain is x  8 (A2) (C2)


Note: Award (A1)(A0) for x  8 .
[6]

98. (a) (i)


y
2

–2 –1 0 1 2 3 4 5 6 7 8 x
–1

–2
(A2) (C2)

(ii)
y
2

–2 –1 0 1 2 3 4 5 6 7 8 x
–1

–2
(A2) (C2)

(b) A′ (3, 2) (Accept x  3 , y  2 ) (A1)(A1) (C2)


[6]

99. (a) (i) p  2 q  4 (or p  4, q  2 ) (A1)(A1) (N1)(N1)

(ii) y  a ( x  2)( x  4)
8  a (6  2)(6  4) (M1)
8  16a
1
a (A1) (N1)
2

1
(iii) y  ( x  2)( x  4)
2
1 2
y  ( x  2 x  8)
2
1
y  x2  x  4 (A1) (N1) 5
2
dy
(b) (i)  x 1 (A1) (N1)
dx
(ii) x 1  7 (M1)
x  8, y  20  P is (8, 20)  (A1)(A1) (N2) 4

(c) (i) when x = 4, gradient of tangent is 4 – 1 = 3 (may be implied) (A1)


1
gradient of normal is  (A1)
3
1  1 4
y  0   ( x  4)  y   x   (A1) (N3)
3  3 3
1 2 1 4
(ii) x  x  4   x  (or sketch/graph) (M1)
2 3 3
1 2 2 16
x  x 0
2 3 3
3x2  4 x  32  0 (may be implied) (A1)
(3x  8)( x  4)  0
8
x   or x  4
3
8
x   (2.67) (A1) (N2) 6
3
[15]

100. (a) p = 1 and q = 3 (or p =3, q = 1) (A1)(A1) (C2)


(accept (x + 1)(x  3))

(b) EITHER
by symmetry (M1)
OR
dy
differentiating = 2x  2 = 0 (M1)
dx
OR
Completing the square (M1)
x2 + 2x  3 = x2  2x + 1  4 = (x  1)2  4
THEN
x = 1, y =  4 (so C is (1,  4)) (A1)(A1)(C2)(C1)

(c) 3 (A1) (C1)


(accept (0,  3))
[6]
101. (a) METHOD 1
(f ◦ g) (4) = f (g (4)) = f (1) (M1)
=2 (A1) (C2)
METHOD 2
2
(f ◦ g) (x) = (M1)
x 3
(f ◦ g) (4) = 2 (A1) (C2)

1
(b) Let y =
x 3
 1
Correct simplification y(x  3) = 1  x  3   (A1)
 y
1  1 3 y 
x 3    (A1)
y  y 
Interchanging x and y (may happen earlier) (M1)
1  1  3x 
y  3   (C3)
x  x 

(c) x0 ( \ {0} etc) (A1) (C1)


[6]

102. 10 000e 0.3t = 1500 (A1)


For taking logarithms (M1)
 0.3t ln e = ln 0.15 (A1)
ln 0.15
t (A1)
 0.3
= 6.32 (A1)
7 (years) (A1) (C6)
Note: Candidates may use a graphical method.
Award (A1) for setting up the correct
equation, (M1)(A1) for a sketch, (A1)
for showing the point of intersection,
(A1) for 6.32, and (A1) for 7.
[6]
103. (a) b=6 (A1) (C1)

(b)
y
B

x
1 2

(A3) (C3)

(c) x = 1.05 (accept (1.05, −0.896) ) (correct answer only, no additional


solutions) (A2) (C2)
[6]

104. (a) METHOD 1


53  13
Finding gradient m = ( 5) (A1)
10  2
y  13 = 5(x  2) (M1)
y = 5x + 3 (AG) (N0)
METHOD 2
u3 = 13 and u11 = 53 (M1)
u1 = 3 and d = 5 (A1)
y = 5x + 3 (AG) (N0)
Note: Award no marks for showing that (2, 13) and
(10, 53) satisfy y = 5x + 3.

(b) 3 kg (A1) (N1)

(c) Increase is 5 kg (per week) (A1) (N1)


(d) 98 = 5x + 3 (M1)
5x = 95
x = 19 (A1) (N2)
[6]

105. (a) (i) x =10 (A1) (N1)


(ii) y=8 (A1) (N1)

(b) (i) 6.4 (or (0, 6.4) ) (A1) (N1)


(ii) 8 (or (8, 0) ) (A1) (N1)

(c)

y=8
(0, 6, 4)

(8, 0)
x = 10

(A1)(A1)(A1)(A1) (N4)
Note: Award (A1) for both asymptotes
correctly drawn, (A1) for both
intercepts correctly marked,
(A1)(A1) for each branch drawn
in approximately correct
positions. Asymptotes and
intercepts need not be labelled.
(d) There is a vertical translation of 8 units.
0
(accept translation of   ) (A2) (N2)
 8
[10]

106. (a) x = 1.43 (A2) (N2)

(b) f (x) = 0
f  (x) = 12x3  12x2  60x  36 (may be implied) (A1)
Setting first derivative equal to zero (M1)
f  (x) = 12x3  12x2  60x  36 = 0
x = 1 (is other solution) (A1) (N2)

(c) f (x) = 0
f (x) = 36x2  24x  60 (may be implied) (A1)
Setting second derivative equal to zero (M1)
f (x) = 36x2  24x  60 = 0
5
x  , 1 (A1)(A1) (N3)
3

(d) (1, 125) (or x = 1, y = 125) (A1)(A1) (N2)


Note: Award no marks if this answer is seen
together with extra answers.

(e) x = 4 , x = 1.43 (allow ft from part (a)) (A1)(A1) (N2)

1
(f) tangent to graph of horizontal  tangent to graph of f is
f
horizontal (M1)
x=3 (A1) (N2)
[15]
107.
y
6

5
(a)

3
(b)

–6 –5 –4 –3 –2 –1 0 1 2 3 4 5 6 x
–1

–2

–3

–4

–5

–6
(a) (A1)(A1) (C2)
(b) (A1)(A3) (C4)

(a) Note: Award (A1) for the correct line, (A1) for using the given domain.
(b) Correct domain (A1)
EITHER
The correct line drawn (A3)
OR
g (x) = f (x + 3) – 2
= (2(x + 3) + 1) – 2 (M1)
= 2x + 5 (A1)
Candidate’s line drawn (A1)

OR
g (–3) = –1 g (–1) = 3 (A1)(A1)
Line joining g (–3) and g (–1) drawn (A1)
[6]
108. Discriminant ∆ = b2 – 4ac (= (–2k)2 – 4) (A1)
∆>0 (M2)
Note: Award (M1)(M0) for ∆ ≥ 0.
(2k)2 – 4 > 0  4k2 – 4 > 0
EITHER
4k2 > 4 (k2 > 1) (A1)
OR
4(k – 1)(k + 1) > 0 (A1)
OR
(2k – 2)(2k + 2) > 0 (A1)

THEN
k < –1 or k > 1 (A1)(A1) (C6)
Note: Award (A1) for –1 < k <1.
[6]

109. (a) (i) 2420 (A1)


(ii) 1420 + 100n > 2000 (M1)
n > 5.8
1999 (accept 6th year or n = 6) (A1) (N1) 3
Note: Award (A0) for 2000, or after 6 years, or n = 6, 2000.

(b) (i) 1 200 000(1.025)10 = 1 536 101


(accept 1 540 000 or 1.54(million)) (A1)

1 536 101  1 200 000


(ii) × 100 (M1)
1200 000
28.0% (accept 28.3% from 1 540 000) (A1) (N2)

(iii) 1 200 000(1.025)n > 2 000 000 (accept an equation) (M1)


 2 
n log 1.025 > log    n > 20.69 (M1)(A1)
 1.2 
2014 (accept 21st year or n = 21) (A1) (N3) 7
Note: Award (A0) for 2015, after 21 years, or n = 21, so 2015.
1 200 000
(c) (i) = 845 (A1)
1420
1 200 000 (1.025) n
(ii) < 600 (M1)(M1)
1420  100 n
 n > 14.197
15 years (A2) (N2) 5
[15]

110. (a) y  2x  1
x  2 y 1 (M1)
x 1
y
2
x 1
f 1 ( x)  (A1) (C2)
2

(b) g  f (2)   g (3) (A1)


 3(3)2  4
 23 (A1) (C2)

(c) f  g ( x)   f (3x 2  4)
 2(3x2  4)  1 (A1)
 6x  7
2
(A1) (C2)
[6]
111. Note: Award no marks if candidates work in degrees.
(a) (A1)(A1)(A1)(A1) (C4)
y
4
3
2
1
0
0.5 1 1.5 2 2.5 3 x
–1
P
–2
–3
–4

(b) 1.26, 2.26 (A1)(A1) (C1)(C1)


[6]

112. (a) p  100e0 (M1)


 100 (A1) (C2)

dp
(b) Rate of increase is (M1)
dt
dp
 0.05 100e0.05t  5e 0.05t (A1)(A1)
dt
When t  10
dp
 5e0.05(10)
dt
 5e0.5 
 8.24  5 e  (A1) (C4)
[6]

113. (a) (i) 1 (A1) (C1)


(ii) 2 (A1) (C1)
(iii) f (14)  f (2)  or f (5) or f (8)  (M1)
= –1 (A1) (C2)

(b) There are five repeated periods of the graph, each with two solutions, (R1)
(ie number of solutions is 5 × 2)
 10 (A1) (C2)
[6]

114. (a) h3 (A1)


k2 (A1) 2

f ( x)   ( x  3)  2
2
(b)
  x2  6 x  9  2 (must be a correct expression) (A1)
  x2  6 x  7 (AG) 1

(c) f ( x)  2 x  6 (A2) 2
(d) (i) tangent gradient  2 (A1)
1
gradient of L 
2
(A1) (N2) 2

(ii) EITHER
1
equation of L is y  xc (M1)
2
c  1 . (A1)
1
y  x 1
2
OR
1
y  1  ( x  4)
2
(A2) (N2) 2
(iii) EITHER
1
 x2  6 x  7  x 1 (M1)
2
2 x  11x  12  0 (may be implied)
2
(A1)
(2 x  3)( x  4)  0 (may be implied) (A1)
x  1.5
(A1) (N3) 4
OR
1
 x2  6 x  7  x  1 (or a sketch) (M1)
2
x  1.5
(A3) (N3) 8
[13]

115. (a) (i) f ( x)  6sin 2 x (A1)(A1)


(ii) EITHER
f ( x)  12sin x cos x  0
 sin x  0 or cos x  0 (M1)
OR
sin 2 x  0 ,
for 0  2 x  2 (M1)
THEN
π
x  0, , π
2
(A1)(A1)(A1) (N4) 6

(b) (i) translation (A1)


in the y-direction of –1 (A1)
(ii) 1.11 (1.10 from TRACE is subject to AP) (A2) 4
[10]

116. (a) (i) a  1   accept (1   , 0)  (A1)


(ii) b  1   accept (1   , 0)  (A1) 2
1 2
(b) (i)   2.14
h ( x)dx   h ( x)dx
1
(M1)(A1)(A1)
OR
1 2
  2.14
h ( x) dx   1
h ( x) dx (M1)(A1)(A1)
OR
1 1
  2.14
h ( x)dx   h ( x)dx
2
(M1)(A1)(A1)
(ii) 5.141...  (0.1585...)
= 5.30 (A2) 5

(c) (i) y = 0.973 (A1)


(ii) 0.240  k  0.973 (A3) 4
[11]

117. (a) x = e–y (M1)


ln x = –y (A1)
y = f –1(x) = –ln x (A1) (C3)

(b) (g ° f ) (x) = g(e–x) (M1)


–x
e
= (A2) (C3)
1  e– x
x

1 x
Note: Award (M1)(A1) for = e (ie for (f ° g) (x))
[6]

118. Method 1
b2 – 4ac = 9 – 4k (M1)
9 – 4k > 0 (M1)
2.25 > k (A1)
crosses the x-axis if k = 1 or k = 2 (A1)(A1)
2
probability = (A1) (C6)
7
Method 2
y

(M2)(M1)
Note: Award (M2) for one (relevant) curve;
(M1) for a second one.
k = 1 or k = 2 (G1)(G1)
2
probability = (A1) (C6)
7
[6]

119.
sketch relation letters
(i) A F (A1)(A1) (C2)
(ii) C E (A1)(A1) (C2)
(iii) B D (A1)(A1) (C2)
[6]

120. (a) Since the vertex is at (3, 1)


h=3 (A1)
k=1 (A1) 2

(b) (5, 9) is on the graph  9 = a(5 – 3)2 + 1 (M1)


= 4a + 1 (A1)
=>9–1=4a=8 (A1)
=>a=2 (AG) 3
Note: Award (M1)(A1)(A0) for using a reverse proof, ie
substituting for a, h, k and showing that (5, 9) is on the graph.
(c) y = 2(x – 3)2 + 1 (M1)
= 2x2 – 12x + 19 (AG) 1

(d) (i) Graph has equation y = 2x2 – 12x + 19


dy
= 4x – 12 (A1)
dx
(ii) At point (5, 9), gradient = 4(5) – 12 = 8 (A1)
(iii) Equation: y – 9 = 8(x – 5) (M1)(A1)
8x – y – 31 = 0
OR
9 = 8(5) + c (M1)
c = –31
y = 8x – 31 (A1) 4
[10]

121. One solution  discriminant = 0 (M2)


2
3 – 4k = 0 (A2)
9 = 4k
9 1 
k =   2 , 2.25  (A2) (C6)
4 4 
Note: If candidates correctly solve an incorrect equation,
award M2 A0 A2(ft), if they have the first line or equivalent,
otherwise award no marks.
[6]

122. (a) (i) p=2 (A2) (C2)


q
(ii) 10 = (or equivalent) (M1)
3–2
q = 10 (A1) (C2)

(b) Reflection, in x-axis (A1)(A1) (C2)


[6]

123. (a) Initial mass  t = 0 (A1)


mass = 4 (A1) (C2)
(b) 1.5 = 4e–0.2t (or 0.375 = e–0.2t) (M2)
ln 0.375 = –0.2t (M1)
t = 4.90 hours (A1) (C4)
[6]

124. (a) a = 3, b = 4 (A1)


f (x) = (x – 3)2 + 4 A1 (C2)

(b) y = (x – 3)2 + 4
METHOD 1
x = (y – 3)2 + 4 (M1)
x – 4 = (y – 3)2
x4 = y – 3 (M1)
y = x4 + 3 (A1) 3
METHOD 2
y – 4 = (x – 3)2 (M1)
y4 =x–3 (M1)
y4 +3=x
y= x4 + 3
 f –1(x) = x4 + 3 (A1) 3

(c) x4 (A1)(C1)


[6]

125. (a) f (3) = 23 (M1)


3
2
(g ° f ) (3) = (M1)
2 –2
3

8
= (A1)
6
4
(g ° f ) (3) = (C3)
3
y
(b) x= (M1)
y–2
x (y –2) = y  y (x – 1) = 2x
2x
y= (A1)
x – 1
10
y= = 2.5 (A1) (C3)
5 – 1
Note: Interchanging x and y may take place at any time.
[6]

126. log27 (x(x – 0.4)) = l (M1)(A1)


x2 – 0.4x = 27 (M1)
x = 5.4 or x = –5 (G2)
x = 5.4 (A1) (C6)
Note: Award (C5) for giving both roots.
[6]

127. (a) (i) h=–1 (A2) (C2)


(ii) k=2 (A1) (C1)

(b) a(l + l)2 + 2 = 0 (M1)(A1)


a = –0.5 (A1) (C3)
[6]

1
1  1 -kx 
0 e dx   k e  0
128. (a) -kx (A1)

1
= – (e–k – e0) (A1)
k
1 –k
= – (e – 1) (A1)
k
1
= – (1 – e–k) (AG) 3
k
(b) k = 0.5
(i)
y

(0,1)
1

x
–1 0 1 2 3

(A2)
Note: Award (A1) for shape, and (A1) for the point (0,1).

(ii) Shading (see graph) (A1)

1
e
(iii) Area = -kx dx for k = 0.5 (M1)
0

1
= (1 – e0.5)
0 .5
= 0.787 (3 sf) (A1)
OR
Area = 0.787 (3 sf) (G2) 5

dy
(c) (i) = –ke–kx (A1)
dx
(ii) x=1 y = 0.8  0.8 = e –k (A1)
ln 0.8 = –k
k = 0.223 (A1)

dy
(iii) At x = 1 = –0.223e–0.223 (M1)
dx
= –0.179 (accept –0.178) (A1)
OR
dy
= –0.178 or – 0.179 (G2) 5
dx
[13]
129. (a) 2x2 – 8x + 5 = 2(x2 – 4x + 4) + 5 – 8 (M1)
= 2(x – 2)2 – 3 (A1)(A1)(A1)
=> a = 2, p = 2, q = –3 (C4)

(b) Minimum value of 2(x – 2)2 = 0 (or minimum value occurs when x = 2) (Ml)
 Minimum value of f (x) = –3 (A1) (C2)
OR
Minimum value occurs at (2, –3) (M1)(A1) (C2)
[6]

130. METHOD 1
Using gdc equation solver for
ex + 2x – 5 = 0, (M1)(A1)
x = 1.0587 (G3)
= 1.059 (4 sf) (A1) (C6)
METHOD 2
Using gdc to graph y = ex and y = 5 – 2x and find x-coordinate at point of
intersection. (M1)
y
y=e

y = 5 – 2x

0 x

(M1)
x = 1.0587 (G3)
= 1.059 (4 sf) (A1) (C6)
[6]

6 x
131. (a) y=
2
6 y
=> x = (M1)
2
=> y = 6 – 2x = g–1(x) (A1) (C2)
(b) (f ° g–1) (x) = 4[(6 – 2x) –1] = 4(5 –2x) = 20 – 8x (M1)(A1)
20 – 8x = 4 => 8x = 16 (M1)
=> x = 2 (A1) (C4)
[6]

15
132. 15% per annum = % = 1.25% per month (M1)(A1)
12
Total value of investment after n months, 1000(1.0125)n > 3000 (M1)
=> (1.0125)n > 3
log 3
n log (1.0125) > log (3) => n > (M1)
log 1.0125
Whole number of months required so n = 89 months. (A1) (C6)
Notes: Award (C5) for the answer of 90 months obtained from
using n – 1 instead of n to set up the equation.
Award (C2) for the answer 161 months obtained by using
simple interest.
Award (C1) for the answer 160 months obtained by using
simple interest.
[6]

133. (a) g (x) = 2 f (x – l)


x 0 1 2 3
x–1 –1 0 1 2
f (x – 1) 3 2 0 1
g (0) = 2 f (–1) = 6 (A1) (C1)
g (1) = 2 f (0) = 4 (A1) (C1)
g (2) = 2 f (l) = 0 (A1) (C1)
g (3) = 2 f (2) = 2 (A1) (C1)
(b) Graph passing through (0, 6), (1, 4), (2, 0), (3, 2) (A1)
Correct shape. (A1)
y
8

6
E
5

4
A
3
B
2
D
1
C
–4 –3 –2 –1 0 1 2 3 4 5 x

–1

–2
(C2)
[6]

134. (a) At A, x = 0 => y = sin (e0) = sin (1) (M1)


=> coordinates of A = (0,0.841) (A1)
OR
A(0, 0.841) (G2) 2

(b) sin (ex) = 0 => ex =  (M1)


=> x = ln  (or k = π) (A1)
OR
x = ln  (or k = π) (A2) 2
(c) (i) Maximum value of sin function = 1 (A1)
dy
(ii) = ex cos (ex) (A1)(A1)
dx
Note: Award (A1) for cos (ex) and (A1) for ex.

dy
(iii) = 0 at a maximum (R1)
dx
ex cos (ex) = 0
=> ex = 0 (impossible) or cos (ex) = 0 (M1)
π π
=> ex = => x = ln (A1)(AG) 6
2 2
ln 
(d) (i) Area = 
0
sin (e x ) dx (A1)(A1)(A1)

Note: Award (A1) for 0, (A1) for ln π, (A1) for sin (ex).
(ii) Integral = 0.90585 = 0.906 (3 sf) (G2) 5

(e)
y = x3

(M1)
At P, x = 0.87656 = 0.877 (3 sf) (G2) 3
[18]

135. (a) x1 = –0.790 and x1 = 1.79 (A1)(A1) 2


(b) (i) a = –0.790 (A1)
(ii) b = 1.79 (A1) 2
(c) When x is large, the value of g (x) becomes much larger than the value
of 2x3. (R1)
2 x3
As a consequence, the value of approaches 0.
g ( x)
Thus f (x) approaches 1. (R1)(AG) 2

(d) (i) At A, x = –1 (A1)


(ii) At B, x = 1 (A1) 2

(e) Horizontal point of inflexion (A2)


OR
Gradient of tangent = 0 => f '(x) = 0 (A1)
Point of inflexion => f "(x) = 0 (A1) 2
[10]

136. y = (x +2)(x – 3) (M1)


= x2 – x – 6 (A1)
Therefore, 0 = 4 – 2p + q (A1)(A1)(C2)(C2)
OR
y = x2 – x – 6 (C3)
OR
0 = 4 – 2p + q (A1)
0 = 9 + 3p + q (A1)
p = –1, q = –6 (A1)(A1)(C2)(C2)
[4]

15.2
137. (a) = 14.8 million (M1)(A1) (C2)
1.027

15.2
(b) = 13.3 million (M1)(A1) (C2)
(1.027 ) 5
OR
14.8
= 13.3 million (M1)(A1) (C2)
(1.027 ) 4
[4]
138. (a)
y
2

x
–2 –1 0 1

–1

–2
(A1)(A1)(C1)(C1)

(b) x = –1.29 (A2) (C2)


[4]

139. 3  2x = 5 (M1)
3 – 2x = 25 (A1)
–2x = 22 (A1)
x = –11 (A1) (C4)

OR
Let y = 3  2 x
 y2 = 3 – 2x (M1)
3  y2
x= (A1)
2
3  x2
 f –1(x) =
2
3  25
 f –1(5) = (M1)
2
= –11 (A1) (C4)
[4]
140. (a)
y

3.5

2.5

1.5

1
(1, 12 )
0.5

–2 –1 0 1 2 3 x
–0.5

–1

–1.5

–2

–2.5 (A2) (C2)

 3
(b) Minimum: 1,  (A1) (C1)
 2
Maximum: (2, 2) (A1) (C1)
[4]

141. (a) Value = 1500(1.0525)3 (M1)


= 1748.87 (A1)
= 1749 (nearest franc) (A1) 3

(b) 3000 = 1500(1.0525)t  2 = 1.0525t (M1)


log 2
t= = 13.546 (A1)
log 1.0525
It takes 14 years. (A1) 3

(c) 3000 = 1500(1 +r)10 or 2(1 +r)10 (M1)


 10 2 = 1 + r or log 2 = 10 log (1 + r) (M1)
log 2
r= 2 –1
10
or r= 10 10 –1 (A1)
r = 0.0718 [or 7.18%] (A1) 4
[10]
142. (a) (i)
y y = sin (1+sinx)
1

x
1 2 3 4 5 6 (A4)
Notes: Only a rough sketch of the graph is required (no scales
necessary).
Award (A1) for any one (local) maximum.

Award (A1) for the minimum at , (A1) for the second
2
minimum.

(ii) Maximum/minimum points at:


0.6075, 1.571, 2.534, 4.712 (G1)(G1)(G1)(G1)(A1) 9
Note: Award the (A1) if all four answers are correct to 4 sf.

(b) (i) See graph (A1)



4.712
(ii)
0
2 sin (1 + sin x)dx or  0
sin (1 + sin x)dx (A2)

(iii) 3.517 (G2) 5

(c) For all x, –1 ≤ sin x ≤ 1; hence 0 ≤ 1 + sin x ≤2. (R1)


On the interval [0, 2] sin x ≥ 0; hence sin (1 +sin x) ≥ 0 (R1) 2
[16]

143. (a) (i) AP = ( x  8) 2  (10  6) 2  x 2  16 x  80 (M1) (AG)


(ii) OP = ( x  0) 2  (10  0) 2  x 2  100 (A1) 2
AP 2  OP 2  OA 2
(b) cos OP̂A  (M1)
2AP  OP
( x  16 x  80)  ( x 2  100 )  (8 2  6 2 )
2
= (M1)
2 x 2  16 x  80 x 2  100
2 x 2  16 x  80
= (M1)
2 x 2  16 x  80 x 2  100
x 2  8 x  40
cos OP̂A  (AG) 3
{( x 2  16 x  80)( x 2  100 )}

(c) For x = 8, cos OP̂A = 0.780869 (M1)


arccos 0.780869 = 38.7° (3 sf) (A1)
OR
8
tan OP̂A  (M1)
10
OP̂A = arctan (0.8) = 38.7° (3 sf) (A1) 2

(d) OP̂A = 60°  cos OP̂A = 0.5


x 2  8 x  40
0.5 = (M1)
{( x 2  16 x  80)( x 2  100)}
2x2 – 16x + 80 – {( x 2  16 x  80)( x 2  100 )} = 0 (M1)
x = 5.63 (G2) 4

(e) (i) f (x) = 1 when cos OP̂A = 1 (R1)


hence, when OP̂A = 0. (R1)
This occurs when the points O, A, P are collinear. (R1)

3x
(ii) The line (OA) has equation y = (M1)
4
40
When y = 10, x = (= 13 13 ) (A1)
3
OR
40
x= (= 13 13 ) (G2) 5
3
Note: Award (G1) for 13.3.
[16]

144.

8
y = x2

6 y = 5–3(x–4)
2

–2 0 2 4 6
q=5 (A1) (C1)
k = 3, p = 4 (A3) (C3)
[4]
145. METHOD 1
1 1
log9 81 + log9   + log9 3 = 2 – 1 + (M1)
9 2
3
 = log9 x (A1)
2
3
x= 92 (M1)
 x = 27 (A1) (C4)

METHOD 2
1  1 
log 81 + log9   + log9 3 = log9 81 3 (M2)
9  9 
= log9 27 (A1)
 x = 27 (A1) (C4)
[4]
146.
y

n–1

x
m–1 m

x
(a) y = 0  x = 0 or sin =0 (M1)
3
x
 = 0, 
3
 x = 0, 3
m = 10 (A1)
OR
From a graphic display calculator
y = 0  x = 9.43 (or x between 9 and 10) (M1)
 m = 10 (A1) (C2)

(b) ymax = 5.46 (or between 5 and 6) (M1)


n=6 (A1) (C2)
[4]

147. f (x) = 2e3x. Let x = 2e3y (M1)


x
 = e3y (A1)
2
 x
 ln   = 3y (A1)
2
1  x
y= ln   (A1)
3 2
1  x
that is f –1(x) = ln   (C4)
3 2
[4]

148. (a) (i) a = –3 (A1)


(ii) b=5 (A1) 2

(b) (i) f (x) = –3x2 + 4x + 15 (A2)


(ii) –3x2 + 4x + 15 = 0
–(3x + 5)(x – 3) = 0 (M1)
5
x = – or x = 3 (A1)(A1)
3
OR
5
x = – or x = 3 (G3)
3
(iii) x = 3  f (3) = –33 + 2(32) + 15(3) (M1)
= –27 + 18 + 45 =36 (A1)
OR
f (3) = 36 (G2) 7

(c) (i) f (x) = 15 at x = 0 (M1)


Line through (0, 0) of gradient 15
 y = 15x (A1)
OR
y = 15x (G2)

(ii) –x3 + 2x2 + 15x = 15x (M1)


 –x3 + 2x2 = 0
 –x2 (x – 2) = 0
x=2 (A1)
OR
x=2 (G2) 4

(d) Area =115 (3 sf) (G2)


OR
5
6  x4 x3 x2 
Area =  0
( x3  2 x 2  15 x)dx  
 4
 2  15 
3 2 0
(M1)

1375
= = 115 (3 sf) (A1) 2
12
[15]
149. (a) f (x) = x2 – 6x + 14
f (x) = x2 – 6x + 9 – 9 + 14 (M1)
f (x) = (x – 3)2 + 5 (M1)

(b) Vertex is (3, 5) (A1)(A1)


[4]

150. (a) At t = 2, N = 10e0.4(2) (M1)


N = 22.3 (3 sf)
Number of leopards = 22 (A1)

(b) If N = 100, then solve 100 = 100e0.4t


10 = e04t
ln 10 = 0.4t
ln 10
t= ~ 5.76 years (3 sf) (A1)
0 .4
[4]

151. (a) Let y = f (x) = x  1


Exchange x and y and solve for y.
x = y 1 (M1)
x2 = y + 1
f –1(x) = x2 – 1 (or y = x2 – 1) (A1)

(b) Domain of f –1(x) = range of f (x) (M1)


x>0 (A1)
[4]
152. (a) Correct vertical shift (A1)
Coordinates of the images (see diagram) (A1) (A1)
y

B(5, 4)

B’(3, 1)

A(–5, –4)

A’(–7, –7)

(b) Asymptote: y = –3 (A1)


[4]
153. (a)
y

–1 –0.5 0 0.5 1 1.5 2 x

Note: Award (A2) for sine curve, (A1) for parabola.

(b) x = 0.876726 (6 sf) (M1)(A1)


Note: Candidates may use the ‘intersect’ function at the point
of intersection of the curves, or find the zero of x2 – sin x = 0.
[4]
154. (a)
y

4 MAXIMUM
{0.5<
(A1)
x<1
3.5<y<4 POINT

2
{
integers (A1)
1 on axis

x
1 2 3 4 5
LEFT RIGHT
(A1) 3.5<x<4 (A1)
–1 INTERCEPT 3<x<3.5 INTERCEPT
(A1) {
3.2<x<3.6 MINIMUM
–0.2<y <0 POINT
5

(b)  is a solution if and only if  +  cos  = 0. (M1)


Now  +  cos  =  + (–1) (A1)
=0 (A1) 3

(c) By using appropriate calculator functions x = 3.696 722 9... (M1)


 x = 3.69672 (6sf) (A1) 2

(d) See graph: (A1)


π
 (π  x cos x)dx
0
(A1) 2
π
(e) EITHER  (π  x cos x)dx
0
= 7.86960 (6 sf) (A3) 3
Note: This answer assumes appropriate use of a calculator eg
 fnInt(Y1 , X , 0, π)  7.869604401
‘fnInt’: 
with Y1  π  x cos x
π
 (π  x cos x)dx  [πx  x sin x  cos x]
π
OR 0
0

= ( – 0) + ( sin  – 0 × sin 0) + (cos  – cos 0) (A1)


= 2 + 0 + –2 = 7.86960 (6 sf) (A1) 3
[15]

155. (a) When t = 0, (M1)


h = 2 + 20 × 0 – 5 × 02 = 2 h=2 (A1) 2

(b) When t = 1, (M1)


h = 2 + 20 × 1 – 5 × 12 (A1)
= 17 (AG) 2

(c) (i) h = 17  17 = 2 + 20t – 5t2 (M1)


(ii) 5t2 – 20t + 15 = 0 (M1)
 5(t2 – 4t + 3) = 0
 (t – 3)(t – 1) = 0 (M1)
Note: Award (M1) for factorizing or using the formula
 t = 3 or 1 (A1) 4
Note: Award (A1) for t = 3

(d) (i) h = 2 + 20t – 5t2


dh
 = 0 + 20 – 10t
dt
= 20 – 10t (A1)(A1)

(ii) t=0 (M0)


dh
 = 20 – 10 × 0 = 20 (A1)
dt

dh
(iii) =0 (M1)
dt
 20 – 10t = 0  t = 2 (A1)

(iv) t=2 (M1)


 h = 2 + 20 × 2 – 5 × 22 = 22  h = 22 (A1) 7
[15]

156. (a) f –1(2)  3x + 5 = 2 (M1)


x = –1 (A1) (C2)

(b) g(f (–4) = g(–12 + 5)


= g(–7) (A1)
= 2(1 + 7)
= 16 (A1) (C2)
[4]
π
 (π  x cos x)dx  [πx  x sin x  cos x] 4x2 + 4kx + 9 = 0
π
OR 0
0
157.
Only one solution  b – 4ac = 0
2
(M1)
16k2 – 4(4)(9) = 0 (A1)
k2 = 9
k = 3 (A1)
But given k > 0, k = 3 (A1) (C4)
OR
One solution  (4x2 + 4kx + 9) is a perfect square (M1)
4x2 + 4kx + 9 = (2x  3)2 by inspection (A2)
given k > 0, k = 3 (A1) (C4)
[4]

158. (a) C has equation x = 2y (A1)


ie y = log2 x (A1) (C2)
OR Equation of B is x = log2y (A1)
Therefore equation of C is y = log2 x (A1) (C2)

(b) Cuts x-axis  log2 x = 0


x = 2° (A1)
x=1
Point is (1, 0) (A1) (C2)
[4]
159. (a) y = (x – 1)2 (A2) (C2)
(b) y = 4(x – 1)2 (A1) (C1)
(c) y = 4(x – 1)2 + 3 (A1) (C1)
Note: Do not penalize if these are correctly expanded.
[4]

 π
160. From sketch of graph y = 4 sin  3x   (M2)
 2
or by observing sin   1.
k > 4, k < –4 (A1)(A1)(C2)(C2)
4

0
–2 – 0  2

–1

–2

–3

–4
[4]

161. Graph of quadratic function.


Expression + – 0
a (A1) (C1)
c (A1) (C1)
b2 – 4ac (A1) (C1)
b (A1) (C1)
[4]
162. Note: A reminder that a candidate is penalized only once in this question
for not giving answers to 3 sf
(a) V(5) = 10000 × (0.9335) = 7069.8 …
= 7070 (3 sf) (A1) 1

(b) We want t when V = 5000 (M1)


5000 = 10000 × (0.933)t
0.5 = 0.933t (A1)
log (0.5)  ln (0.5) 
 t  or 
log (0.933)  ln (0.933) 
9.9949 = t
After 10 minutes 0 seconds, to nearest second (or 600 seconds). (A1) 3

(c) 0.05 = 0.933t (M1)


log (0.05)
= t = 43.197 minutes (M1)(A1)
log (0.933)
 3/4 hour (AG) 3

(d) (i) 10000 – 10000(0.933)0.001 = 0.693 (A1)


dV
(ii) Initial flow rate = where t = 0, (M1)
dt
dV 0.693
 = 693
dt 0.001
= 690 (2 sf) (A1)
OR
dV
= 690 (G2) 3
dt
[10]

163. (a) x2 – 3x – 10 = (x – 5)(x + 2) (M1)(A1) (C2)


(b) x2 – 3x – 10 = 0  (x – 5)(x + 2) = 0 (M1)
 x = 5 or x = –2 (A1) (C2)
[4]
1
164. (a) p=– ,q=2 (A1)(A1) (C2)
2
or vice versa
(b) By symmetry C is midway between p, q (M1)
Note: This (M1) may be gained by implication.
 1 2 3
 x-coordinate is 2  (A1) (C2)
2 4
[4]

165. (a) p=3 (A1) (C1)



(b) Area = 
0
2 3 cos xdx (M1)

= [3 sin x] 02 (A1)
= 3 square units (A1) (C3)
[4]

166. (g ° f ) (x) = 0  2 cos x + 1 = 0 (M1)


1
 cos x = – (A1)
2
2 4
x= , (A1)(A1) (C4)
3 3
Note: Accept 120°, 240°.
[4]
2x  1
167. (a) (i) f (x) =
x3
7
=2+ by division or otherwise (M1)
x3
Therefore as x   f (x)  2 (A1)
 y = 2 is an asymptote (AG)
2x  1
OR lim =2 (M1)(A1)
x  x  3

 y = 2 is an asymptote (AG)
OR make x the subject
yx – 3y = 2x + 1
x(y – 2) = 1 + 3y (M1)
1  3y
x= (A1)
y2
 y = 2 is an asymptote (AG)
Note: Accept inexact methods based on the ratio of the
coefficients of x.

(ii) Asymptote at x = 3 (A1)


(iii) P(3, 2) (A1) 4

1 1 
(b) f (x) = 0  x = –   , 0 (M1)(A1)
2 2 
1  1
x = 0  f (x) = –  0,   (M1)(A1) 4
3  3
Note: These do not have to be in coordinate form.
(c)
y

3 x

(A4) 4
Note: Asymptotes (A1)
Intercepts (A1)
“Shape” (A2).

( x  3)(2)  (2 x  1)
(d) f (x) = (M1)
( x  3) 2
7
= (A1)
( x  3) 2
= Slope at any point
Therefore slope when x = 4 is –7 (A1)
And f (4) = 9 ie S(4, 9) (A1)
 Equation of tangent: y – 9 = –7(x – 4) (M1)
7x + y – 37 = 0 (A1) 6

7
(e) at T, = –7 (M1)
( x  3) 2
 (x – 3)2 = 1 (A1)
x – 3 = ±l (A1)
x  4 or 2  S (4, 9)
 (A1)(A1) 5
y  9 or – 5 T (2,  5)

 4  2 9 5
(f) Midpoint [ST] =  , 
 2 2 
= (3, 2)
= point P (A1) 1
[24]
168. (7 – x)(1 + x) = 0 (M1)
 x = 7 or x = –1 (A1)(C1)(C1)
7  1
B: x = = 3; (A1)
2
y = (7 – 3)(l + 3) = 16 (A1) (C2)
[4]

169. (a) I
(b) III
(c) IV
Note: Award (C4) for 3 correct, (C2) for 2 correct, (C1) for 1
correct.
[4]

170. ln (x – 2)  0 since we need to find its square root (M1)(R1)


x–21 (A1)
x3 (A1) (C4)
Note: x > 3: deduct [1 mark] ([2 marks] if no working shown).
[4]

171. 1.023t = 2 (M1)


ln 2
t= (M1)(A1)
ln 1.023
= 30.48...
30 minutes (nearest minute) (A1) (C4)
Note: Do not accept 31 minutes.
[4]
172. x = g–1(f (0.25)) (M1)
= log2 ((0.25)1/2) (A1)
1
= log2   (A1)
2
= –1 (A1)
OR
f –1(x) = x2 (M1)
= (f –1 ° g)(x) = f –1(2x) = 22x (M1)
Therefore, 22x = 0.25 = 2–2 (M1)
 2x = –2
 x = –1 (A1) (C4)
[4]

173. METHOD 1
using double-angle identity (seen anywhere) A1
e.g. sin 2x = 2sin x cos x, 2cos x = 2sin x cos x

evidence of valid attempt to solve equation (M1)


e.g. 0 = 2sin x cos x – 2cos x, 2cos x (1– sin x) = 0
cos x = 0, sin x =1 A1A1
 3 5
x  ,x  ,x  A1A1A1 N4
2 2 2
[7]
METHOD 2

A1A1M1A1
Notes: Award A1 for sketch of sin 2x, A1 for a sketch of 2 cos x,
M1 for at least one intersection point seen, and A1 for 3
approximately correct intersection points. Accept sketches
drawn outside [0, 3π], even those with more than 3
intersections.
 3 5
x  ,x  ,x A1A1A1 N4
2 2 2
[7]

174. (a) (i) 100 (metres) A1 N1

(ii) 50 (metres) A1 N1 2

(b) (i) identifying symmetry with h(2) = 9.5 (M1)


subtraction A1
e.g. 100 – h(2), 100 – 9.5
h(8) = 90.5 AG N0

(ii) recognizing period (M1)


e.g. h(21) = h(1)
h(21) = 2.4 A1 N2 4
(c)

A1A1A1 N3 3
Note: Award A1 for end points (0, 0) and (40, 0), A1 for range
0 ≤ h ≤ 100, A1 for approximately correct sinusoidal shape,
with two cycles

(d) evidence of a quotient involving 20, 2π or 360º to find b (M1)


2π 360
e.g.  20, b 
b 20
2π  π 
b  
20  10 
(accept b =18 if working in degrees) A1 N2
a = –50, c = 50 A2A1 N3 5
[14]

175. (a) evidence of choosing cosine rule (M1)


e.g. a2 + b2 – 2ab cos C
correct substitution A1
e.g. 72 + 92 – 2(7)(9) cos 120º
AC =13.9 (= 193 ) A1 N2 3
(b) METHOD 1
evidence of choosing sine rule (M1)
sin Aˆ sin Bˆ
e.g. 
BC AC
correct substitution A1
sin Aˆ sin 120
e.g. 
9 13.9
  34.1 A1 N2 3

METHOD 2
evidence of choosing cosine rule (M1)
ˆ AB2  AC 2  BC 2
e.g. cos A 
2ABAC
correct substitution A1
7 2  13.9 2  9 2
e.g. cos Aˆ 
27 13.9 
  34.1 A1 N2 3
[6]

176. (a) METHOD 1


evidence of recognizing the amplitude is the radius (M1)
e.g. amplitude is half the diameter
8
a A1
2
a=4 AG N0 2

METHOD 2
evidence of recognizing the maximum height (M1)
e.g. h = 6, a sin bt + 2 = 6
correct reasoning
e.g. a sin bt = 4 and sin bt has amplitude of 1 A1
a=4 AG N0 2
(b) METHOD 1
period = 30 (A1)
2
b A1
30

b AG N0 2
15

METHOD 2
correct equation (A1)
e.g. 2 = 4 sin 30b + 2, sin 30b = 0
30b = 2π A1

b AG N0 2
15

(c) recognizing h′(t) = –0.5 (seen anywhere) R1


attempting to solve (M1)
e.g. sketch of h′, finding h′
correct work involving h′ A2
4π π 
e.g. sketch of h′ showing intersection, –0.5 = cos t 
15  15 
t = 10.6, t = 19.4 A1A1 N3 6

(d) METHOD 1
valid reasoning for their conclusion (seen anywhere) R1
e.g. h(t) < 0 so underwater; h(t) > 0 so not underwater
evidence of substituting into h (M1)
19.4π
e.g. h(19.4), 4 sin 2
15
correct calculation A1
e.g. h(19.4) = –1.19
correct statement A1 N0 4
e.g. the bucket is underwater, yes
METHOD 2
valid reasoning for their conclusion (seen anywhere) R1
e.g. h(t) < 0 so underwater; h(t) > 0 so not underwater
evidence of valid approach (M1)
e.g. solving h(t) = 0, graph showing region below x-axis
correct roots A1
e.g. 17.5, 27.5
correct statement A1 N0 4
e.g. the bucket is underwater, yes
[14]

177. (a)

A1A1A1 N3 3
Note: Award A1 for labelling 4° with horizontal, A1 for
labelling [AU] 25 metres, A1 for drawing [TU].

(b) TÂU = 86º (A1)


evidence of choosing cosine rule (M1)
correct substitution A1
e.g. x2 = 252 + 362 – 2(25)(36) cos 86º
x = 42.4 A1 N3 4
[7]
178. (a) evidence of finding height, h (A1)
h
e.g. sin θ = , 2 sin θ
2
evidence of finding base of triangle, b (A1)
b
e.g. cos θ = , 2 cos θ
2
attempt to substitute valid values into a formula for the area
of the window (M1)
e.g. two triangles plus rectangle, trapezium area formula
correct expression (must be in terms of θ) A1
1 
e.g. 2  2 cos   2 sin    2  2 sin  , 2 sin  2  2  4 cos  
1
2  2
attempt to replace 2sinθ cosθ by sin 2θ M1
e.g. 4 sin θ + 2(2 sin θ cos θ)
y = 4 sin θ + 2 sin 2θ AG N0 5

(b) correct equation A1


e.g. y = 5, 4 sin θ + 2 sin 2θ = 5
evidence of attempt to solve (M1)
e.g. a sketch, 4 sin θ + 2 sin θ – 5 = 0
θ = 0.856 (49.0º), θ = 1.25 (71.4º) A1A1 N3 4


(c) recognition that lower area value occurs at θ = (M1)
2

finding value of area at θ = (M1)
2
  
e.g. 4 sin    2 sin  2   , draw square
2  2
A=4 (A1)
recognition that maximum value of y is needed (M1)
A = 5.19615… (A1)
4 < A < 5.20 (accept 4 < A < 5.19) A2 N5 7
[16]
3 3 
179. (a) tan θ =  do not accept x  A1 N1
4 4 

3 4
(b) (i) sin θ = , cos θ = (A1)(A1)
5 5
correct substitution A1
 3  4 
e.g. sin 2θ = 2   
 5  5 
24
sin 2θ = A1 N3
25

(ii) correct substitution A1


2 2 2
3  4 3
e.g. cos 2θ = 1 – 2   ,     
5  5 5
7
cos 2θ = A1 N1
25
[7]

180. (a) (i) evidence of finding the amplitude (M1)


73
e.g. , amplitude = 5
2
p = –5 A1 N2

(ii) period = 8 (A1)


 2π π 
q = 0.785     A1 N2
 8 4

73
(iii) r= (A1)
2
r=2 A1 N2

(b) k = –3 (accept y = –3) A1 N1


[7]

181. (a) appropriate approach (M1)


e.g. 6 = 8θ
AÔC = 0.75 A1 N2

(b) evidence of substitution into formula for area of triangle (M1)


1
e.g. area = × 8 × 8 × sin(0.75)
2
area = 21.8… (A1)
evidence of substitution into formula for area of sector (M1)
1
e.g. area = × 64 × 0.75
2
area of sector = 24 (A1)
evidence of substituting areas (M1)
1 2 1
e.g. r   ab sin C , area of sector – area of triangle
2 2
area of shaded region = 2.19 cm2 A1 N4

(c) attempt to set up an equation for area of sector (M1)


1
e.g. 45 = × 82 × θ
2
CÔE = 1.40625 (1.41 to 3 sf) A1 N2

(d) METHOD 1
attempting to find angle EOF (M1)
e.g. π – 0.75 – 1.41
EÔF = 0.985 (seen anywhere) A1
evidence of choosing cosine rule (M1)
correct substitution A1
e.g. EF = 8 2  8 2  2  8  8  cos 0.985
EF = 7.57 cm A1 N3

METHOD 2
attempting to find angles that are needed (M1)
e.g. angle EOF and angle OEF
EÔF = 0.9853... and OÊF (or OF̂E) = 1.078... A1
evidence of choosing sine rule (M1)
correct substitution (A1)
EF 8
e.g. 
sin0.985 sin 1.08
EF = 7.57 cm A1 N3
METHOD 3
attempting to find angle EOF (M1)
e.g. π – 0.75 – 1.41
EÔF = 0.985 (seen anywhere) A1
evidence of using half of triangle EOF (M1)
0.985
e.g. x = 8 sin
2
correct calculation A1
e.g. x = 3.78
EF = 7.57 cm A1 N3
[15]

182. (a) correct substitution in l = rθ (A1)


 1
e.g. 10 × , × 2π × 10
3 6
20 π  10 π 
arc length =   A1 N2
6  3 

1 π  100 π 
(b) area of large sector = 10 2     (A1)
2 3 6 
1 2 π  64 π 
area of small sector =  8     (A1)
2 3 6 
evidence of valid approach (seen anywhere) M1
1 π
e.g. subtracting areas of two sectors,  (10  8 )
2 2
2 3
 36π 
area shaded = 6π  accept , etc. A1 N3
 6 
[6]

183. (a) attempt to substitute 1 – 2 sin2 θ for cos 2θ (M1)


correct substitution A1
e.g. 4 – (1 – 2 sin2 θ) + 5 sin θ
4 – cos 2θ + 5 sin θ = 2 sin2 θ + 5 sin θ + 3 AG N0
(b) evidence of appropriate approach to solve (M1)
e.g. factorizing, quadratic formula
correct working A1
5 1
e.g. (2 sin θ + 3)(sin θ + 1), (2x + 3)(x + 1) = 0, sin x =
4
 3
correct solution sin θ = –1  do not penalise for including sin   –  (A1)
 2

θ= A2 N3
2
[7]

184. (a) (i) correct approach A1


 5 1
e.g. OC  OA,     
 2  0
 4
AC    AG N0
 2

(ii) appropriate approach (M1)


 4  1
e.g. D – B,      , move 3 to the right and 6 down
  1  5 
 3 
BD    A1 N2
  6

(iii) finding the scalar product A1


e.g. 4(3) + 2(–6), 12 – 12
valid reasoning R1
e.g. 4(3) + 2(–6) = 0, scalar product is zero
AC is perpendicular to BD AG N0

(b) (i) correct “position” vector for u; “direction” vector for v A1A1 N2
 5 1  4   2
e.g. u =  , u   ; v   , v   
 2  0  2  1 
 5   4
accept in equation e.g.    t  
 2   2
(ii) any correct equation in the form r = a + tb, where b = BD
 1   3   x   4    1
e.g. r =    t  ,       t   A2 N2
 5    6   y    1  2 

(c) METHOD 1
substitute (3, k) into equation for (AC) or (BD) (M1)
e.g. 3 = 1 + 4s, 3 = 1 + 3t
value of t or s A1
1 1 2 1
e.g. s = , , t  , ,
2 2 3 3
substituting A1
1
e.g. k = 0 + ( 2) ,
2
k=1 AG N0

METHOD 2
setting up two equations (M1)
e.g. 1 + 4s = 4 + 3t, 2s = –1 – 6t; setting vector equations of lines equal
value of t or s A1
1 1 2 1
e.g. s = , , t  ,
2 2 3 3
substituting A1
 4  1 3 
e.g. r =      ,
  1 3   6 
k=1 AG N0

 1 
(d) PD    (A1)
  2
PD  2 2  12 ( 5 ) (A1)

AC  4 2  2 2 ( 20 ) (A1)

1  1 
area =  AC  PD    20  5  M1
2  2 
=5 A1 N4
[17]
185. (a) correct substitution into the formula for the area of a triangle A1
1 1
e.g. × 5 × 13.6 × sin C = 20, × 5 × h = 20
2 2
attempt to solve (M1)
8
e.g. sin C = 0.5882... , sin C =
13.6
Ĉ = 36.031...° (0.6288… radians) (A1)
AĈB = 144° (2.51 radians) A1 N3

(b) evidence of choosing the cosine rule (M1)


correct substitution A1
e.g. (AB)2 = 52 + 13.62 – 2(5)(13.6)cos143.968...
AB = 17.9 A1 N2
[7]

186. (a) valid approach (M1)


e.g. 15 mins is half way, top of the wheel, d + 1
height = 101 (metres) A1 N2

(b) evidence of identifying rotation angle after 6 minutes A1


2π 1
e.g. , of a rotation, 72°
5 5
evidence of appropriate approach (M1)
e.g. drawing a right triangle and using cosine ratio
correct working (seen anywhere) A1
2π x
e.g. cos  , 15.4(508...)
5 50
evidence of appropriate method M1
e.g. height = radius + 1 – 15.45...
height 35.5 (metres) (accept 35.6) A1 N2
(c) METHOD 1

evidence of substituting into b = (M1)
period
correct substitution

e.g. period = 30 minutes, b = A1
30
π
b = 0.209   A1 N2
 15 
substituting into h(t) (M1)
e.g. h(0) = 1, h(15) = 101
correct substitution A1
 π 
1 = 50 sin   c  + 51
 15 
c = 7.5 A1 N2

METHOD 2
evidence of setting up a system of equations (M1)
two correct equations
e.g. 1 = 50 sin b(0 – c) + 51, 101 = 50 sin b(15 – c) + 51 A1A1
attempt to solve simultaneously (M1)
e.g. evidence of combining two equations
π
b = 0.209   , c = 7.5 A1A1 N2N2
 15 

(d) evidence of solving h(t) = 96 (M1)


e.g. equation, graph
t = 12.8 (minutes) A2 N3
[16]

187. (a) (i) x = 3 cos θ A1 N1

(ii) y = 3 sin θ A1 N1
(b) finding area (M1)
1
e.g. A = 2x × 2y, A = 8 × bh
2
substituting A1
1
e.g. A = 4 × 3 sin θ × 3 cos θ, 8 × × 3 cos θ × 3 sin θ
2
A = 18(2 sin θ cos θ) A1
A = 18 sin 2θ AG N0

dA
(c) (i) = 36 cos 2θ A2 N2
d

(ii) for setting derivative equal to 0 (M1)


dA
e.g. 36 cos 2θ = 0, =0
d
π
2θ = (A1)
2
π
θ= A1 N2
4

(iii) valid reason (seen anywhere) R1


π d2 A
e.g. at ,  0 ; maximum when f″(x) < 0
4 d 2
d2 A
finding second derivative = –72 sin 2θ A1
d 2
π
evidence of substituting M1
4
 π π
e.g. –72 sin  2  ,72 sin  ,72
 4 2
π
θ= produces the maximum area AG N0
4
[13]

188. (a) (i) evidence of approach (M1)


e.g. PQ  PO  OQ , Q – P
  1
 
PQ   2  A1 N2
1
 

 2
 
(ii) PR   2  A1 N1
 4
 

(b) METHOD 1
choosing correct vectors PQ and PR (A1)(A1)
finding PQ PR, PQ , PR (A1) (A1)(A1)

PQ  PR = –2 + 4 + 4 (= 6)
PQ  (1) 2  2 2  12  ( 6 ), PR  2 2  2 2  4 2 ( 24)
substituting into formula for angle between two vectors M1
6
e.g. cos RP̂Q 
6  24
1
simplifying to expression clearly leading to A1
2
6 6 6
e.g. , ,
6 2 6 144 12
1
cos RP̂Q  AG N0
2

METHOD 2
evidence of choosing cosine rule (seen anywhere) (M1)
 3
 
QR   0  A1
 3
 
QR  18 , PQ  6 and PR  24 (A1)(A1)(A1)

( 6 ) 2  ( 24 ) 2  ( 18 ) 2
cos RP̂Q  A1
2 6  24
6  24 – 18  12 
cos RP̂Q    A1
24  24 
1
cos RP̂Q  AG N0
2
(c) (i) METHOD 1
evidence of appropriate approach (M1)
e.g. using sin 2 RP̂Q  cos 2 RP̂Q  1 , diagram
substituting correctly (A1)
2
1
e.g. sin RP̂Q  1 –  
2
3  3 
sin RP̂Q   A1 N3
4  2 

METHOD 2
1
since cos P̂  , P̂  60 (A1)
2
evidence of approach
e.g. drawing a right triangle, finding the missing side (A1)
3
sin P̂  A1 N3
2

(ii) evidence of appropriate approach (M1)


1
e.g. attempt to substitute into ab sin C
2
correct substitution
1 3
e.g. area = 6  24  A1
2 2
area = 3 3 A1 N2
[16]

189. (a) METHOD 1


choosing cosine rule (M1)
substituting correctly A1
e.g. AB = 3.9 2  3.9 2  2(3.9)(3.9) cos 1.8
AB = 6.11(cm) A1 N2

METHOD 2
evidence of approach involving right-angled triangles (M1)
substituting correctly A1
x 1
e.g. sin 0.9 = , AB = 3.9 sin 0.9
3.9 2
AB = 6.11 (cm) A1 N2

METHOD 3
choosing the sine rule (M1)
substituting correctly A1
sin 0.670... sin 1.8
e.g. 
3.9 AB
AB = 6.11 (cm) A1 N2

(b) METHOD 1
reflex AÔB = 2π – 1.8 (= 4.4832) (A2)
1
correct substitution A = (3.9)2(4.4832...) A1
2
2
area = 34.1 (cm ) A1 N2

METHOD 2
finding area of circle A = π(3.9)2 (= 47.78...) (A1)
1
finding area of (minor) sector A = (3.9)2(1.8) (= 13.68...) (A1)
2
subtracting M1
e.g. π(3.9)2 – 0.5(3.9)2(1.8), 47.8 – 13.7
area = 34.1 (cm2) A1 N2

METHOD 3
finding reflex AÔB = 2π – 1.8 (= 4.4832) (A2)
finding proportion of total area of circle A1
2π  1.8 
e.g.  π(3.9) 2 ,  πr 2
2π 2π
area = 34.1 (cm) A1 N2
[7]

190. (a) METHOD 1


evidence of choosing the cosine formula (M1)
correct substitution A1
7 2  7 2  13 2
e.g. cos AĈB 
277
AĈB = 2.38 radians (= 136°) A1 N2
METHOD 2
evidence of appropriate approach involving right-angled triangles (M1)
correct substitution A1
1  6.5
e.g. sin  AĈB  
2  7
AĈB = 2.38 radians (= 136°) A1 N2

(b) METHOD 1
AĈD = π – 2.381 (180 – 136.4) (A1)
evidence of choosing the sine rule in triangle ACD (M1)
correct substitution A1
6.5 7
e.g. 
sin 0.760 ... sinA D̂C
AD̂C = 0.836... (= 47.9...°) A1
CÂD = π – (0.760... + 0.836...) (180 – (43.5... + 47.9...))
= 1.54 (= 88.5°) A1 N3

METHOD 2
1 1 
AB̂C  (π  2.381)  (180  136.4)  (A1)
2 2 
evidence of choosing the sine rule in triangle ABD (M1)
correct substitution A1
6.5 13
e.g. 
sin 0.380 ... sin AD̂C
AD̂C = 0.836... (= 47.9...°) A1
CÂD = π – 0.836... – (π – 2.381...) (= 180 – 47.9... – (180 – 136.4))
= 1.54 (= 88.5°) A1 N3
Note: Two triangles are possible with the given information.
If candidate finds ADˆ C = 2.31 (132°) leading to
CAˆ D = 0.076 (4.35°), award marks as
per markscheme.
[8]
191. (a)

A1A1A1 N3
Note: Award A1 for approximately sinusoidal shape,
A1 for end points approximately correct, (–2π, 4),
(2π, 4) A1 for approximately correct position of graph,
(y-intercept (0, 4) maximum to right of y-axis).

(b) (i) 5 A1 N1

(ii) 2π (6.28) A1 N1

(iii) –0.927 A1 N1

(c) f(x) = 5 sin (x + 0.927) (accept p = 5, q = 1, r = 0.927) A1A1A1 N3

(d) evidence of correct approach (M1)


e.g. max/min, sketch of f′(x) indicating roots

one 3 s.f. value which rounds to one of –5.6, –2.5, 0.64, 3.8 A1 N2

(e) k = –5, k = 5 A1A1 N2


(f) METHOD 1
graphical approach (but must involve derivative functions) M1
e.g.

each curve A1A1


x = 0.511 A2 N2

METHOD 2
1
g′(x) = A1
x 1
f′(x) = 3 cos x – 4 sin x (5 cos(x + 0.927)) A1
evidence of attempt to solve g′(x) = f′(x) M1
x = 0.511 A2 N2
[18]

192. evidence of substituting for cos2x (M1)


evidence of substituting into sin2 x + cos2 x = 1 (M1)
correct equation in terms of cos x (seen anywhere) A1
e.g. 2cos2 x – 1 – 3 cos x – 3 = 1, 2 cos2 x – 3 cos x – 5 = 0
evidence of appropriate approach to solve (M1)
e.g. factorizing, quadratic formula
appropriate working A1
3  49
e.g. (2 cos x – 5)(cos x + 1) = 0, (2x – 5)(x + 1), cos x =
4
correct solutions to the equation
5 5
e.g. cos x = , cos x = –1, x = , x = –1 (A1)
2 2
x=π A1 N4
[7]
193. (a) choosing sine rule (M1)
correct substitution A1
AD 4
e.g. 
sin0.8 sin 0.3
AD = 9.71 (cm) A1 N2

(b) METHOD 1
finding angle OAD = π – 1.1 = (2.04) (seen anywhere) (A1)
choosing cosine rule (M1)
correct substitution A1
e.g. OD2 = 9.712 + 42 – 2 × 9.71 × 4 × cos(π – 1.1)
OD = 12.1 (cm) A1 N3

METHOD 2
finding angle OAD = π – 1.1 = (2.04) (seen anywhere) (A1)
choosing sine rule (M1)
correct substitution A1
OD 9.71 4
e.g.  
sin(π – 1.1) sin 0.8 sin 0.3
OD = 12.1 (cm) A1 N3

(c) correct substitution into area of a sector formula (A1)


e.g. area = 0.5 × 42 × 0.8
area = 6.4 (cm2) A1 N2

(d) substitution into area of triangle formula OAD (M1)


correct substitution A1
1 1
e.g. A = × 4 × 12.1 × sin 0.8, A = × 4 × 9.71 × sin 2.04,
2 2
1
A= × 12.1 × 9.71 × sin 0.3
2
subtracting area of sector OABC from area of triangle OAD (M1)
e.g. area ABCD = 17.3067 – 6.4
area ABCD = 10.9 (cm2) A1 N2
[13]

194. (a) (i) sin 140 = p A1 N1


(ii) cos 70 = q A1 N1

(b) METHOD 1
evidence of using sin2  + cos2  = 1 (M1)
e.g. diagram, 1 p 2
(seen anywhere)
cos 140 =  1  p 2 (A1)
cos 140 =  1  p 2
A1 N2
METHOD 2
evidence of using cos2  = 2 cos2   1 (M1)
cos 140 = 2 cos2 70  1 (A1)
cos 140 = 2( q)2 1 (= 2q2  1) A1 N2

(c) METHOD 1
sin 140  p
tan 140 =  A1 N1
cos 140  1 p 2
METHOD 2
p
tan 140 = A1 N1
2q  1
2

[6]

195. (a) period =  A1 N1

(b)
y
4
3
2
1
0 π π 3π 2π x
–1 2 2
–2
–3
–4
A1A1A1 N3
Note: Award A1 for amplitude of 3, A1 for their
period, A1 for a sine curve passing through
(0, 0) and (0, 2).
(c) evidence of appropriate approach (M1)
e.g. line y = 2 on graph, discussion of number of solutions in
the domain
4 (solutions) A1 N2
[6]

196. (a) evidence of choosing the formula cos2 A = 2 cos2 A  1 (M1)


Note: If they choose another correct formula, do
not award the M1 unless there is evidence
1
of finding sin2 A = 1 .
9
correct substitution A1
2 2
1 8 1
e.g. cos 2A =    , cos 2 A  2     1
 3 9  3
7
cos 2 A   A1 N2
9
(b) METHOD 1
evidence of using sin2 B + cos2 B = 1 (M1)
2
2 5
e.g.    cos B 1,
2
(seen anywhere),
3 9
5  5 
cos B =    (A1)
9  3 

5  5 
cos B =    A1 N2
9  3 
METHOD 2
diagram M1
e.g.

for finding third side equals 5 (A1)


5
cos B =  A1 N2
3
[6]

197. (a) (i) range of f is [1, 1], (1  f (x)  1) A2 N2


(ii) sin3 x = 1  sin x = 1 A1
justification for one solution on [0, 2] R1

e.g. x  , unit circle, sketch of sin x
2
1 solution (seen anywhere) A1 N1

(b) f (x) = 3 sin2 x cos x A2 N2


b
(c) using V    y 2 dx (M1)
a
2

 1

 x  dx
V  0
2  3 sin x cos 2




(A1)


 0
2 3 sin 2 x cos x dx A1

  
  
V   sin 3 x 02     sin 3    sin 3 0   A2
  2 

evidence of using sin 1 and sin 0 = 0 (A1)
2
e.g. (1  0)
V= A1 N1
[14]

198. (a) evidence of using area of a triangle (M1)


1
e.g. A   2  2  sin θ
2
A = 2 sin  A1 N2

(b) METHOD 1
PÔA =    (A1)
area OPA = 2  2  sin   θ  (= 2 sin (  ))
1
A1
2
since sin (  ) = sin  R1
then both triangles have the same area AG N0
METHOD 2
triangle OPA has the same height and the same base as triangle OPB R3
then both triangles have the same area AG N0

 2  2
1 2
(c) area semi-circle = A1
2
area  APB = 2 sin  + 2 sin  (= 4 sin ) A1
S = area of semicircle  area APB (= 2  4 sin ) M1
S = 2( − 2 sin ) AG N0
(d) METHOD 1
attempt to differentiate (M1)
dS
e.g.   4 cos θ

setting derivative equal to 0 (M1)
correct equation A1
e.g. 4 cos  = 0, cos  = 0, 4 cos  = 0

= A1 N3
2
EITHER
evidence of using second derivative (M1)
S() = 4 sin  A1
 
S    4 A1
2

it is a minimum because S    0 R1 N0
2
OR
evidence of using first derivative (M1)

for  < , S () < 0 (may use diagram) A1
2

for  > , S () > 0 (may use diagram) A1
2
it is a minimum since the derivative goes from negative
to positive R1 N0
METHOD 2
2  4 sin  is minimum when 4 sin  is a maximum R3
4 sin  is a maximum when sin  = 1 (A2)

= A3 N3
2

(e) S is greatest when 4 sin  is smallest (or equivalent) (R1)


 = 0 (or ) A1 N2
[18]
199. (a) choosing sine rule (M1)
sin R sin 75
correct substitution  A1
7 10
sin R = 0.676148...
PR̂Q = 42.5 A1 N2

(b) P = 180  75  R
P = 62.5 (A1)
substitution into any correct formula A1
1
e.g. area  PQR =  7 10  sin (their P)
2
2
= 31.0 (cm ) A1 N2
[6]

200. (a) evidence of appropriate approach M1


2
e.g. 3 = r
9
r =13.5 (cm) A1 N1

(b) adding two radii plus 3 (M1)


perimeter = 27+3 (cm) (= 36.4) A1 N2

(c) evidence of appropriate approach M1


1 2 2
e.g. 13.5 
2 9
area = 20.25 (cm2) (= 63.6) A1 N1
[6]

201. (a) (i) 7 A1 N1


(ii) 1 A1 N1
(iii) 10 A1 N1
(b) (i) evidence of appropriate approach M1
18  2
e.g. A 
2
A=8 AG N0

(ii) C = 10 A2 N2

(iii) METHOD 1
period = 12 (A1)
evidence of using B  period = 2 (accept 360) (M1)
2
e.g. 12 =
B
π
B (accept 0.524 or 30) A1 N3
6
METHOD 2
evidence of substituting (M1)
e.g. 10 = 8 cos 3B + 10
simplifying (A1)
 
e.g. cos 3B = 0  3B  
 2
π
B (accept 0.524 or 30) A1 N3
6

(c) correct answers A1A1


e.g. t = 3.52, t = 10.5, between 03:31 and 10:29 (accept 10:30) N2
[11]

sin x
202. (a) changing tan x into A1
cos x
sin x
e.g. sin3 x + cos3 x
cos x
simplifying A1
e.g. sin x (sin2 x + cos2 x), sin3 x + sin x – sin3 x
f(x) = sin x AG N0
(b) recognizing f(2x) = sin 2x, seen anywhere (A1)
evidence of using double angle identity sin (2x) = 2 sin x cos x,
seen anywhere (M1)
2
evidence of using Pythagoras with sin x = M1
3
e.g. sketch of right triangle, sin2 x + cos2 x = 1
5  5 
cos x =  accept (A1)
3  3 

 2  5 
f(2x) = 2    A1
 3  3 
4 5
f(2x) =  AG N0
9
[7]

203. (a) finding AB̂C = 110° (= 1.92 radians) (A1)


evidence of choosing cosine rule (M1)
e.g. AC2 = AB2 + BC2 – 2(AB)(BC) cos AB̂C
correct substitution A1
e.g. AC2 = 252 + 402 – 2(25)(40) cos 110°
AC = 53.9 (km) A1 N3

(b) METHOD 1
correct substitution into the sine rule A1
sin BÂC sin 110 
e.g. 
40 53.9
BÂC = 44.2° A1
bearing = 074° A1 N1
METHOD 2
correct substitution into the cosine rule A1
40 2  25 2  53.9 2
e.g. cos BÂC 
 2(25)(53.9)
BÂC = 44.3° A1
bearing = 074° A1 N1
[7]

204. (a) p = 30 A2 N2
(b) METHOD 1

Period = (M2)
q
π
= (A1)
2
 q=4 A1 N4
METHOD 2
1
Horizontal stretch of scale factor = (M2)
q
1
scale factor = (A1)
4
 q=4 A1 N4
[6]

205. (a) Evidence of using Pythagoras (M1)


e.g. diagram, sin2 x + cos2 x = 1
Correct calculation (A1)
144
e.g. 5, 1 
169
5
sin θ = A1 N3
13

(b) Evidence of using formula for cos 2θ (M1)


e.g. cos 2θ = 2 cos2 θ – 1
Correct substitution/calculation A1
2
 12 
e.g. 2    1
 13 
119
cos 2θ = A1 N2
169

5
(c) sin (θ + π) = –sin θ =   A1 N1
13
[7]

206. (a) Attempt to factorise (M1)


correct factors (2sin θ – 1) (sin θ + 1) = 0 A1
1
sin θ = , sin θ = –1 A1A1 N2
2

(b) other solutions are 150°, 270° A1A1 N1N1


[6]

207. (a) When t = 1, l = 33 + 5 cos 720 (M1)


l = 33 + 5 = 38 A1 N2

(b) Minimum when cos = –1 (M1)


lmin = 33 – 5 (M1)
= 28 A1 N3

(c) 33 = 33 + 5cos720t (0 = 5 cos 720t) M1


720t = 90 A1
90  1 
t=   A1 N1
720  8 

(d) Evidence of dividing into 360 (M1)


360  1 
period =   A1 N2
720  2 
[10]


208. (a) period = =π M1A1 N2
2

π
(b) m= A2 N2
2

π
(c) Using A = 
0
2 sin 2 xdx (M1)
π
 1 2
Integrating correctly, A =   cos 2 x  A1
 2 0
1 1
Substituting, A =  cos π  ( cos 0) (M1)
2 2
1 1  1 1
Correct values, A =  (1)  ( (1))     A1A1
2 2  2 2
A=1 A1 N2
[10]
209. (a) Evidence of using the cosine rule (M1)
p2  r 2  q2
eg cos PQ̂R = , q 2  p 2  r 2  2 pr cos PQ̂R
2 pr
Correct substitution A1
42  62  52 2
eg , 5  4 2  6 2  2  4  6 cos Q
2 4 6
27
cos PQ̂R =  0.5625 (A1)
48
PQ̂R = 55.8 (0.973 radians) A1 N2

1
(b) Area = pr sin PQ̂R
2
1
For substituting correctly  4  6 sin 55.8 A1
2
= 9.92 (cm2) A1 N1
[6]

210. Note: Throughout this question, do not accept methods which involve
finding  .
(a) Evidence of correct approach A1
BC
eg sin  = , BC  32  2 2  5
AB
5
sin  = AG N0
3

(b) Evidence of using sin 2 = 2 sin  cos  (M1)


 5  2 
= 2  
 3  A1
 3 
4 5
= AG N0
9
(c) Evidence of using an appropriate formula for cos 2 M1
4 5 4 5  80 
eg  , 2  1, 1  2  , 1  
9 9 9 9  81 
1
cos 2 =  A2 N2
9
[6]

211. (a) For using perimeter = r + r + arc length (M1)


20 = 2r + r A1
20  2r
 AG N0
r

(b) Finding A =
1 2  20  2r 
r 
2  r 

 10r  r
2
 (A1)

For setting up equation in r M1


Correct simplified equation, or sketch
eg 10r – r2 = 25, r2 – 10r + 25 = 0 (A1)
r = 5 cm A1 N2
[6]
5 212. Notes:Candidates may have differing answers due to using approximate
answers from previous parts or using answers from the GDC.
Some leeway is provided to accommodate this.
(a) METHOD 1
Evidence of using the cosine rule (M1)
a b c
2 2 2
eg cos C = , a 2  b 2  c 2  2bc cos A
2ab
Correct substitution
32  2 2  4 2 2 2
eg cos AÔP = , 4  3  2 2  2  3 2 cos AÔP A1
2  3 2
cos AÔP = 0.25
 26  
AÔP = 1.82    (radians) A1 N2
 45 
METHOD 2
Area of AOBP = 5.81 (from part (d))
Area of triangle AOP = 2.905 (M1)
2.9050 = 0.5  2  3  sin AÔP A1
AÔP = 1.32 or 1.82
 26  
AÔP = 1.82    (radians) A1 N2
 45 

(b) AÔB = 2(  1.82) (= 2  3.64) (A1)


 38  
= 2.64    (radians) A1 N2
 45 
(c) (i) Appropriate method of finding area (M1)
1 2
eg area = θr
2
1 2
Area of sector PAEB =  4 1.63 A1
2
= 13.0 (cm2)
(accept the exact value 13.04) A1 N2
1 2
(ii) Area of sector OADB =  3  2.64 A1
2
= 11.9 (cm2) A1 N1

(d) (i) Area AOBE = Area PAEB  Area AOBP (= 13.0  5.81) M1
= 7.19 (accept 7.23 from the exact answer for PAEB) A1 N1
(ii) Area shaded = Area OADB  Area AOBE (= 11.9  7.19) M1
= 4.71 (accept answers between 4.63 and 4.72) A1 N1
[14]

213. (a) Evidence of choosing cosine rule (M1)


eg a2 = b2 + c2  2bc cos A
Correct substitution A1
eg (AD)2 = 7.12 + 9.22  2(7.1) (9.2) cos 60
(AD)2 = 69.73 (A1)
AD = 8.35 (cm) A1 N2

(b) 180  162 = 18 (A1)


Evidence of choosing sine rule (M1)
Correct substitution A1
DE 8.35
eg =
sin 18 sin 110 
DE = 2.75 (cm) A1 N2
(c) Setting up equation (M1)
1 1
eg ab sin C = 5.68, bh = 5.68
2 2
Correct substitution A1
1 1
eg 5.68 = (3.2) (7.1) sin DB̂C ,  3.2  h = 5.68, (h = 3.55)
2 2
sin DB̂C = 0.5 (A1)
DB̂C 30 and/or 150 A1 N2

(d) Finding A B̂ C (60 + D B̂ C) (A1)


Using appropriate formula (M1)
eg (AC)2 = (AB)2 + (BC)2, (AC)2 = (AB)2 + (BC)2  2 (AB)
(BC) cos ABC
Correct substitution (allow FT on their seen AB̂C )
eg (AC)2 = 9.22 + 3.22 A1
AC = 9.74 (cm) A1 N3

(e) For finding area of triangle ABD (M1)


1
Correct substitution Area =  9.2  7.1 sin 60 A1
2
= 28.28... A1
Area of ABCD = 28.28... + 5.68 (M1)
= 34.0 (cm2) A1 N3
[21]
214. (a)
y
10

–360° –180° 0 180° 360° x

–5

–10
Correct asymptotes A1A1 N2

(b) (i) Period = 360 (accept 2) A1 N1


(ii) f (90) = 2 A1 N1

(c) 270, 90 A1A1 N1N1


Notes: Penalize 1 mark for any additional values.
Penalize 1 mark for correct answers given
 3  
in radians  ,  , or 4.71, 1.57  .
 2 2 
[6]

215. Note: Accept exact answers given in terms of .


(a) Evidence of using l = r (M1)
arc AB = 7.85 (m) A1 N2

1 2
(b) Evidence of using A  r θ (M1)
2
Area of sector AOB = 58.9 (m2) A1 N2
(c) METHOD 1

π6
2 π
3


angle = 30 (A1)
6

attempt to find 15 sin M1
6

height = 15 + 15 sin
6
= 22.5 (m) A1 N2
METHOD 2

π
3


angle = 60 (A1)
3

attempt to find 15 cos M1
3

height = 15 + 15 cos
3
= 22.5 (m) A1 N2
    
(d) (i) h  15 15 cos    (M1)
4 2 4
= 25.6 (m) A1 N2
 
(ii) h(0) = 15  15 cos  0   (M1)
 4
= 4.39(m) A1 N2
(iii) METHOD 1
Highest point when h = 30 R1
 
30 = 15  15 cos  2t   M1
 4
 
cos  2t   = 1 (A1)
 4
 3 
t = 1.18  accept  A1 N2
 8 
METHOD 2
h
30

2π t
Sketch of graph of h M2
Correct maximum indicated (A1)
t = 1.18 A1 N2
METHOD 3
Evidence of setting h(t) = 0 M1
 
sin  2t    0 (A1)
 4
Justification of maximum R1
eg reasoning from diagram, first derivative test, second
derivative test
 3 
t = 1.18  accept  A1 N2
 8 

 
(e) h(t) = 30 sin  2t   (may be seen in part (d)) A1A1 N2
 4

(f) (i)
h(t)
30

π π t
2

–30

A1A1A1 N3
Notes: Award A1 for range 30 to 30, A1
for two zeros.
Award A1 for approximate correct
sinusoidal shape.
(ii) METHOD 1
Maximum on graph of h (M1)
t = 0.393 A1 N2
METHOD 2
Minimum on graph of h (M1)
t = 1.96 A1 N2
METHOD 3
Solving h(t) = 0 (M1)
One or both correct answers A1
t = 0.393, t = 1.96 N2
[22]
216. METHOD 1
1 2
Evidence of correctly substituting into A = r θ A1
2
Evidence of correctly substituting into l = r A1
For attempting to eliminate one variable … (M1)
leading to a correct equation in one variable A1

r=4 = (= 0.524, 30) A1A1 N3
6
METHOD 2
Setting up and equating ratios (M1)
4 2
 
3  3 A1A1
r 2 2r
Solving gives r = 4 A1
2  1 2 4 
r =   or r     A1
3  2 3 

=  0.524 , 30 A1
6

r=4 =  0.524 , 30 N3
6
[6]

  3 
217. a = 4, b = 2, c =  or etc  A2A2A2 N6
2  2 
[6]

  5 
218. (a) PQ =   A1A1 N2
  3

(b) Using r = a + tb
 x  1  5 
      t   A2A1A1 N4
 y   6    3
[6]
219. METHOD 1
Evidence of correctly substituting into l = r A1
1 2
Evidence of correctly substituting into A = r  A1
2
For attempting to solve these equations (M1)
eliminating one variable correctly A1
r = 15  = 1.6 (= 91.7) A1A1 N3
METHOD 2
Setting up and equating ratios (M1)
24 180
 A1A1
2r r 2
Solving gives r = 15 A1
 1 2 
r = 24  or r θ 180  A1
 2 
 = 1.6 (= 91.7) A1
r = 15  = 1.6 (= 91.7) N3
[6]

220. (a) For correct substitution into cosine rule A1


BD = 4  8  2  4  8 cos θ
2 2

For factorizing 16, BD = 165  4 cos θ  A1


= 4 5  4 cos θ AG N0
(b) (i) BD = 5.5653 ... (A1)
sin CB̂D sin 25
 M1A1
12 5.5653
sin CB̂D = 0.911 (accept 0.910, subject to AP) A1 N2
(ii) CB̂D = 65.7 A1 N1
Or CB̂D = 180  their acute angle (M1)
= 114 A1 N2
(iii) BD̂C = 89.3 (A1)
BC 5.5653 BC 12
 or  (or cosine rule) M1A1
sin 89.3 sin 25 sin 89.3 sin 65.7
BC = 13.2 (accept 13.17…) A1
Perimeter = 4 + 8 + 12 + 13.2
= 37.2 A1 N2

1
(c) Area =  4  8  sin 40 A1
2
= 10.3 A1 N1
[16]

221. (a) Evidence of choosing the double angle formula (M1)


f (x) = 15 sin (6x) A1 N2

(b) Evidence of substituting for f (x) (M1)


eg 15 sin 6x = 0, sin 3x = 0 and cos 3x = 0
6x = 0, , 2
 
x = 0, , A1A1A1 N4
6 3
[6]
222. (a) (i) OP = PQ (= 3cm) R1
So  OPQ is isosceles AG N0
3 3 4
2 2 2
(ii) Using cos rule correctly eg cos OP̂Q = (M1)
2  3 3
9  9 16  2 
cos OP̂Q =   A1
18  18 
1
cos OP̂Q = AG N0
9
(iii) Evidence of using sin2 A + cos2 A = 1 M1
1  80 
sin OP̂Q = 1    A1
81  81 
80
sin OP̂Q = AG N0
9
1
(iv) Evidence of using area triangle OPQ =  OP  PQ sin P M1
2
1 80 9
eg 3 3 ,  0.9938 
2 9 2

Area triangle OPQ =


80
2
 20   4.47  A1 N1

(b) (i) OP̂Q = 1.4594...


OP̂Q = 1.46 A1 N1
(ii) Evidence of using formula for area of a sector (M1)
1 2
eg Area sector OPQ =  3 1.4594 
2
= 6.57 A1 N2

 1.4594 
(c) QÔP =  0.841 (A1)
2
1 2
Area sector QOS =  4  0.841 A1
2
= 6.73 A1 N2
(d) Area of small semi-circle is 4.5 (= 14.137...) A1
Evidence of correct approach M1
eg Area = area of semi-circle  area sector OPQ  area sector QOS +
area triangle POQ
Correct expression A1
eg 4.5  6.5675...  6.7285... + 4.472..., 4.5  (6.7285... + 2.095...),
4.5 (6.5675... + 2.256...)
Area of the shaded region = 5.31 A1 N1
[17]

223. (a) p = 30 A2 2
(b) METHOD 1
Period = 2 (M2)
q
=  (A1)
2
q=4 A1 4

METHOD 2
Horizontal stretch of scale factor = 1 (M2)
q
1
scale factor = (A1)
4
q=4 A1 4
[6]

224. (a) using the cosine rule (A2) = b2 + c2 –2bc cos  (M1)
substituting correctly BC2 = 652 +1042 –2 (65) (104) cos 60° A1
= 4225 + 10816 – 6760 = 8281
 BC = 91 m A1 3

1
(b) finding the area, using bc sin  (M1)
2
substituting correctly, area = 1 (65) (104) sin 60° A1
2
= 1690 3 (Accept p = 1690) A1 3
(c) (i) A1 =  1  (65) (x) sin 30° A1
2
= 65x AG 1
4
(ii) A2 =  1  (104) (x) sin 30° M1
2
= 26x A1 2

(iii) starting A1 + A2 = A or substituting 65x + 26x = 1690 3 (M1)


4
simplifying 169x = 1690 3 A1
4
4  1690 3
x= A1
169
 x = 40 3 (Accept q = 40) A1 4

(d) (i) Recognizing that supplementary angles have equal sines


eg AD̂C = 180 – AD̂B  sin AD̂C = sin AD̂B R1

(ii) using sin rule in ΔADB and ΔACD (M1)


65 sin 30
substituting correctly BD   BD  A1
sin 30 sin AD̂B 65 sin AD̂B
sin 30
and DC  104  DC  M1
sin 30 sin AD̂B 104 sin AD̂C
since sin AD̂B = sin AD̂C
BD  DC  BD  65 A1
65 104 DC 104
 BD  5 AG 5
DC 8
[18]
1
225. (a) A  r 2
2
1
27  (1.5) r 2 (M1)(A1)
2
r 2  36 (A1)
r  6 cm (A1) (C4)

(b) Arc length  r  1.5 6 (M1)


Arc length = 9 cm (A1) (C2)
Note: Penalize a total of (1 mark) for missing units.
[6]

226. (a) when y  0 (may be implied by a sketch) (A1)



x or 2.79 (A1) (C2)
9

(b) METHOD 1
Sketch of appropriate graph(s) (M1)
Indicating correct points (A1)
x  3.32 or x  5.41 (A1)(A1)(C2)(C2)
METHOD 2
 π 1
sin  x    
 9 2
π 7π π 11π
x  , x  (A1)(A1)
9 6 9 6
7π π 11π π
x  , x 
6 9 6 9
19π 31π
x , x ( x  3.32, x  5.41) (A1)(A1)(C2)(C2)
18 18
[6]
227. (a) for using cosine rule  a 2  b2  c 2  2ab cos C  (M1)
BC  15  17  2 1517  cos 29
2 2 2
(A1)
BC  8.24 m
(A1) (N0) 3
Notes: Either the first or the second line may be implied, but
not both. Award no marks if 8.24 is obtained by assuming a
right (angled) triangle (BC = 17 sin 29).

(i)
A C
29°

17
85°
B
A CB  180  (29  85)  66
for using sine rule (may be implied) (M1)
AC 17
 (A1)
sin85 sin 66
17sin85
AC 
sin 66
AC  (18.5380 )  18.5 m (A1) (N2)

1
(ii) Area  17 18.538... sin 29 (A1)
2
 76.4 m2 (Accept 76.2 m2 )
(A1) (N1) 5
(c) AĈB from previous triangle  66
ˆ  180  66  114 (or 29  85)
Therefore alternative ACB (A1)
ˆ  180  (29  114)  37
ABC
A C
29° 114°

17

37°

B
AC 17
 (M1)(A1)
sin37 sin114
AC  (11.19906 )  11.2 m
(A1) (N1) 4

(d)
A C
29°

17

B
Minimum length for BC when AĈB = 90°or diagram
showing right triangle (M1)
CB
sin 29 
17
CB  17sin 29
CB  (8.2417 )  8.24 m
(A1) (N1) 2
[14]
1
228. (a) (i) f ( x)   2cos2 x  sin x
2
 cos 2 x  sin x (A1)(A1) (N2)
Note: Award (A1)(A1) for 2sin x  sin x  1 only if work
2

shown, using product rule on sin x cos x  cos x .

(ii) 2sin 2 x  sin x  1  (2sin x  1)(sin x  1) or


2(sin x  0.5) (sin x  1) (A1) (N1)

(iii) 2sin x  1 or sin x  1


1
sin x 
2
π 5π 3π
x   (0.524) x   (2.62) x   (4.71) (A1)(A1)(A1)(N1)
6 6 2
(N1)(N1) 6

π
(b) x   0.524 (A1) (N1) 1
6

(c) (i) EITHER


π
curve crosses axis when x  (may be implied) (A1)
2
π 5π
Area   π2 f ( x)dx   π
6
f ( x)dx (M1)(A1) (N3)
6 2

OR
5

Area =  
6
f ( x ) dx (M1)(A2) (N3)
6

(ii) Area  0.875  0.875 (M1)


 1.75 (A1) (N2) 5
[12]
1
229. Using area of a triangle = ab sin C (M1)
2
1
20  (10)(8) sin Q (A1)(A1)(A1)
2
Note: Accept any letter for Q
sin Q = 0.5 (A1)

PQ̂R = 30 or or 0.524 (A1) (C6)
6
[6]

230. (a) 3(1  2 sin2 x) + sin x = 1 (A1)


6 sin2 x  sin x  2 = 0 (p = 6, q = 1, r = 2) (A1) (C2)

(b) (3 sin x  2)(2 sin x + 1) (A1)(A1) (C2)

(c) 4 solutions (A2) (C2)


[6]

1 2 1
231. Area of large sector r θ = 162 × 1.5 (M1)
2 2
= 192 (A1)
1 1
Area of small sector r2θ = × 102 × 1.5 (M1)
2 2
= 75 (A1)
Shaded area = large area – small area = 192 – 75 (M1)
= 117 (A1) (C6)
[6]
232. (a)
y
2

1.5

0.5

0
0.5 1 1.5 2 2.5 3 3.5 x

–0.5

–1

–1.5

–2 (A1)(A1) (C2)
Note: Award (A1) for the graph crossing the y-axis between 0.5
and 1, and (A1) for an approximate sine curve crossing the x-
axis twice. Do not penalize for x >3.14.

π 1
(b) (Maximum) x = 0.285…    (A1)
 4 2
x = 0.3 (1 dp) (A1) (C2)
 3π 1 
(Minimum) x = 1.856…    (A1)
 4 2
x = 1.9 (1 dp) (A1) (C2)
[6]

1
233. Area of a triangle = × 3 × 4 sin A (A1)
2
1
× 3 × 4 sin A = 4.5 (A1)
2
sin A = 0.75 (A1)
A = 48.6° and A = 131° (or 0.848, 2.29 radians) (A1)(A2) (C6)
Note: Award (C4) for 48.6° only, (C5) for 131° only.
[6]
234. METHOD 1
2 cos2 x = 2 sin x cos x (M1)
2 cos2 x – 2 sin x cos x = 0
2 cos x(cos x – sin x) = 0 (M1)
cos x = 0, (cos x – sin x) = 0 (A1)(A1)
π π
x= ,x= (A1)(A1) (C6)
2 4

METHOD 2
Graphical solutions
EITHER
for both graphs y = 2 cos2 x, y = sin 2 x, (M2)
OR
for the graph of y = 2 cos2 x – sin 2 x. (M2)

THEN
Points representing the solutions clearly indicated (A1)
1.57, 0.785 (A1)
π π
x= ,x= (A1)(A1) (C6)
2 4
Notes: If no working shown, award (C4) for one correct
answer.
Award (C2)(C2) for each correct decimal answer 1.57, 0.785.
Award (C2)(C2) for each correct degree answer 90°, 45°.
Penalize a total of [1 mark] for any additional answers.
[6]

235. (a) (i) 10 + 4 sin 1 = 13.4 (A1)


(ii) At 2100, t = 21 (A1)
10 + 4 sin 10.5 = 6.48 (A1) (N2) 3
Note: Award (A0)(A1) if candidates use t = 2100 leading to
y = 12.6. No other ft allowed.

(b) (i) 14 metres (A1)


t t
(ii) 14 = 10 + 4 sin    sin   = 1 (M1)
2 2
 t = π (3.14) (correct answer only) (A1) (N2) 3
(c) (i) 4 (A1)
t
(ii) 10 + 4 sin   = 7 (M1)
2
t
 sin   = –0.75 (A1)
2
 t = 7.98 (A1) (N3)

(iii) depth < 7 from 8 –11 = 3 hours (M1)


from 2030 – 2330 = 3 hours (M1)
therefore, total = 6 hours (A1) (N3) 7
[13]

236. (a) Angle A  80 (A1)


AB 5
 (M1)
sin 40 sin80
AB  3.26 cm (A1) (C3)

1 1
(b) Area  ac sin B  (5)(3.26)sin 60 (M1)(A1)
2 2
 7.07 (accept 7.06) cm2 (A1) (C3)
Note: Penalize once in this question for absence of units.
[6]
237. METHOD 1
1 2
Area sector OAB  (5) (0.8) (M1)
2
 10 (A1)
ON  5cos0.8   3.483... (A1)
AN  5sin 0.8   3.586..... (A1)
1
Area of  AON  ON  AN
2
 6.249... (cm2 ) (A1)
Shaded area  10  6.249..
 3.75 (cm2 ) (A1) (C6)

METHOD 2
A

O N B

F
1 2
Area sector ABF  (5) (1.6) (M1)
2
 20 (A1)
1 2
Area OAF  (5) sin1.6 (M1)
2
 12.5 (A1)
Twice the shaded area  20  12.5 ( 7.5) (M1)
1
Shaded area  (7.5)
2
 3.75 (cm2 ) (A1) (C6)
[6]
238. 3 = p + q cos 0 (M1)
3=p+q (A1)
–1 = p + q cos  (M1)
–1 = p – q (A1)
(a) p = 1 (A1) (C3)
(b) q = 2 (A1) (C3)
[6]

239. Method 1
y

0 1.80 2.51 x

0 (C2)
1.80 [3 sf] (G2) (C2)
2.51 [3 sf] (G2) (C2)

Method 2
3x = ±0.5x + 2 (etc.) (M1)
 3.5x = 0, 2, 4 or 2.5x = 0, 2, 4 (A1)
7x = 0, 4, (8) or 5x = 0, 4, (8) (A1)
4π 4π
x = 0, or x = 0, (A1)(A1)(A1)
7 5
4π 4π
x = 0, , (C2)(C2)(C2)
7 5
[6]
1
240. (a) area of sector ΑΒDC = π(2)2 = π (A1)
4
area of segment BDCP = π – area of ABC (M1)
=π–2 (A1) (C3)

(b) BP = 2 (A1)
1
area of semicircle of radius BP = π( 2 )2 = π (A1)
2
area of shaded region = π – (π – 2) = 2 (A1) (C3)
[6]

241. (a) OR  PQ
=q–p
10   7 
=   –   (A1)(A1)
 1   3
 3 
=   (A1) 3
 – 2

PO  PQ
(b) cos OP̂Q  (A1)
PO  PQ

PO  – 72  – 32 = PQ  32  – 2 =
2
58 , 13 (A1)(A1)
PO  PQ = –21 + 6 = –15 (A1)
– 15 – 15
cos OP̂Q   (AG) 4
58 13 754

(c) (i) Since OP̂Q + PQ̂R = 180° (R1)


 15 
cos PQ̂R = –cos OP̂Q    (AG)
 754 
2
 15 
(ii) sin PQ̂R = 1–   (M1)
 754 
529
= (A1)
754
23
= (AG)
754
OR
15
cos  =
754

754
15

x P
(M1)
therefore x = 754 – 225 = 529  x = 23
2
(A1)
23
 sin  = (AG)
754
Note: Award (A1)(A0) for the following solution.
15
cos  =   = 56.89°
754
 sin  = 0.8376
23 23
= 0.8376  sin  =
754 754

(iii) Area of OPQR = 2 (area of triangle PQR) (M1)


1
=2× PQ  QR  sin PQ̂R (A1)
2
1 23
=2× 13 58 (A1)
2 754
= 23 sq units. (A1)
OR
Area of OPQR = 2 (area of triangle OPQ) (M1)
1
= 2  7  1 – 3  10 (A1)(A1)
2
= 23 sq units. (A1) 7
Notes: Other valid methods can be used.
Award final (A1) for the integer answer.
[14]

PR 9
242. (a) Sine rule  (M1)(A1)
sin35 sin 120
9 sin 35
PR =
sin 120
= 5.96 km (A1) 3

(b) EITHER
Sine rule to find PQ
9 sin 25
PQ = (M1)(A1)
sin 120
= 4.39 km (A1)
OR
Cosine rule: PQ2 = 5.962 + 92 – (2)(5.96)(9) cos 25 (M1)(A1)
= 19.29
PQ = 4.39 km (A1)
4.39
Time for Tom = (A1)
8
5.96
Time for Alan = (A1)
a
4.39 5.96
Then = (M1)
8 a
a = 10.9 (A1) 7
(c) RS2 = 4QS2 (A1)
4QS2 = QS2 + 81 – 18 × QS × cos 35 (M1)(A1)
3QS2 + 14.74QS – 81 = 0 (or 3x2 + 14.74x – 81 = 0) (A1)
 QS = –8.20 or QS = 3.29 (G1)
therefore QS = 3.29 (A1)
OR
QS 2QS
 (M1)
sinS R̂Q sin35
1
 sin SR̂Q  sin 35 (A1)
2
SR̂Q = 16.7° (A1)
Therefore, QŜR = 180 – (35 + 16.7)
= 128.3° (A1)
9 QS  SR 
   (M1)
sin 128.3 sin16.7  sin 35 
9 sin 16.7  9 sin 35 
QS =  
sin 128.3  2 sin 128.3 
= 3.29 (A1) 6
[16]

 π 1  π 1
243. (a) (i) cos  –  , sin  –   – (A1)
 4 2  4 2
 π  π
therefore cos  –   sin  –  = 0 (AG)
 4  4

(ii) cos x + sin x = 0  1 + tan x = 0


 tan x = –l (M1)

x= (A1)
4
Note: Award (A0) for 2.36.

OR

x= (G2) 3
4
(b) y = ex(cos x + sin x)
dy
= ex(cos x + sin x) + ex(–sin x + cos x) (M1)(A1)(A1) 3
dx
= 2ex cos x

dy
(c) = 0 for a turning point  2ex cos x = 0 (M1)
dx
 cos x = 0 (A1)
π π
x= a= (A1)
2 2
π π
2 π π
y = e (cos + sin ) = e 2
2 2
π
b=e2 (A1) 4
Note: Award (M1)(A1)(A0)(A0) for a = 1.57, b = 4.81.

d2 y
(d) At D, =0 (M1)
dx 2
2ex cos x – 2exsin x = 0 (A1)
2ex (cos x – sin x) = 0
 cos x – sin x = 0 (A1)
π
x= (A1)
4
π
π π
 y = e 4 (cos + sin ) (A1)
4 4
π
= 2e 4
(AG) 5

3
(e) Required area = 
0
4 ex (cos x + sin x)dx (M1)
= 7.46 sq units (G1)
OR
Αrea = 7.46 sq units (G2) 2
Note: Award (M1)(G0) for the answer 9.81 obtained if the
calculator is in degree mode.
[17]
4 
244. (a) (i) A is  , 0  (A1)(A1) (C2)
3 
(ii) B is (0, –4) (A1)(A1) (C2)
Note: In each of parts (i) and (ii), award C1 if A and B are
interchanged, C1 if intercepts given instead of coordinates.

1 4
(b) Area = ×4× (M1)
2 3
8
= (= 2.67) (A1) (C2)
3
[6]

245. (a) (3 sin x – 2)(sin x – 3) (A1)(A1) (C2)


2
Note: Award A1 if 3x – 11x + 6 correctly factorized to give
(3x – 2)(x – 3) (or equivalent with another letter).

(b) (i) (3 sin x – 2)(sin x – 3) = 0


2
sin x = sin x = 3 (A1)(A1) (C2)
3

(ii) x = 41.8°, 138° (A1)(A1) (C2)


Notes: Penalize [1 mark] for any extra answers and [1 mark]
for answers in radians.
ie Award A1 A0 for 41.8°, 138° and any extra answers.
Award A1 A0 for 0.730, 2.41.
Award A0 A0 for 0.730, 2.41 and any extra answers.
[6]

246. Note: Do not penalize missing units in this question.


2
(a) AB = 122 + 122 – 2 × 12 × 12 × cos 75° (A1)
= 122(2 – 2 cos 75°) (A1)
2
= 12 × 2(1 cos 75°)
AB = 12 2(1  cos 75) (AG) 2
Note: The second (A1) is for transforming the initial expression
to any simplified expression from which the given result can be
clearly seen.
(b) PÔB = 37.5° (A1)
BP = 12 tan 37.5° (M1)
= 9.21 cm (A1)
OR
BP̂A = 105° BÂP = 37.5° (A1)
AB BP
 (M1)
sin 105 sin 37.5
AB sin 37.5
BP = = 9.21(cm) (A1) 3
sin 105 

1  1 
(c) (i) Area ∆OBP =  12  9.21  or  12  12 tan 37.5  (M1)
2  2 
= 55.3 (cm ) (accept 55.2 cm2)
2
(A1)
1
(ii) Area ∆ABP = (9.21)2 sin105° (M1)
2
= 41.0 (cm2) (accept 40.9 cm2) (A1) 4

1 π  75 
(d) Area of sector =  12 2  75   or  π  12 2  (M1)
2 180  360 
2 2
= 94.2 (cm ) (accept 30π or 94.3 (cm )) (A1) 2

(e) Shaded area = 2 × area ∆OPB – area sector (M1)


= 16.4 (cm2) (accept 16.2 cm2, 16.3 cm2) (A1) 2
[13]

247. Note: Do not penalize missing units in this question.


(a) (i) At release(P), t = 0 (M1)
s = 48 + 10 cos 0
= 58 cm below ceiling (A1)

(ii) 58 = 48 +10 cos 2πt (M1)


cos 2πt = 1 (A1)
t = 1sec (A1)
OR
t = 1sec (G3) 5
ds
(b) (i) = –20π sin 2πt (A1)(A1)
dt
Note: Award (A1) for –20π, and (A1) for sin 2t.
ds
(ii) v= = –20π sin 2πt = 0 (M1)
dt
sin 2 πt = 0
1
t = 0, ... (at least 2 values) (A1)
2
s = 48 + 10 cos 0 or s = 48 +10 cos π (M1)
= 58 cm (at P) = 38 cm (20 cm above P) (A1)(A1) 7
Note: Accept these answers without working for full marks.
May be deduced from recognizing that amplitude is 10.

(c) 48 +10 cos 2πt = 60 + 15 cos 4πt (M1)


t = 0.162 secs (A1)
OR
t = 0.162 secs (G2) 2

(d) 12 times (G2) 2


Note: If either of the correct answers to parts (c) and (d) are
missing and suitable graphs have been sketched, award (G2)
for sketch of suitable graph(s); (A1) for t = 0.162; (A1) for 12.
[16]

248. (a) l = r or ACB = 2 × OA (M1)


= 30 cm (A1) (C2)

(b) AÔB (obtuse) = 2 – 2 (A1)


1 1
Area =  r 2 = (2 – 2)(15)2 (M1)(A1)
2 2
= 482 cm2 (3 sf) (A1) (C4)
[6]
249.
d B
A

50 80
70°

P
(M1)(A2)
OR
2.5 × 20 = 50 (M1)(A1)
2.5 × 32 = 80 (A1)
d = 502 + 802 – 2 × 50 × 80 × cos 70°
2
(M1)(A1)
d = 78.5 km (A1) (C6)
[6]

250. (a) (i) –1 (A1) (C1)


(ii) 4 (accept 720°) (A2) (C2)

(b)
y

 32

(G1)
number of solutions: 4 (A2) (C3)
[6]
251.
Statement (a) Is the statement true for all (b) If not true, example
real numbers x? (Yes/No)
A No x = –l (log10 0.1 = –1) (a) (A3)
(C3)
B No x = 0 (cos 0 = 1) (b) (A3)
(C3)
C Yes N/A
Notes: (a) Award (A1) for each correct answer.
(b) Award (A) marks for statements A and B only if NO in
column (a).
Award (A2) for a correct counter example to statement A, (A1)
for a correct counter example to statement B (ignore other
incorrect examples).
Special Case for statement C:
Award (A1) if candidates write NO, and give a valid reason (eg

arctan 1 = ).
4
[6]

7 2
6
252. (a)  2 (M1)
sin A sin 45
2 2
sin A = 6   (A1)
2 7 2
6
= (AG) 2
7

(b)
A

D h

B C
(i) BD̂C + BÂC = 180° (A1)
6
(ii) sin A =
7
=> A = 59.0° or 121° (3 sf) (A1)(A1)
=> BĈD = 180° – (121° + 45°)
= 14.0° (3 sf) (A1)

7 2
BD
(iii)  2 (M1)
sin 14 sin 45
=>BD = 1.69 (A1) 6

1
 BD  h
Area BDC 2
(c)  (M1)(A1)
Area BAC 1
 BA  h
2
BD
= (AG) 2
BA
OR
1
BD  6 sin 45
Area ΔBCD 2
 (M1)(A1)
Area ΔBAC 1
BA  6 sin 45
2
BD
= (AG) 2
BA
[10]

sin B sin 48


253. Using sine rule:  (M1)(A1)
5 7
5
 sin B = sin 48° = 0.5308… (M1)
7
 B = arcsin (0.5308) = 32.06° (M1)(A1)
= 32° (nearest degree) (A1) (C6)
Note: Award a maximum of [5 marks] if candidates give the
answer in radians (0.560).
[6]
254. (a) x is an acute angle => cos x is positive. (M1)
cos2 x + sin2 x = 1 => cos x = 1 – sin x
2
(M1)
2
1
=> cos x = 1–   (A1)
3
8 2 2
= (= ) (A1) (C4)
9 3

2
1
(b) cos 2x = 1 – 2 sin2 x = 1 – 2   (M1)
3
7
= (A1) (C2)
9
Notes: (a) Award (M1)(M0)(A1)(A0) for
  1 
cos  sin –1    = 0.943.
 3  
  1 
(b) Award (M1)(A0) for cos  2 sin –1    = 0.778.
 3  
[6]

255. (a) 2 sin2 x = 2(1 – cos2 x) = 2 – 2 cos2 x = l + cos x (M1)


=> 2 cos2 x + cos x – l = 0 (A1) (C2)
Note: Award the first (M1) for replacing sin2 x by
1 – cos2 x.

(b) 2 cos2 x + cos x – 1 = (2 cos x – 1)(cos x +1) (A1) (C1)

1
(c) cos x = or cos x = –l
2
=> x = 60°, 180° or 300° (A1)(A1)(A1) (C3)
Note: Award (A1)(A1)(A0) if the correct answers are given in
 5
radians (ie , , , or 1.05, 3.14, 5.24)
3 3
[6]
256. (a) The smallest angle is opposite the smallest side.
82  7 2  52
cos θ = (M1)
28 7
88 11
=  = 0.7857
112 14
Therefore, θ = 38.2° (A1) (C2)

1
(b) Area = × 8 × 7 × sin 38.2° (M1)
2
= 17.3 cm2 (A1) (C2)
[4]

257. (a) 3 sin2 x + 4 cos x = 3(1 – cos2 x) + 4cos x


= 3 – 3 cos2 + 4 cos x (A1) (C1)

(b) 3 sin2 x + 4 cos x – 4 = 0 3 – 3 cos2 x + 4 cos x – 4 = 0


 3 cos2 x – 4 cos x + 1 = 0 (A1)
(3 cos x – 1)(cos x – 1) = 0
1
cos x = or cos x = 1
3
x = 70.5° or x = 0° (A1)(A1) (C3)
Note: Award (C1) for each correct radian answer, ie x = 1.23
or x = 0.
[4]
258. OT̂A = 90° (A1)
AT = 12 2  6 2
= 6 3
π
TÔA = 60° = (A1)
3
Area = area of triangle – area of sector
1 1 π
= ×6× 6 3 – ×6×6× (M1)
2 2 3
2
= 12.3 cm (or 18 3 – 6) (A1) (C4)
OR
TÔA = 60° (A1)
1
Area of  = × 6 × 12 × sin 60 (A1)
2
1 π
Area of sector = ×6×6× (A1)
2 3
Shaded area = 18 3 – 6 = 12.3 cm2 (3 sf) (A1) (C4)
[4]

1 2 1
259. (a) Area = r   (152)(2) (M1)
2 2
2
= 225 (cm ) (A1) (C2)

1 2
(b) Area ∆OAB = 15 sin 2 = 102.3 (A1)
2
Area = 225 – 102.3 = 122.7 (cm2)
= 123 (3 sf) (A1) (C2)
[4]

sin (AĈB) sin 50


260. (a)  (M1)
20 17
20 sin 50
 sin (AĈB)  = 0.901
17
AĈB > 90°  AĈB = 180° – 64.3° = 115.7°
AĈB = 116 (3 sf) (A1) (C2)
(b) In Triangle 1, AĈB = 64.3°
 BÂC = 180° – (64.3° + 50°)
= 65.7° (A1)
1
Area = (20)(17) sin 65.7° = 155 (cm2) (3 sf) (A1) (C2)
2
[4]

261. METHOD 1
The value of cosine varies between –1 and +1. Therefore:
t = 0  a + b = 14.3
t = 6  a – b = 10.3
 2a = 24.6  a = 12.3 (A1) (C1)
 2b = 4.0  b = 2 (A1) (C1)
2π(12)
Period = 12 hours  = 2π (M1)
k
 k = 12 (A1) (C2)

METHOD 2
y

14.3

10.3

6 12 18 24 t (h)
From consideration of graph: Midpoint = a = 12.3 (A1) (C1)
Amplitude = b = 2 (A1) (C1)

Period = = 12 (M1)

k
 k = 12 (A1) (C2)
[4]
262.
C

32km

A
48km B
48 2  32 2  56 2
cos CÂB  (M1)(A1)
2(48)(32)
CÂB = arccos(0.0625) (A1)
 86° (A1)
[4]

263. (a) 2 cos2 x + sin x = 2(1 – sin2 x) + sin x


= 2 – 2 sin2 x + sin x (A1)

(b) 2 cos2 x + sin x = 2


 2 – 2 sin2 x + sin x = 2
sin x – 2 sin2 x = 0
sin x(1 – 2 sin x) = 0
1
sin x = 0 or sin x = (M1)
2
sin x = 0  x = 0 or  (0° or 180°) (A1)
Note: Award (A1) for both answers.
1 π 5π
sin x = x= or (30° or 150°) (A1)
2 6 6
Note: Award (A1) for both answers.
[4]

1
264. (a) (i) Q= (14.6 – 8.2) (M1)
2
= 3.2 (A1)
1
(ii) P = (14.6 + 8.2) (M0)
2
= 11.4 (A1) 3
π 
(b) 10 = 11.4 + 3.2 cos  t  (M1)
6 
7 π 
so = cos  t 
16 6 
7 π
therefore arccos   t (A1)
 16  6
π
which gives 2.0236... = t or t = 3.8648. t = 3.86(3 sf) (A1) 3
6

(c) (i) By symmetry, next time is 12 – 3.86... = 8.135... t = 8.14 (3 sf) (A1)
(ii) From above, first interval is 3.86 < t < 8.14 (A1)
This will happen again, 12 hours later, so (M1)
15.9 < t < 20.1 (A1) 4
[10]

265. 3 cos x = 5 sin x


sin x 3
  (M1)
cos x 5
 tan x = 0.6 (A1)
x = 31° or x = 211° (to the nearest degree) (A1)(A1)(C2)(C2)
Note: Deduct [1 mark] if there are more than two answers.
[4]

5 12
266. sin A =  cos A =  (A1)
13 13
12
But A is obtuse  cos A = – (A1)
13
sin 2A = 2 sin A cos A (M1)
5  12 
=2×   
13  13 
120
=– (A1) (C4)
169
[4]
267. (a) y =  sin x – x

y
3

2 (1.25, 1.73)

1
(2.3, 0)

–3 –2 –1 x
1 2 3
(–2.3, 0)
–1

(–1.25, –1.73) –2

–3
(A5) 5
Notes: Award (A1) for appropriate scales marked on the axes.
Award (A1) for the x-intercepts at (2.3, 0).
Award (A1) for the maximum and minimum points at (1.25,
1.73).
Award (A1) for the end points at (3, 2.55).
Award (A1) for a smooth curve.
Allow some flexibility, especially in the middle three marks
here.

(b) x = 2.31 (A1) 1

x2
(c)  ( π sin x  x)dx   π cos x 
2
C (A1)(A1)
Note: Do not penalize for the absence of C.
1
Required area =  0
( π sin x  x)dx (M1)
= 0.944 (G1)
OR area = 0.944 (G2) 4
[10]
268. (a)

30º

Acute angle 30° (M1)


Note: Award the (M1) for 30° and/or quadrant diagram/graph
seen.
2nd quadrant since sine positive and cosine negative
  = 150° (A1) (C2)

1
(b) tan 150° = –tan 30° or tan 150° = 2 (M1)
3

2
1
tan 150° = – (A1) (C2)
3
[4]

PQ
269. (a) = tan 36°
40
 PQ  29.1 m (3 sf) (A1) (C1)
(b)
B
40m 30

Q 70
A
AQ̂B = 80° (A1)
AB 40
 (M1)
sin 80 sin 70
Note: Award (M1) for correctly substituting.
 AB = 41 9. m (3 sf) (A1) (C3)
[4]

270. Perimeter = 5(2π – 1) + 10 (M1)(A1)(A1)


Note: Award (M1) for working in radians; (A1) for 2π – 1; (A1)
for +10.
= (10π + 5) cm (= 36.4, to 3 sf) (A1) (C4)
[4]
271. (a)(i) & (c)(i)
y

(1.1, 0.55)

(1.51, 0)
0 x
1 2

–1

(2, –1.66)

–2

(A3)
Notes: The sketch does not need to be on graph paper. It should
have the correct shape, and the points (0, 0), (1.1, 0.55), (1.57,
0) and (2, –1.66) should be indicated in some way.
Award (A1) for the correct shape.
Award (A2) for 3 or 4 correctly indicated points, (A1) for 1 or 2
points.

(ii) Approximate positions are


positive x-intercept (1.57, 0) (A1)
maximum point (1.1, 0.55) (A1)
end points (0, 0) and (2, –1.66) (A1)(A1) 7

(b) x2 cos x = 0 x ≠ 0 ⇒ cos x = 0 (M1)


π
x= (A1) 2
2
Note: Award (A2) if answer correct.
(c) (i) see graph (A1)

(ii)  0
2 x 2 cos x dx (A2) 3
Note: Award (A1) for limits, (A1) for rest of integral correct (do
not penalize missing dx).

(d) Integral = 0.467 (G3)


OR

Integral = x 2 sin x  2 x cos x  2 sin x 0  π/2
(M1)
π 2
π 
=  (1)  2 (0)  2(1) – [0 + 0 – 0] (M1)
 4 2 
π
= – 2 (exact) or 0.467 (3 sf) (A1) 3
2
[15]

272. (a) From graph, period = 2π (A1) 1


(b) Range = {y –0.4 < y < 0.4} (A1) 1

d
(c) (i) f (x) = {cos x (sin x)2}
dx
= cos x (2 sin x cos x) – sin x (sin x)2 or –3 sin3 x + 2 sin x(M1)(A1)(A1)
Note: Award (M1) for using the product rule and (A1) for each
part.

(ii) f (x) = 0 (M1)


 sin x{2 cos x – sin2 x} = 0 or sin x{3 cos x – 1} = 0 (A1)
 3 cos2 x – 1 = 0
1
 cos x = ±   (A1)
3
1
At A, f (x) > 0, hence cos x =   (R1)(AG)
3
 2

 1    1   
(iii) f (x) =  1 –    (M1)
 3    3   
 
2 1 2
=   3 (A1) 9
3 3 9

π
(d) x= (A1) 1
2

1
 (cos x)(sin x) dx  sin 3 x  c
2
(e) (i) (M1)(A1)
3

1  
3
π/2 π
 (cosx)(sin x) dx   sin   (sin 0) 3 
2
(ii) Area = (M1)
0 3  2 
1
= (A1) 4
3

(f) At C f (x) = 0 (M1)


 9 cos3 x – 7 cos x = 0
 cos x(9 cos2 x – 7) = 0 (M1)
π 7
x= (reject) or x = arccos = 0.491 (3 sf) (A1)(A1) 4
2 3
[20]

273. Note: Award (M1) for identifying the largest angle.


4  52  7 2
2
cos  = (M1)
2 45
1
=– (A1)
5
  = 101.5° (A1)

OR Find other angles first


 = 44.4°  = 34.0° (M1)
  = 101.6° (A1)(A1) (C4)
Note: Award (C3) if not given to the correct accuracy.
[4]

274. AB = r
1 2 2
= r  (M1)(A1)
2 r
2
= 21.6 × (A1)
5.4
= 8 cm (A1)

1
OR × (5.4)2 = 21.6
2
4
= (= 1.481 radians) (M1)
2 .7
AB = r (A1)
4
= 5.4 × (M1)
2 .7
= 8 cm (A1) (C4)
[4]

275. (a) OA = 6  A is on the circle (A1)

OB = 6  B is on the circle. (A1)

 5 
OC   
 11 
= 25  11
=6  C is on the circle. (A1) 3

(b) AC  OC  OA
 5  6
=      (M1)
 11   0 
 1 
=   (A1) 2
 11 
AO  AC
(c) cos OAˆ C  (M1)
AO AC
  6   1 
 . 
 0   11 
=
6 1  11
6
= (A1)
6 12
1 3
=  (A1)
2 3 6
6 2  ( 12 ) 2  6 2
OR cos OAˆ C  (M1)(A1)
2  6  12
1
= as before (A1)
12
OR using the triangle formed by AC and its horizontal and
vertical components:
AC  12 (A1)
1
cos OAˆ C  (M1)(A1) 3
12
Note: The answer is 0.289 to 3 sf

(d) A number of possible methods here


BC  OC  OB
 5    6
=      (A1)
 11   0 
 11 
=   (A1)
 11 
BC  = 132
1
ABC =  132  12 (A1)
2
= 6 11 (A1)

OR ABC has base AB = 12 (A1)


and height = 11 (A1)
1
 area =  12  11 (A1)
2
= 6 11 (A1)

3
OR Given cos BAˆ C 
6
33 1 33
sin BAˆ C   ABC   12  12  (A1)(A1)(A1)
6 2 6
= 6 11 (A1) 4
[12]
1
276. tan2 x = (M1)
3
1
 tan x =  (M1)
3
 x = 30° or x = 150° (A1)(A1)(C2)(C2)
[4]

277. h = r so 2r2 = 100  r2 = 50 (M1)


l = 10 = 2r (M1)
2 π 50
= (A1)
10
2 π5 2
=
10
 =  2 = 4.44 (3sf) (A1) (C4)
Note: Accept either answer.
[4]

278. (a) f (1) = 3 f (5) = 3 (A1)(A1) 2

(b) EITHER distance between successive maxima = period (M1)


=5–1 (A1)
=4 (AG)

OR Period of sin kx = ; (M1)
k

so period = (A1)
π
2
=4 (AG) 2
π  3π 
(c) EITHER A sin   + B = 3 and A sin   + B = –1 (M1) (M1)
2  2 
 A + B = 3, – A + B = –1 (A1)(A1)
 A = 2, B = 1 (AG)(A1)
OR Amplitude = A (M1)
3  (1) 4
A=  (M1)
2 2
A=2 (AG)
Midpoint value = B (M1)
3  (1) 2
B=  (M1)
2 2
B=1 (A1) 5
Note: As the values of A = 2 and B = 1 are likely to be quite
obvious to a bright student, do not insist on too detailed a
proof.

π 
(d) f (x) = 2 sin  x  + 1
2 
π π 
f (x) =  2 cos  x  + 0 (M1)(A2)
2 2 
π
Note: Award (M1) for the chain rule, (A1) for   , (A1) for
2
π 
2 cos  x  .
2 
π 
=  cos  x  (A1) 4
2 
Notes: Since the result is given, make sure that reasoning is
valid. In particular, the final (A1) is for simplifying the result of
the chain rule calculation. If the preceding steps are not valid,
this final mark should not be given. Beware of “fudged” results.

π 
(e) (i) y = k – x is a tangent  – =  cos  x  (M1)
2 
π 
 –1 = cos  x  (A1)
2 
π
 x =  or 3 or ...
2
 x = 2 or 6 ... (A1)
Since 0  x  5, we take x = 2, so the point is (2, 1) (A1)
(ii) Tangent line is: y = –(x – 2) + 1 (M1)
y = (2 + 1) – x
k = 2 + 1 (A1) 6

π 
(f) f (x) = 2  2 sin  x  + 1 = 2 (A1)
2 
π  1
 sin  x   (A1)
2  2
π π 5π 13π
 x  or or
2 6 6 6
1 5 13
x = or or (A1)(A1)(A1) 5
3 3 3
[24]

279. (a) correct end points (A1)(A1)


max = 27, min = 4
range = 23 A1 N3 3

(b) Graph 3 A2 N2 2
[5]

280. (a) Σfx = 1(2) + 2(4) + ... + 7(4), Σfx = 146 + 5x (seen anywhere) A1

evidence of substituting into mean =


 fx
(M1)
 f
correct equation A1
146  5 x
e.g. = 4.5, 146 + 5x = 4.5(34 + x)
34  x
x = 14 A1 N2

(b) σ = 1.54 A2 N2
[6]

281. (a) (i) evidence of appropriate approach (M1)


e.g. 9 + 25 + 35, 34 + 35
p = 69 A1 N2
(ii) evidence of valid approach (M1)
e.g. 109 – their value of p, 120 – (9 + 25 + 35 + 11)
q = 40 A1 N2

(b) evidence of appropriate approach (M1)

e.g. substituting into


 fx
, division by 120
n
mean = 3.16 A1 N2

(c) 1.09 A1 N1
[7]

282. (a) σ = 1.61 A2 N2

(b) median = 4.5 A1 N1

(c) Q1 = 3, Q3 = 5 (may be seen in a box plot) (A1)(A1)


IQR = 2 (accept any notation that suggests the interval 3 to 5) A1 N3
[6]

283. (a) evidence of using mid-interval values (5, 15, 25, 35, 50, 67.5, 87.5) (M1)
σ = 19.8 (cm) A2 N3

(b) (i) Q1 = 15, Q3 = 40 (A1)(A1)


IQR = 25 (accept any notation that suggests the interval 15 to 40) A1 N3

(ii) METHOD 1
60 % have a length less than k (A1)
0.6 × 200 = 120 (A1)
k 30 (cm) A1 N2

METHOD 2
0.4 × 200 = 80 (A1)
200 – 80 = 120 (A1)
k = 30 (cm) A1 N2
(c) l < 20 cm  70 fish (M1)
70
P(small) = (= 0.35) A1 N2
200

(d)
Cost $X 4 10 12
P(X = x) 0.35 0.565 0.085
A1A1 N2

(e) correct substitution (of their p values) into formula for E(X) (A1)
e.g. 4 × 0.35 + 10 × 0.565 + 12 × 0.085
E(X) = 8.07 (accept $8.07) A1 N2
[15]

284. (a) 18 A1 N1

(b) (i) 10 A2 N2

(ii) 44 A2 N2
[5]

285. (a) evidence of using ∑ fi = 100 (M1)


k=4 A1 N2

(b) (i) evidence of median position (M1)


e.g. 50th item, 26 + 10 + 20 = 56
median = 3 A1 N2

(ii) Q1 = and Q3 = 5 (A1)(A1)


interquartile range = 4 (accept 1 to 5 or 51, etc.) A1 N3
[7]

286. (a) (i) p = 65 A1 N1


(ii) for evidence of using sum is 125 (or 99  p) (M1)
q = 34 A1 N2
(b) evidence of median position (M1)
125
e.g. 63rd student,
2
median is 17 (sit-ups) A1 N2

(c) evidence of substituting into


 f ( x) (M1)
125
1511 1621 1733 1834  1918  208 2176
e.g. ,
125 125
mean = 17.4 A1 N2
[7]

287. (a) median m = 32 A1 N1

(b) lower quartile Q1 = 22, upper quartile Q3 = 40 (A1)(A1)


interquartile range = 18 A1 N3

(c)
Time (minutes) Number of students
0 ≤ t < 10 5
10 ≤ t < 20 11
20 ≤ t < 30 20
30 ≤ t < 40 24
40 ≤ t < 50 14
50 ≤ t < 60 6
A1A1 N2
[6]
Ú
ü288.

(a) Lines on graph (M1)


100 students score 40 marks or fewer. A1 N2

(b) Identifying 200 and 600 A1


Lines on graph (M1)
a = 55, b = 75 A1A1 N1N1
[6]

289. (a) (i) m = 165 A1 N1


(ii) Lower quartile (1st quarter) = 160 (A1)
Upper quartile (3rd quarter) = 170 (A1)
IQR = 10 A1 N3
(b) Recognize the need to use the 40th percentile, or 48th student (M1)
eg a horizontal line through (0, 48)
a = 163 A1 N2
[6]

290. (a) A = 18, B = 19, C = 23, D = 31, E = 36 A1A1A1A1A1 N5


(b) IQR = 12 A1 N1
[6]

291. (a) Correct mid interval values 14, 23, 32, 41, 50 (A1)

Substituting into
 fw
M1
 f
7(14)  12(23)  13(32)  10(41)  8(50)
eg w =
50
1600
w = A1
50
w = 32 (kg) AG N0

(b) METHOD 1
Total weight of other boxes = 1600  50x (A1)
Total number of other boxes = 50  x (A1)
Setting up their equation M1
1600  50 x
eg = 30, 1600  50x = 1500  30x
50  x
x=5 A1 N3
METHOD 2
Let z be the number of other boxes in Class E (accept any symbol
in the working, even including x).
Total weight of other boxes = 1200 + 50z (A1)
Total number of other boxes = 42 + z (A1)
Setting up their equation M1
1200  50 z
eg = 30, 1200 + 50z = 1260 + 30z
42  z
z=3
x=5 A1 N3
(c) Setting up their inequality M1
Correct substitution A1
98  276  416  41 (10  y)  400 1600  41 y
eg  33,  33
50  y 50  y
1600 + 41y  1650 + 33y (A1)
8y  50 (y  6.25) A1
6 A1 N1
Note: If candidates don’t use the mid-interval values, but
assume that all the new boxes weigh the minimum
amount for Class D, award marks as follows:
Setting up their inequality M1
Correct substitution A1
1600  36.5 y
eg  33
50  y
1600 + 36.5y  1650 + 33y (A1)
3.5y  50 (y  14.28...) A1
14 A1 N1
[12]

292. (a)
Age range Frequency Mid - interval value
0  age < 20 40 10
20  age < 40 70 30
40  age < 60 100 50
60  age < 80 50 70
80  age  100 10 90
A1A1 N2
(b) For attempting to find f x (M1)
Correct substitution (A1)
eg 40  10 + ... + 10  90 = 11900
For dividing by 270 (M1)
11900
eg
270
Mean = 44.1 A1 N4
[6]

293. b = 3, c = 3 A1A1 N2
 abcd 
using mean   4 M1
 4 
using range (d  a = 6) M1
a = 2, d = 8 A1A1 N2
[6]

294. (a) (i) r = 10 A2 N2


(ii) s = 13 A2 N2

(b) Using
 x
 10 A1
12
t = 18 A1 N1
[6]

295. (a) D B C A1A1A1 N3


(b) B A C A1A1A1 N3
[6]

296. (a) 3 A1 N1

(b) 6 A2 N2
(c) Recognizing the link between 6 and the upper quartile (M1)
eg 25% scored greater than 6,
0.25  32 (A1)
8 A1 N3
[6]

297. (a) (i) 50 (accept 49, “fewer than 50”) A1 N1


(ii) Cumulative frequency (7) = 90 (A1)
90  50 (M1)
= 40 A1 N2
(iii) 75th or 75.5th person A1
median = 6.25 (min), 6 min 15 secs A1 N1

(b) Evidence of finding 40% (60%) of 150 M1


Number spending less than k minutes is (150  60) = 90 (A1)
k=7 A1 N2

(c) (i)
t (minutes) 0t<2 2t<4 4t<6 6t<8 8  t < 10 10  t < 12
Frequency 10 23 37 38 27 15
A1A1A1 N3
(ii) Evidence of using all correct mid-interval values (1, 3, 5, 7, 9, 11) A1
 110  3 23  5 37  7  38  9  27 1115 
mean =  
 150 
= 6.25 (min), 6 min 15 secs A1 N1
[14]
298.

800

700

600
Number
of 500
candidates
400

300

200

100

10 20 30 40 50 60 70 80 90 100

Mark

(a) Lines on graph (M1)


100 students score 40 marks or fewer. A1 2
(b) Identifying 200 and 600 A1
Lines on graph. (M1)
a = 55, b = 75. A1A1 4
[6]

x  2230 
299. (a) mean     (M1)
n  45 
x  49.6 (Accept 50) (A1) (C2)
(b) y  y (may be implied) (M1)
n2
 y  2230  37  30 (A1)
2297
y (A1)
47
 48.9 (Accept 49) (A1) (C4)
[6]

300. (a) Mean =


fx
f
f x = (1) (0) + (2) (4) + (3) (6) + (4) (k) + (5) (8) + (6)
(6) + (7) (6) (A1)
 f k + 30 (A1)
144  4k
Using mean 4.6 = (M1)
k  30
4.6k + 138 = 144 + 4k (A1)
0.6k = 6
k = 10 (A1) (C5)

(b) Mode = 4 (A1) (C1)


(accept 5, if k < 8 )
[6]

301. (a) 76 (mice) (A1) (N1)

(b) 11.2 (seconds) (A1) (N1)


(c) (i) p = 76  (16 + 22) = 38 (allow ft from (ii) (a)) (A1) (N1)
q = 132  76 = 56 (A1) (N1)
7.5  16  .....14.5  23  3363 
(ii) x=   (M1)
16  ...23  300 
= 11.2 (accept 11.21) (A1) (N2)
[6]

302. (a)
Mark (x) 0 ≤ x < 20 20 ≤ x < 40 40 ≤ x < 60 60 ≤ x < 80 80 ≤ x < 100
Number of
22 50(±1) 66(±1) 42(±1) 20
Students
(A1)(A1)(A1) (C3)

(b) 40th Percentile  80th student fails, (mark 42%) (M2)


Pass mark 43% (Accept mark > 42.) (A1) (C3)
[6]

303. List of frequencies with p in the middle


23  p
eg 5 + 10, p, 6 + 2  15, 8, or 15 < , or p > 7. (M1)
2
Consideration that p < 10 because 2 is the mode or discretionary for
further processing. (M1)
Possible values of p are 8 and 9 (A2)(A2) (C6)
[6]

304. (a) line(s) on graph (M1)


median is 183 (A1) (C2)

(b) Lower quartile Q1  175 (A1)


Upper quartile Q3  189 (A1)
IQR is 14
(Accept 189 – 175, 175 to 189, 189 to 175 and 175 – 189) (M1)(A1) (C4)
[6]
305. d = 11; c = 11 (A1)(A1)(C1)(C1)
d – a = 8 (or 11 – a = 8) (A1)
a3 (A1) (C2)
3  b  11  11  sum 
 8  or  8 (A1)
4  4 
b=7 (A1) (C2)
[6]

306.
x f f
4 2 2
5 5 7
6 4 11
7 3 14
8 4 18
10 2 20
12 1 21
(a) m=6 (A2) (C2)

(b) Q1 = 5 (A2) (C2)

(c) Q3 = 8 (A1)
IQR = 8 – 5 (M1)
= 3 (accept 5 – 8 or [5, 8]) (C2)
[6]
307.
90
80

Cumulative frequency
70
60
50
40
30
20
10
0
0 5 10 15 20 25 30 35 40 45
LQ = 14 M = 20 UQ = 24
Diameter (mm)
(a) (i) Correct lines drawn on graph, (A1)(C1)
median = 20 (A1)(C1)

(ii) Correct lines drawn on graph, (A1)(C1)


UQ = Q3 = 24 (A1)(C1)

(b) IQR = Q3 – Q1(or UQ – LQ) (M1)


= 10 (accept 14 to 24) (A1) (C2)
Note: Accept 14 to 24, 24 to 14, 14 – 24
or 24 –14.
[6]

308. Jan–Sept  = 630 × 9 = 5670 (M1)(A1)


Oct–Dec  = 810 × 3 = 2430 (M1)(A1)
5670  2430
x (M1)
12
mean = 675 (A1) (C6)
[6]

309. (a) (i) median fare = $24 (±0.5) (A1)


(ii) fare  $35 => number of cabs is 154 (or 153) (A1) 2

(b) 40% of cabs = 80 cabs (A1)


fares up to $22 (A1)
distance = $22 ÷ $0.55 (M1)
a = 40 km (A1) 4
(c) Distance 90 km => fare = 90 × $0.55
= $49.50 (A1)
Fare $49.50 => number of cabs = 200 –186 (M1)
= 14 (A1)
14
Thus percentage is = 7% (A1) 4
200
[10]

310. Median = middle value => b = 11 (A1)


a  b  c a  11  c
Mean =  = 9 => a + 11 + c = 27 (M1)
3 3
=> a + c = 16 (A1)
Range = c – a = 10 (M1)(A1)
Solving equations simultaneously gives a = 3 (A1) (C6)
[6]
311. (a)

Houses
100
91±1

90

80 75

70

60 350 000

50

40

30
25

20
135±5

240±5

10

100 200 300 400 500 Selling price ($1000)


(A1)(A2)(A1) 4
Notes: Award (A1) for correct axes, scales and labelling, (A1)
for correctly plotted points.
Award (A2) for good curve correctly drawn, (A1) for badly
drawn, correct curve.
Award (A1) for a correct polygon.
(b) Q1 = 135  5 Q3 = 240  5 (M1)(A1)
Interquartile range = 105  10. (Accept 135 – 240 or 240 – 135.) (A1) 3
Note: Award (M1) for the correct lines on the graph.
(c) a = 94 – 87 = 7, b = 100 – 94 = 6 (A1)(A1) 2
1250  46150  29250  7350  6450
(d) mean = (M1)
100
= 199 or $199000 (A1)
OR
mean = 199 or $199000 (G2) 2
(e) (i) $350000 => 91.5
Number of De luxe houses 100 – 91.5 (M1)
= 9 or 8 (A1)
6  5  5 6  5  15
(ii) P (both > 400000) =    or    (M1)(A1) 4
9  8  12 8  7  28
4 9
Note: Award (M1)(A0) for the answers or obtained
9 16
from correct independent probabilities.
[15]
312. (a) Median = middle number of 75 (M1)
= 38th number
=4 (A1) (C2)
5  18  48  72  100  42
(b) Mean = (M1)
75
285
=
75
= 3.8 (A1) (C2)
[4]
313. (a) s = 7.41(3 sf) (G3) 3
(b)
Weight (W) W ≤ 85 W ≤ 90 W ≤ 95 W ≤ 100 W ≤ 105 W ≤ 110 W ≤ 115
Number of 5 15 30 56 69 76 80
packets
(A1) 1
(c) (i) From the graph, the median is approximately 96.8.
Answer: 97 (nearest gram). (A2)
(ii) From the graph, the upper or third quartile is approximately 101.2.
Answer: 101 (nearest gram). (A2) 4
(d) Sum = 0, since the sum of the deviations from the mean is zero. (A2)
OR
  Wi 
 
(Wi  W )  Wi   80
 80 
 =0 (M1)(A1) 2
 
(e) Let A be the event: W > 100, and B the event: 85 < W ≤ 110
P( A  B )
P(AB) = (M1)
P(B)
20
P(A  B) = (A1)
80
71
P(B) = (A1)
80
P(AB) = 0.282 (A1)
OR
71 packets with weight 85 < W ≤ 110. (M1)
Of these, 20 packets have weight W > 100. (M1)
20
Required probability = (A1)
71
= 0.282 (A1) 4
Notes: Award (A2) for a correct final answer with no
reasoning.
Award up to (M2) for correct reasoning or method.
[14]
314. (a) (Using mid-intervals)
65(7)  75(25)  ...  135(5)
v (M1)
7  25  ...  5
29450
= = 98.2 km h–1 (A1)
300
OR
v = 98.2 (G2) 2
(b) (i) a = 165, b = 275 (A1)
(ii)

320

CUMULATIVE FREQUENCY
280
255 Cars
Number of cars

240

200

160

120

80

40 105 kmh –1

20 40 60 80 100 120 140 v (kmh–1)


(A4) 5
Note: Award (A1) for properly marked scales and axes,
(A2) for 9 correctly plotted points, (A1) for 7 or 8 points,
(A1) for a smooth curve through the points.
(c) (i) Vertical line on graph at 105 km h–1 (M1)
300  200
× 100% = 33.3(±1.3%) (A1)
300
OR
33.3(±1.3%) (A2)
(ii) 15% of 300 = 45 300 – 45 = 255
Horizontal line on graph at 255 cars (M1)
Speed = 114(± 2 km h–1) (A1)
OR
Speed = 114(± 2 km h–1) (A2) 4
[11]
315. (a) (i) 10 (A1)
(ii) 14 + 10 = 24 (A1) 2
(b)
xi fi
 15 1 
 25 5 
 35 7 
 45 9 
 
(A1) 55 10 (A1)
 65 16 
 75 14 
 85 10 
 
 95 8 
80 (AG)
Note: Award (A0) for using the mid-interval values of 14.5,
24.5 etc.
(i)  = 63 (A1)
(ii)  = 20.5 (3 sf) (A1) 4
(c) Assymetric diagram/distribution (A1) 1
(d)

60

cumulative
frequency (A1)
40.5

(A1){ points and


curve
20

50 60 65 70 80
length (cm) (A1) 3
OR Median = 65 (A3) 3
Note: This answer assumes appropriate use of a calculator with
correct arguments.
OR Linear interpolation on the table: (M1)
 48  40.5   40.5  32 
   60    × 70 = 65 (2sf) (A1)(A1) 3
 48  32   48  32 
[10]
316. (a) x = $59 (G2)
OR
10  24  30 16  ...  110 10  130  4
x (M1)
24  16  ...  10  4
7860
=
134
= $59 (A1) 2
(b)
Money ($) <20 <40 <60 <80 <100 <120 <140
Customers 24 40 62 102 120 130 134 (A1)

140

120
number
of 100
customers
80

60

40

20

20 40 60 80 100 120 140

money ($) (A4) 5


Note: Award (A1) for the correct scale, (A1) for the points, and
(A2) for the curve.
(c) (i) t = 2d2/3 + 3
Mean d = 59 (M1)
Mean t  2 × (59)2/3 + 3 (M1)
 33.3 min. (3 sf) (accept 33.2) (A1)
(ii) t > 37  2d2/3 + 3 > 37 (M1)
2d2/3 > 34
d2/3 > 17 (A1)
d > (17)3/2
d > 70.1
From the graph, when d = 70.1, n = 82 (A1)
number of shoppers = 134 – 82 (A1)
= 52 (A1) 8
[15]
(10  1)  (20  2)  (30  5)  (40  k )  (50  3)
317. = 34 (M1)(A1)
k  11
40 k  350
= 34 (A1)
k  11
k=4 (A1) (C4)
[4]
318. (a)
x 15 45 75 105 135 165 195 225
f 5 15 33 21 11 7 5 3 (M1)
_
x = 97.2 (exactly) (A1) 2
(b)
x 30 60 90 120 150 180 210 240
Σf 5 20 53 74 85 92 97 100 (A1) 1
Note: Award (A1) for correct values for x, f.
(c)

100
f 90
80
70
60
customers

50
40
30
20
10 Q3
a n
20 40 60 80 100 120 140 160 180 200 220 240

(A4) 4
Notes: Award (A2) for 6 or more points correct, (A1) for 4/5
points correct.
Award (A1) for a reasonable graph, (A1) for the correct axes
and the given scales.
(d) Median = 87 ± 2 (A1)
Lower quartile = 65 ± 2 (A1)
Upper quartile = 123 ± 2 (A1) 3
[10]
319. (a)
Mark  10  20  30  40  50  60  70  80  90  100
No. of Candidates 15 65 165 335 595 815 905 950 980 1000
(A3) 3
Note: Award (A1) for 165, (A1) for 1000, (A1) if all other
entries are correct.
(b)

1000

900

800

700

600
No. of candidates

500

400

300

200

100

10 20 30 40 50 60 70 80 90 100
Marks
(A5) 5
Notes: Vertical axis and scale (A1)
Horizontal axis and scale (A1)
Points (A1)
Curve (allow polygon) (A2)
(c) (i) Median = 46 (M1)(A1)
(ii) Scores < 35: 240 candidates (M1)(A2)
(iii) Top 15%  Mark  63 (M1)(A1)(A1) 8
Notes: Accept the answers from the student’s graph.
In each part, award (M1) for the dotted lines on the graph.
[16]
(72  1.79)  (28  1.62)
320. Mean = (M1)(M1)(M1)
100
= 1.7424 (= 1.74 to 3 sf) (A1) (C4)
[4]
300
321. (a) m= (M1)
25
= 12 (A1) (C2)
 625 
(b) s=   (M1)
 25 
=5 (A1) (C2)
[4]

Potrebbero piacerti anche